SCE Practicaldiabetes - Elzohry
SCE Practicaldiabetes - Elzohry
SCE Practicaldiabetes - Elzohry
StudyPRN
Endocrinology and
Diabetes – SCE
Revision
Khalid Yusuf
Sohag Teaching Hospital
(Egypt)
[email protected] 2017
FB: Sohag Endocrine Group
StudyPRN (Endocrinology and Diabetes – SCE Revision)
https://www.studyprn.com
Adrenal 30 1-30
Gonads 29 97-125
Other 88 249-336
Parathyroid/calcium 21 126-146
Thyroid 56 147-202
Total: 478
Which of the following genetic tests would you 3- Refer for adrenal surgery
request?
4- Repeat MRI scan in 3-6 months
(Please select 1 option)
5- Reassure and discharge
1- BRAC
2- FGF 1 Answer & Comments
3- NF type 1 Answer: 4- Repeat MRI scan in 3-6 months
4- NF type 2
From investigations this mass is measured at 2cm
5- SDH type B/D
in diameter. There is no evidence of functionality
of this tumour from testing either. It does not
Answer & Comments meet the size criteria of >4cm for consideration of
surgical removal. The only concern is that this may
Answer: 5- SDH type B/D
be an adrenal carcinoma and therefore the patient
should be re-scanned in approximately 3 months
Phaeochromocytoma susceptibility genes include looking for significant size changes. If there are no
Succinate dehydrogenase types BCD, VHL and RET size changes in the interval there is no evidence to
protooncogene (MEN II). The phenotypic support further radiological imaging. FNA does not
characteristics are likely to be present for NF 1 and provide any useful differentiating information
so this is not the most appropriate investigation in between carcinoma and benign disease and
this patient. All patients with a phaeo diagnosed should not be undertaken as, if this were a
before 45 years of age should be investigated carcinoma FNA could result in seeding.
further for a potential genetic cause for their
Phaeo.
[ Q: 7 ] Question ID #88
4- Treat with metyrapone This patient has an adrenal carcinoma which is the
cause of Cushing’s syndrome. Surgery has
5- Treat with mitotane
effectively treated the hyperocrtisolism and the
patient now requires steroid replacement.
Answer & Comments Although at this stage the patient has no evidence
of metastases, further serial scanning will be
Answer: 1- Refer for bilateral adrenalectomy required. The prognosis associated with adrenal
carcinoma is usually very poor although there is
This patient has Cushing’s syndrome due to emerging evidence that the prognosis may be
bilateral adrenal nodular enlargement which is improved with adjuvant mitotane therapy.
independent of ACTH. This could be due to food-
induced Cushing’s and a meal test may be [ Q: 9 ] Question ID #131
appropriate but the condition of ACTH-
independent macronodular adrenal hyperplasia is
described and seems the more probable An 18 year old female who presents with
explanation. The most appropriate treatment oligomenorrhoea is found to have non-classical
would be congenital adrenal hyperplasia. She enquires as to
the risk of any of her future children suffering the
disorder.
[ Q: 8 ] Question ID #91
Which of the following is correct concerning this
A 55 year old female is confirmed to have adrenal risk?
dependent Cushing’s syndrome following (Please select 1 option)
presentation with a 6 month history of weight
gain, weakness and thinning of the skin. CT 1- Virtually 0%
abdomen has demonstrated a 9cm left adrenal
2- 1%
tumour and no other abnormalities are noted on
this scan. An adrenalectomy is performed and 3- 5%
histology reports an adrenal carcinoma.
4- 25%
Post-operative assessment reveals that the 5- 50%
patient is steroid dependent and is prescribed
10mg of hydrocortisone bd. She then undergoes a
Answer & Comments
1mg overnight dexamethasone test and her 9am
cortisol is measured at 22 nmol/l (150-800). Answer: 2- 1%
Answer: 4- Mitotane
A 45 year old male is investigated for possible gall A 40-year-old man was referred to the endocrine
stones and undergoes an ultrasound which as well clinic for his resistant hypertension. He is currently
as displaying some gall stones reveals a possible taking atenolol 50 mg od, bendrofluazide 2.5 mg
adrenal lesion. Further imaging with CT confirms od, doxazosin 4 mg od, amlodipine 10 mg od
the presence of gall stones but also reveals a 2cm and ramipril 10 mg od.
right adrenal lesion. On endocrine review, the
patient has no symptoms to suggest functionality On examination, his pulse was 64 beats per
of this lesion but has been taking atenolol 25 mg minute, his blood pressure was 162/96 mmHg,
daily for hypertension. Investigations reveal: heart sounds were normal and there were no
cushingoid features.
24 hour Urine fee
148 nmol/d (<290) Investigations
cortisol
220 nmol/d (120-
Urine Noraderaline Serum Urea 4.4mmol/L (2.5-7.5)
590)
Urine Adrenaline 110 nmol/d (30-190) Serum Creatinine 88 µmol/L (60-110)
Urine Metadrenaline 5.2 micromol/l (0-2) Serum Potassium 3.06 mmol/L(3.5–4.9)
2.1 micromol/l (0-
Urine normetadrenaline Which is the most important medication to be
5.3)
discontinued before testing his
Plasma Renin Activity 1.2 pmol/min (0.5-2) aldosterone:plasma renin activity ratio?
400 pmol/l (150- (Please select 1 option)
Aldosterone
800)
1- Amlodipine
Which of the following is the likely diagnosis? 2- Atenolol
(Please select 1 option)
3- Bendrofluazide
1- Adrenal carcinoma
4- Doxazosin
2- Aldosteronoma
5- Ramipril
3- Cushing's syndrome
4- Incidentaloma with factitious results Answer & Comments
Which of the following antibodies would be the usually used during the preoperative management
most appropriate to check in this patient? of patients awaiting surgery or if surgical
(Please select 1 option)
treatment is contraindicated as a palliative
measure. Nausea, hypertension and features of
1- Anti-21-hydroxylase antibody hirsutism are the major associated side effects.
The dose titration of drug is usually based on
2- Anti-intrinsic factor antibody
cortisol day curve. In children Ketoconazole is
3- Anti-peroxidase antibody preferred over Metyrapone as a preoperative
medication (while awaiting adrenal surgery).
4- Anti-rho antibody
5- Anti-TTG antibody [ Q: 16 ] Question ID #643
5- 21 ß hydroxylase
Answer & Comments
Answer: 3- 11 ß hydroxylase
Gitelman`s syndrome is characterised by mutation
in the SCL12A3 gene that encodes for a Na-Cl
Metyrapone is a competitive inhibitor of 11 ß transporter at distal convoluted tubule. As a result
hydroxylase enzyme (CYP11B1) which is expressed the affected individuals develop Na loss (salt
in Zona fasciculata and Zona glomerulosa. This wasting), hypovolemia, hypocalciuria,
enzyme is required for conversion of deoxycortisol hypomagnesaemia and metabolic alkalosis. The
to cortisol and its inhibition leads to decrease in fluid loss prompts activation of Renin-Angiotensin
cortisol concentration and an increase in ACTH system leading to increase in both renin and
through feedback mechanism. Metyrapone is
1- Decreased weight
The 2004 WHO classification of endocrine 2- Improved sense of well being
tumours classifies catecholamine secreting
tumours into intra-adrenal paragangliomas 3- Increase in muscle mass
(classically known as phaeochromocytomas) and 4- Increase in fertility
extra-adrenal paragangliomas. The later are
further subdivided into those arising from 5- Maintenance of blood pressure
sympathetic tissue or parasympathetic tissues
(usually located in head and neck, rarely produce Answer & Comments
catecholamine). Hereditary catecholamine
producing intra or extra- adrenal paragangliomas Answer: 2- Improved sense of well being
can be caused by one of the following germ line
mutations:
Dehydroepiandrosterone sulphates (DHEAS) are
the androgens secreted by adrenal glands which
1. RET proto-oncogene decline with age. These were commonly termed as
2. von Hippel-Lindau gene (vHL) elixir of youth with alleged role in general well
3. Neurofibromatosis Type 1 gene (NF-1) being, cognitive and sexual function and as a
4. Succinate dehydrogenase subunit B
weight loss therapy. Randomised controlled trials
(SDHB) have failed to demonstrate any apparent benefit
5. Succinate dehydrogenase subunit D with DHEAS in patients with normal adrenal
(SDHB) function. These may play a role in female patients
with adrenal insufficiency by increasing a sense of
Patients with SDHB or SDHD are more likely to well being by bringing about a modest
present with extra-adrenal disease, bilateral improvement in sexual function.
tumours and metastatic disease. According to the
guidelines issued by The First International
Symposium on Phaeochromocytomas (October [ Q: 25 ] Question ID #655
2005), genetic screening should be individualised
and is generally warranted in: A 65 year old woman is investigated lady for an
assumed secondary hypertension. She has been
Patients aged < 50 years, presenting with found to have a blood pressure of 165/95mmHg.
catecholamine secreting There is little to find on examination and her BMI
Multiple tumours is 26.4 kg/m2. Currently she takes amlodipine
Malignant tumours 10mg daily. Results of her investigations are
Bilateral tumours shown below.
Answer: 1- Adrenal CT
A 70 year old male has a history of hypertension
which is adequately controlled with doxazosin yet
This patient is likely to have Conn's syndrome. She has persistent hypokalaemia. The patient is
has an elevated PRA:Aldosterone ratio ~1500 but referred to clinic as the potassium is persistently
has a normal potassium that may confound some between 2.6-3.1 mmol/l. Investigations show a
people. About 20% of patients with Conn's Plasma renin activity:aldosterone ratio of 780.
syndrome may have a normal potassium. Primary
Hyperaldosteronism accounts for as much as 10% Which of the following is the most appropriate
of patients diagnosed to have essential management option for him?
hypertension in one secondary care hypertension
(Please select 1 option)
clinic. Screening for excess aldosterone
production is warranted in the following group of 1- Switch Doxazosin to an ACE inhibtor
patients:
2- Start a Potassium supplement
1. Young hypertensive (age <40> 3- Switch Doxazosin to Spironlactone
2. Resistant hypertension (unresponsive to 3
or more antihypertensive agents) 4- Switch Doxazosin to a Thiazide diuretic
3. Associated hypokalaemia 5- Stop Doxazosin
4. Incidental adrenal tumour
Answer & Comments
Measurement of aldosterone: renin ratio is the
best initial diagnostic test. Increased aldosterone Answer: 2- Start a Potassium supplement
and suppressed renin levels are highly indicative of
underlying primary aldosteronism. A number of
medications interfere with test results and should The initial screening test for the diagnosis of
ideally be stopped. The following interfere with primary aldosteronism is measurement of a
plasma renin activity to aldosterone ratio. A ratio cortisol levels during an acute illness are not truly
above 1000 is highly suggestive of primary reflective of adrenal hyper-secretion and a
hyperladosteronism. (In the UK this is used in the suspected adrenal excess cortisol values should
states they often quote the other way around, only be evaluated (if clinically warranted) once the
Aldo:renin ratio. ARR. A ratio of >20 is suggestive patient shows clinical improvement.
of diagnosis of primary aldosteronism). However
false positives and negatives can occur. Some
[ Q: 28 ] Question ID #663
medications can interfere with aldosterone/ renin
levels. Doxazosin has no effect. Also if the patient
is hypokalaemic, aldosterone concentrations can What is the best investigation to monitor a patient
fall and marginal increases in renin can make for a with classic salt wasting congenital adrenal
false negative result. Oral potassium supplements hyperplasia (CAH)?
are given to avoid any false negative test results. A (Please select 1 option)
false negative result can also be obtained in
1- Check 17 hydroxy progesterone (17 OHP)
patients with chronic renal failure due to
persistently elevated renin levels. levels
2- DHEA levels
[ Q: 27 ] Question ID #661 3- Monitor 21 alpha hydroxylase
4- Measure Plasma renin activity
A 38 year old man is admitted after an RTA leading
to head trauma. He is being managed in the 5- USG adrenals
intensive care unit. On day 2 of admission his 24
hour Urinary free cortisol values are noticed to be
Answer & Comments
high. You are asked to give opinion regarding
further assessment for his possible cortisol excess Answer: 1- Check 17 hydroxy progesterone (17
secretion. OHP) levels
A 24 year old presents with hypertension, You are called via the arrest bleep to a 43 year old
hypokalaemia, suppressed renin and aldosterone woman who is on the surgical ward. She had a
values is suspected to have apparent laparoscopic cholecystectomy performed two
mineralocorticoid excess. days ago and has since had nausea and vomiting
and has been receiving IV dextrose. It transpires
Which of the following tests can be used for that she had a fit for which the nursing staff
establishment of her diagnosis? requested the arrest team. She is now drowsy. Her
past medical history is unremarkable.
(Please select 1 option)
1- Check 3 β hydroxy steroid dehydrogenase (3 On examination, she is drowsy with a GCS of 13.
β HSD) enzyme levels Her pulse is 88 bpm and blood pressure 132/88
mmHg with oxygen sats of 100% on air. There is no
2- CT abdomen
focal neurology and her plantar responses are
3- Deoxycortisol levels equivocal.
4- Measure urinary cortisol to cortisone ratio
Of note, her blood results reveal:
5- Serum cortisol
Sodium 118 mmol/l (135-144)
Answer & Comments Potassium 3.5 mmol/l (3.5-5)
Answer: 4- Measure urinary cortisol to cortisone Urea 3.8 mmol/l (3-8)
ratio Creatinine 70 micromol/l (50-100)
Glucose 4.1 mmol/l (3.5-6)
Presence of hypertension, hypokalaemia and a
suppressed renin and aldosterone values are Which of the following is the most appropriate
suggestive of apparent mineralocorticoid excess initial treatment for this patient?
seen due to deficiency of 11β hydroxy steroid (Please select 1 option)
dehydrogenase (11β HSD) enzyme in this patient.
This enzyme is mediates the conversion of cortisol 1- 15mg oral Tolvaptan
to cortisone locally to protect mineralocorticoid 2- 100mg IV Hydrocortisone stat
receptor.
3- 150mls of 3% hypertonic saline over 20 mins
Deficiency of 11 β HSD leads to hampered
4- 1litre Normal saline infused over 4 hrs
conversion of cortisol (which is a more potent
mineralocorticoid as compared to cortisone) to 5- Fluid restriction to 500mls per day
cortisone locally, resulting in a state of apparent
mineralocorticoid excess. The patients usually
Answer & Comments
present with hypertension, hypokalaemia and
metabolic alkalosis in presence of suppressed Answer: 3- 150mls of 3% hypertonic saline over 20
levels of aldosterone and renin. The diagnosis can mins
be confirmed by measurement of urinary
tetrahydrocortisol to cortisone ratio (increased
This patient is likely to have hyponatraemic
levels of tetrahydrocortisol as compared to
encephalopathy which is life threatening and has
cortisone).
presented with a fit. Post-operative
hyponatraemia is not uncommon and is likely to
be due to a combination of SIADH which develops
in the post-op period and the infusion of
inappropriate IVIs. The European Endocrine
Which of the following is the likely diagnosis? 1- Exocrine pancreatic insufficiency, reduced
glucagon and insulin secretion with
(Please select 1 option)
increased hepatic glucose production and
1- Alport syndrome increased insulin resistance
2- Alstrom syndrome 2- Exocrine pancreatic insufficiency, reduced
3- Kearns-Sayre syndrome insulin secretion, increased glucagon
secretion with increased insulin resistance
4- MELAS syndrome
3- Exocrine pancreatic insufficiency, reduced
5- Wolfran Syndrome
insulin and glucagon secretion with reduced
hepatic glucose production and increased
Answer & Comments insulin sensitivity
Answer: 5- Wolfran Syndrome 4- Normal pancreatic exocrine function,
reduced insulin secretion, increased
This patient expresses the clinical features of The glucagon secretion with reduced insulin
DIDMOAD or Wolfram syndrome, characterised sensitivity
by, Diabetes Insipidus (the patient takes
Desmopressin), Diabetes Meliitus of juvenile 5- Exocrine pancreatic insufficiency, reduced
onset, Optic Atrophy and Deafness. Diabetes insulin secretion, normal glucagon
occurs in nearly all patients presenting with this secretion, increased hepatic glucose
syndrome. While little is known around the production with reduced insulin sensitivity.
specific cellular mechanisms, pancreatic islets
from such patients demonstrate a selective
syndrome being associated with in impaired lipid profile. Increasing the statin dose or
glucose tolerance and IUGR. Glycogen storage commencing a fibrate is not appropriate as
disease is a condition associated with impaired hypothyroidism is a risk factor for lipid lowering
glucose tolerance, hepatomegaly and may be a therapy-associated myotoxicity. Omega-3 fish oils
cause of early hypoglycaemia. are not routinely recommended for use in this
setting.
[ Q: 34 ] Question ID #18
[ Q: 35 ] Question ID #19
A 42 year old South Asian man is referred to the
diabetic clinic by his GP. He has a 5 year history of A 33 year old lady with a 10 year history of type 1
type 2 diabetes. He complains of feeling tired and diabetes has been converted from twice daily
has a previous history of ischemic heart disease. Novomix to a basal bolus insulin regimen
On examination his BMI is 34 kg/m2, his pulse is 51 comprising of insulin glargine and novorapid three
bpm and his blood pressure is 148 / 78 mmHg. His months previously at a total daily dose of 72 units.
current medication includes ‘Novomix 30’ 42 units On review in the clinic her glycaemic control has
twice daily, metformin 500 mg twice daily, improved since her insulin regimen change with
simvastatin 20 mg once daily, aspirin 75 mg once her current Hba1c measuring 8.1 % (<6) compared
daily, enalapril 20 mg once daily and atenolol 50 with 8.9% (<6) prior to her regimen change. She is
mg once daily. however unclear as to how to modify her prandial
insulin doses according to her diet.
Investigations reveal:
Which of the following is the most appropriate
HbA1c 7.9% (<6) initial advice for this lady with respect to adjusting
Total Cholesterol 5.6 mmol/l (<5) her prandial insulin dose?
(Please select 1 option)
LDL-Cholesterol 3.7 mol/l (< 3)
Plasma Triglyceride 2.2 mmol/l (0.9 - 2.2) 1- 0.5 units of insulin per 10 gram of dietary
carbohydrate
TSH 16.8 mu / l (0.32 - 5.5)
Free T4 7.9 pmol/l (9.4 - 23.4) 2- 1 units of insulin per 10 gram of dietary
carbohydrate
Which of the following is the most appropriate 3- 2 units of insulin per 10 grams of dietary
approach to optimise this patient’s lipid profile? carbohydrate
(Please select 1 option)
4- 3 units of insulin per 10 grams of dietary
1- Commence fenofibrate carbohydrate
2- Commence Omega-3 fish oil 5- 4 units of insulin per 10 grams of dietary
3- Commence thyroxine carbohydrate
subsequently be modified according to response His most recent Hba1c is 6.2% (<6). His main
and the influence of other factors such as exercise. complaint is that of feeling continually hungry and
he has experienced pre-syncopal like symptoms
[ Q: 36 ] Question ID #20 associated with capillary blood glucose
measurements of between 2.5 and 3.5mmol/l.
insulin therapy. Patients with type 2 diabetes There must be appropriate blood glucose
receiving insulin for > 5 years, particularly when monitoring.
the regimen contained a prandial insulin element
experienced a similar prevalence of Must not be regarded as a likely source of
hypoglycaemia to that observed in patients with danger to the public while driving.
type 1 diabetes.
The visual standards for acuity and visual
[ Q: 38 ] Question ID #26 field must be met.
Answer & Comments Fibrates are PPAR alpha receptor agonists and
through this route are responsible for reduced
Answer: 1- Atorvastatin 20 mg od hepatic triglyceride secretion, increased
lipoprotein lipase activity and increased clearance
Current NICE lipid lowering guidelines(July 2014) - of remnant LDL particles. These class of drugs are
offer atorvastatin 20 mg for the primary particularly useful in reducing triglycerides and
prevention of CVD to people who have a 10% or increasing HDL – the typical dyslipidaemia
greater 10-year risk of developing CVD. Estimate associated with diabetes.
the level of risk using the QRISK2 assessment tool.
Lipid lowering therapy for patients with diabetes [ Q: 41 ] Question ID #122
under the age of 40 should be considered when
there is an associated adverse cardiovascular risk
factor profile such as co-existing poor glycaemic A 56 year old male is commenced upon
control, the presence of microvascular Vildagliptin.
complications, features of metabolic syndrome
and family history of premature cardiovascular Through which of the following mechanisms does
disease. This patient should thus be considered for Vildagliptin function?
lipid lowering therapy with atorvastatin as he has (Please select 1 option)
evidence of microvascular complications in the
form of microalbuminuria and retinopathy 1- DPP-IV inhibitor
combined with poor glycaemic control. This 2- GIP agonist
guidance is based on data from all of the major
statin studies in which the relative cardiovascular 3- GLP-1 agonist
risk reduction associated with cholesterol 4- Potassium channel blocker
lowering occurs independent of baseline
cardiovascular risk, being related to the order of 5- PPAR-gamma agonist
magnitude of cholesterol reduction.
Answer & Comments
[ Q: 40 ] Question ID #108 Answer: 1- DPP-IV inhibitor
were bounding peripheral pulses and no ulcer was 3- Laser treatment is 90% effective in
observed. A digital foot thermometer preventing loss of vision
measurement showed that the temperature was
20 degree centigrade on the left foot and 26 4- Recurrent laser treatments will not affect her
degree centigrade on the right foot. ability to drive
5- She will need to have her eye immobilised to
Which of the following is the most likely diagnosis undergo laser treatment
for this lady?
(Please select 1 option)
Answer & Comments
1- Abscess of the right foot
Answer: 3- Laser treatment is 90% effective in
2- Charcot neuroarthropathy preventing loss of vision
3- Diabetic foot osteomyelitis
According to the Early Treatment of Diabetic
4- Pes planus
Retiopathy study laser treatment is 90% effective
5- Traumatic fracture of the right foot unaware in preventing loss of vision in type 1 diabetes but
by the patient due to neuropathy only 50% in preventing loss of vision if the
predominant type of retinopathy is macular
oedema (mostly seen in type 2 diabetes). Pan-
Answer & Comments retinal treatment is often associated with loss of
Answer: 2- Charcot neuroarthropathy the peripheral visual field which may affect her
ability to drive if both eyes are treated although
this is usually only an issue after several
This patient with diabetic neuropathy presented treatments. She will not need her eye to be
with insidious onset discomfort, mid-foot immobilised but will need to be able to hold it still
deformity and signs of inflammation, which is a during treatment.
common presentation of charcot
neuroarthropathy. Gradual onset and signs of
inflammation is not consistent with a traumatic [ Q: 44 ] Question ID #201
fracture and again is characteristic feature of
charcot neuroarthropathy. A 38 year old lady with 21 year history of type 1
diabetes has a most recent HbA1c of 7.5% (<6).
[ Q: 43 ] Question ID #199 Her current therapy includes Insulin glargine and
humalog via a basal bolus regimen at a total daily
insulin dose of 79 units. She has no clinical
A 28 year old woman with type 1 diabetes for 14 evidence of microvascular complications although
years has poor glycaemic control with an HbA1c of her most recent digital retinal screen
12.4% (< 6). She has pre-proliferative (severe non- demonstrated the presence of background
proliferative) diabetic retinopathy and you are retinopathy in the right eye.
going to refer her to an ophthalmologist for
assessment. Which of the following lesions would be expected
in this retinal image to classify this as mild non-
What advice would you give her when she enquires
proliferativec (background) diabetic retinopathy?
about the likelihood of needing laser treatment?
(Please select 1 option)
(Please select 1 option)
1- Blot haemorraghes
1- Laser treatment will only be necessary if she
develops proliferative retinopathy 2- Branch retinal vein occlusion
examination of her feet reveals reduced vibration, 2- There is a 2 - 3 fold increase risk of Down
proprioception and thermosensation in both feet. syndrome
Retinal screening reveals evidence of
neovascularisation in the left eye. 3- There is a 2 - 4 fold increase risk of neural
tube defects
Which of the following factors has been shown to 4- There is a 2 - 4 fold risk of the baby
have an important role in regulating retinal subsequently developing type 1 diabetes
capillary blood flow?
5- Shoulder dystocia occurs in 15 - 20% of cases
(Please select 1 option)
At which level of eGFR would you advise the GP to Total Cholesterol 4.4 mmol/l (<5)
discontinue metformin therapy? 2.7 mmol/l (0.9
Plasma Triglyceride
- 2.2)
(Please select 1 option)
U+E Normal
1- eGFR ≤ 15 ml/min/1.72m2
Urine Albumin:Creatinine 9.9 mg/mmol
2- eGFR ≤ 30 ml/min/1.72m2 Ratio (ACR) (<2.5)
3- eGFR ≤ 40 ml/min/1.72m2
Which of the following is the following is the most
4- eGFr ≤ 50 ml/min/1.72m2 appropriate blood pressure target for this patient?
5- eGFR ≤ 60 ml/min/1.72m2 (Please select 1 option)
1- 130 / 80 mm/hg
Answer & Comments
2- 140 / 80 mm/hg
Answer: 2- eGFR ≤ 30 ml/min/1.72m2
3- 120 / 80 mm/hg
current HbA1c measuring 7.4% (< 6). His main disease and numbness progress. Of course, these
complaint is that of burning discomfort in both uncontrolled patients are at great risk for diabetic
feet which is more severe at night. He has foot ulcers and amputation.
experience only minimal relief with co-codamol
and non-steroidal anti-inflammatory based Reference:
analgesia. NICE clinical guidelines 173: Neuropathic pain
of vibration and proprioception sensation is the with autonomic nerve dysfucntion. Examination
earliest manifestation of peripheral sensory motor findings include reduced vibration and
neuropathy in diabetes. proprioception sensation, diminished ankle jerks,
muscle wasting and increased blood flow.
[ Q: 54 ] Question ID #224 Reduced light pressure sensation and vibration
sensation are the earliest clinically identifiable
manifestations of large fiber neuropathy.
A 69 year old lady with a 7 year history of type 2
diabetes is reviewed in clinic. Her main complaint
is of numbness in her feet. Current medication [ Q: 55 ] Question ID #225
includes metformin 1 gram twice daily, Insulin
glargine 32 u at night, perindopril 4 mg once daily, A 39 year old man with a 21 year history of type 1
simvastatin 20 mg once daily and aspirin 75 mg diabetes with chronic poor control has a most
once daily. On examination her blood pressure is recent HbA1c of 9.1% (< 6). His current medication
155 / 56 mmHg, her BMI is 33 kg / m2 peripheral includes Novomix 30 56 units in the morning and
pulses are present but vibration sensation is 64 units in the evening. Retinal screening reveals
reduced in both feet. bilateral background retinopathy and urinalysis
reveals an albumin:creatinine ratio of 4.9 mg /
Investigations reveal: mmol creat (< 2.5). His main complaint is that of
burning discomfort in both feet. Examination of
HbA1c 7.9% (<6) his feet reveals them to be cold but with preserved
peripheral pulses and reduced thermosensation,
Urine vibration and proprioception sensation.
3.9 mg/mmol creat
Albumin:Creatinine
(<3.5)
Ratio
Which of the following would suggest a diagnosis
55 ml/min/1.73m² of short fiber neuropathy?
eGFR
(90-110)
(Please select 1 option)
3- Heart rate variability of < 10 beats per On examination her blood pressure is 105 / 65
minute with respiration mmHg and her body weight is 63 kg. Examination
4- QT - interval of < 440 ms of her feet reveals reduced absent ankle jerks
bilaterally and reduced vibration and
5- Longest R-R to shortest R-R ratio during proprioception in both feet. Her main complaint is
valsalva maneuver > 1.2 that of increasingly frequent episodes of fecal
incontinence.
Answer & Comments
Which of the following clinical features would
Answer: 3- Heart rate variability of < 10 beats per support a diagnosis of autonomic neuropathy?
minute with respiration (Please select 1 option)
Answer & Comments Which of the following is the most likely diagnosis
for this gentleman?
Answer: 2- Heart rate increase on standing after
15 seconds <12 beats per minute (Please select 1 option)
Amyotrophic lateral sclerosis (ALS) may occur vibration sensation, thermosensation and
from the teenage years to the late 80s, but peak proprioception sensation in both feet. Dorsalis
age at onset occurs from 55-75 years with mean pedis and posterior tibial pulses are palpable
age of 62 years at diagnosis. Muscle weakness bilaterally. Her current medication includes
usually is asymmetric, and signs of upper and ‘Novomix 30’ 26 units twice daily, Metformin 500
lower motor neuron involvement may be mg twice daily, rosuvastatin 10 mg once daily,
observed. In the initial stages, reflexes should be perindopril 4 mg once daily, amlodipine 5 mg once
normal to increased, but as the disease becomes daily, bisoprolol 2.5 mg once daily and furosemide
more advanced, reflexes may be diminished or 40 mg once daily.
absent. Fasciculations are commonly seen as a
result of lower motor neuron involvement Investigations reveal
Pathologic long-tract signs of upper motor neuron
involvement, such as the extensor plantar Hba1c 7.9% (<6)
response and/or Hoffmann sign, may be present
in patients with hyporeflexia or areflexia, Total Cholesterol 3.6 mmol/l (<5)
depending upon the extent of the upper motor Plasma TG 1.7 mmol/l (0.9 - 2.2)
neuron involvement. In 75-80% of patients, eGFR 47 ml/min/1.73m² (90-110)
symptoms begin with limb involvement, while 19-
25% of patients present with bulbar symptoms. LFT Normal
For those with limb involvement at presentation, TSH 1.21 mu / / (0.32 - 5.5)
incidence of upper limb versus lower limb
involvement is approximately equal. Patients who Which of the following would be the most
have lower limb onset initially may complain of appropriate initial therapy of choice for this lady’s
tripping, stumbling, or awkwardness when symptoms?
running. Those with upper limb onset may have
(Please select 1 option)
difficulty with actions such as buttoning clothes,
picking up small objects, or turning a key. With 1- Amitriptyline
bulbar onset, patients note problems such as
2- Carbemazepine
slurred speech, hoarseness, or decreased volume
of speech. 3- Duloxetine
4- Gabapentin
The absence of any sensory symptoms coupled
with preserved reflexes and no prior history of disc 5- Pregabalin
injury or back pain makes a diagnosis of lumbar
radiculopathy unlikely, while the clinical picture
Answer & Comments
coupled with normal inflammatory markers and
CK levels also makes diagnoses of polymyalgia or Answer: 3- Duloxetine
polymyositis unlikely.
In accordance with NICE guidelines 173, for people
[ Q: 59 ] Question ID #230 with painful diabetic neuropathy, offer oral
duloxetine 60mg, serotonin-norepinephrine
A 75 year old lady with a 10 year history of type 2 reuptake inhibitor (SNRI), as first-line treatment. If
diabetes complicated by ischemic heart disease duloxetine is contraindicated or satisfactory pain
and congestive cardiac failure is reviewed in clinic. reduction is not achieved with maximum tolerated
Her main complaint is that of electric shock like dose of duloxetine, offer oral amitriptyline 10mg,
pains in both feet, particularly at night with only a tricyclic antidepressant (TCA), or oral pregabalin
minimal relief from simple analgesia. 150mg, an anti-epileptic, instead/in addition. If
satisfactory pain reduction is still not achieved
On examination her blood pressure is 106/55 with second-line treatment, refer the patient to a
mmHg, her BMI is 31 kg/m2 and examination of specialist pain service and/or a condition specific
her feet reveals bilateral pitting edema, reduced
service. While waiting for referral, consider oral 4- Arrange a routine referral for nephrology
tramadol or topical lidocaine. assessment
Tight control of blood glucose can reverse the 5- Arrange for an urgent nephrology
changes of diabetic neuropathy, but only if the assessment
neuropathy and diabetes is recent in onset.
Conversely, painful symptoms of neuropathy in Answer & Comments
uncontrolled diabetics tend to subside as the
disease and numbness progress. Of course, these Answer: 4- Arrange a routine referral for
uncontrolled patients are at great risk for diabetic nephrology assessment
foot ulcers and amputation.
This gentleman has demonstrated a significant
Reference: progression in kidney disease as defined by the
NICE clinical guidelines 173: Neuropathic pain observed decline in eGFR over a 6 month period.
A decline in eGFR > 5 ml/min/1.72m2within 1 year
[ Q: 60 ] Question ID #231 defines significant progression of kidney disease
and is an indication for further investigation and
intensification of therapy. Initiation of ACE-I
A 64 year old gentleman with a five year history of
therapy of or Angiotensin receptor blocker
type 2 diabetes is reviewed in clinic. On
therapy may result in decline of eGFR of 15 – 25%
examination, his current medication includes
as part of a normal response. A greater decline in
PIoglitazone 30 mg once daily, gliclazide 160 mg
response to the initiation of such agents should
twice daily, simvastatin 40 mg once daily, ramipril
alert the clinician to the possibility of renovascular
5 mg once daily and amlodipine 5 mg once daily.
disease.
On examination, his blood pressure is 145/75
This gentleman may also have early renal anemia.
mmHg and his BMI is 33 kg/m2. He has bilateral
The WHO definition of normal haemoglobin levels
background retinopathy on retinal screening but
for men and post-menopausal women is > 13 g /
has no evidence of peripheral sensory neuropathy.
dl. In patients with diabetic nephropathy renal
His current eGFR is 43 ml/min/1.73m2 compared
anemia develops earlier than in people with non-
with 59 ml/min/1.73m2 when measured 6 months
diabetic kidney disease.(this is likely to have
previously.
diabetic nephropathy since he has evidence of
other microvascular complications in the form of
Further investigations reveal:
retinopathy). In diabetic kidney disease a
threshold eGFR < 60 ml / min / 1.73m2is advocated
Hba1c 8.1% (<6) by the national service framework as the point at
Haemoglobin 11.9 g/ dl (11 - 16) which to begin screening for renal anemia. Using
this threshold around 85% of cases would be
Total Cholesterol 4.4 mmol/l (<4)
identified.
Urine 8.9 mg/mmol/creat
Albumin:creatinine ratio (<2.5)
[ Q: 61 ] Question ID #232
Which of the following is the most appropriate
approach to the management of this gentleman? A 44 year old Asian lady with a 5 year history of
type 2 diabetes is admitted to hospital with
(Please select 1 option)
progressive pitting odema.
1- Repeat eGFR in 3 months
On examination, her blood pressure is 156/76
2- Repeat eGFR in 6 months
mmHg, her BMI is 38 kg/m2 and fundoscopy
3- Arrange for 24 hour urine creatinine reveals changes of background diabetic
clearance measurement retinopathy in both eyes. Her current medication
produce blood pressure independent Which of the following is the most appropriate
reductions in her albuminuria advice for this lady?
(Please select 1 option)
Answer & Comments 1- Her retinopathy may deteriorate if she
Answer: 1- Her Ramipril dose should be increased breast feeds.
aiming to produce blood pressure dependent 2- She does not require retinal screening during
reductions in her albuminuria her pregnancy.
3- She will require screening three times during
The results of studies such as BENEDICT (Bergamo
her pregnancy.
Nephrologic Diabetes Complications Trial) and
RENAAL (Reduction of Endpoints in NIDDM with 4- She will require an ACE-I to protect her from
the Angiotensin II Antagonist Losartan) suggest worsening retinopathy.
that blockade of the renin angiotensin system with
ACE-Inhibitors or Angiotensin receptor blockers 5- She will require aspirin to protect her from
produces reductions in microalbuminuria and worsening retinopathy.
delays development of microalbuminuria
independent of blood pressure reduction. Dual Answer & Comments
blockade of the rennin angiotensin system with
combination ACE-inhibitor and Angiotensin Answer: 3- She will require screening three times
receptor blocker therapy may reduce during her pregnancy.
microalbuminuria, however there is no evidence
that dual blockade reduces progression to end- Diabetic retinopathy may rapidly deteriorate
stage renal disease. during pregnancy due to the rapid improvement in
glycaemic control that is often achieved; she
The DETAIL study demonstrated that ACE inhibitor therefore needs dilated fundoscopy or
and ARB therapy exerted similar effects on GFR photography every trimester. ACE inhibitors are
(Barnett A.J Am Soc Nephrol. 2006 Apr;17(4 Suppl
contraindicated during pregnancy and aspirin
2):S132-5).
does not have any data to support its use in
pregnancy. Breast feeding will have no effect on
The addition of a direct rennin inhibitor to ongoing
retinopathy.
antihypertensive therapy including agents that
block the rennin angiotensin system has been
shown to result in blood pressure independent [ Q: 64 ] Question ID #237
reduction in microalbuminuria (N Engl J Med. 2008
Jun 5;358(23):2433-46 A 22 year old law student is reviewed in clinic
following her recent diagnosis with type 1
[ Q: 63 ] Question ID #236 diabetes six months previously. She has an HbA1c
of 8.9% (<6) and smokes 10 cigarettes a day. Her
main concern relates to the effect that diabetes
A 26 year old woman with pre-existing type 1
may have on her eyesight.
diabetes for 12 years attends your medical
antenatal clinic as she has just discovered that she
is pregnant. This is an unplanned pregnancy and Which of the following is the most appropriate
her HbA1c is 8.2% (<6). At her most recent retinal approach for this young lady?
screen she was noted to have background (Please select 1 option)
retinopathy affecting the right eye. She is
1- Achieving an HbA1c of 7% would reduce her
concerned as to the possible impact of her
risk of retinopathy developing by about 75%
pregnancy on her eyes.
2- Any future pregnancy will have no effect on
her eyes
3- She is unlikely to develop retinopathy if she 1- Delay giving her bolus until her blood
maintains an HbA1c <6.5% glucoses start to rise post-prandially
4- Stopping smoking will reduce her risk of 2- Do not give a mealtime bolus but measure
developing retinopathy the glucose 2 hours post-prandially and give
an appropriate correction bolus
5- She is unlikely to develop any problems with
her eyes for the next 10 years 3- Do not give a mealtime bolus but increase
the basal rate around the mealtimes to
Answer & Comments cover the rise in glucose.
Answer: 1- Achieving an HbA1c of 7% would 4- Give the bolus of insulin over a protracted
reduce her risk of retinopathy developing by time such as 60 to 120 minutes
about 75% 5- Put a longer acting insulin into the pump
instead of the usual short acting insulin
The Diabetes Control and Complications Trial
showed a 75% reduction in retinopathy Answer & Comments
developing and a 60% reduction in progression
from mild retinopathy associated with a reduction Answer: 4- Give the bolus of insulin over a
of HbA1c from 9% to 7%. Unfortunately even with protracted time such as 60 to 120 minutes
good control she will almost inevitably develop
retinopathy and the risk is greater in type 1
This can be a difficult issue to address to a
diabetes compared to type 2 diabetes as
satisfactory degree. Delaying giving a bolus until
demonstrated by the Wisconsin Epidemiologic
glucoses rise is likely to allow symptoms of
Diabetic Retinopathy Study. Interestingly that
hyperglycaemia to develop. If the basal rate is
study did not find an association between smoking increased this is restrictive in terms of timing of
and retinopathy. Features of retinopathy usually meals and both type and amount of food and
do not appear in patients with type 1 diabetes for allowing freedom of choice of food is a major
up to 5 years following diagnosis, while pregnancy
benefit of using continuous subcutaneous insulin
is associated with potential progression of infusion. Giving a bolus over a protracted period of
retinopathy. time (also known as a square wave bolus) can help
to overcome this problem.
[ Q: 65 ] Question ID #246
[ Q: 66 ] Question ID #299
A 37 year old woman has had type 1 diabetes for
34 years. For the last 2 years she has had a
A 60 year old female is seen as part of the retinal
problem of vomiting 2 hours after her meals
screening service. Her vision is fine at 6/9
intermittently. A barium meal study has suggested
bilaterally and she is noted to have a couple of
gastroparesis. She has tried various treatments for
microaenurysms only in both eyes.
this but all with limited success. She has difficulty
controlling her blood glucose levels post-
When should this patient next have retinal
prandially as these are either high if she doesn’t
screening?
give enough short acting insulin or low if she
vomits 2 hours after a meal. It is suggested that (Please select 1 option)
she tries a continuous subcutaneous insulin 1- one month
infusion to try to help with this problem.
2- three month
You are asked to advice on her mealtime boluses; 3- six months
you would suggest:
4- one year
(Please select 1 option)
1- Detached macula
Which of the following is the likely explanation for
his presentation and biochemistry? 2- Intraretinal microvascular abnormalities
(IRMAs)
Sodium 142 mmol/L (135-145) 3- Venous beading
Potassium 5.5 mmol/L (3.5-5)
4- Microaneurysms
Urea 8.5 mmol/L (3-8)
5- New vessels
Creatinine 133 mmol/L (50-100)
Plasma glucose 10.1 mmol/L (3.5-6.0)
Answer & Comments
HbA1c 7.8% (<6)
Answer: 4- Microaneurysms
(Please select 1 option)
1- Increase atorvastatin to 80 mg
Answer & Comments
2- Start Ezetimibe
Answer: 5- Venous beading
3- Start Niacin
The National Screening Committee has produced 4- Start omega fatty acid
guidance grading the lesions present on retinal
screening. These are 5- No changes required
What should be the weight of monofilament used Which of the following eye complications is likely
to check for peripheral neuropathy in Diabetics? to be associated with pioglitazone usage?
(Please select 1 option) (Please select 1 option)
1- 1g 1- Cataracts
2- 5g 2- Glaucoma
3- 10g 3- Macular oedema
4- 20g 4- Pre-proliferative retinopathy
5- 100g 5- Retinal degeneration
bilateral sensory loss in distal extremities. There is otherwise essentially normal. His biochemistry
no superficial skin ulcer and range of movements shows:
is restricted as the foot looks swollen and
deformed. Albumin 42 g/L (30-50 g/L)
ALP 202 U/L (50-125 U/L)
What is the next best step in her management?
ALT 116 mU/L (5-58)
(Please select 1 option)
Bilirubin 35 micromol/L (0-18)
1- Complete contact cast
Proteins 72 g/L (60-80)
2- IV antibiotics
HbA1c 8.1% (<6)
3- NSAIDs
4- Oral antibiotics In view of the deranged LFTs an ultrasound
abdomen is arranged which reveals fatty
5- Urgent vascular assessment infiltration of the liver.
1- Exenetide
Charcot`s arthropathy is a rare, progressive
degeneration of weight bearing joints. It is seen 2- Insulin
associated with Diabetes Mellitus, alcoholic
neuropathy, cerebral palsy and spinal cord lesions. 3- Metformin
The underlying mechanism leading to Charcot`s 4- Pioglitazone
arthropathy is unclear although recurrent micro-
trauma to a joint (predisposed by absent 5- Sulfonylureas
peripheral sensation and proprio-reception) or
dysregulated autonomic nervous system (leading Answer & Comments
to increased perfusion) seem to play a part in its
pathogenesis. Answer: 3- Metformin
The commonest joints afflicted with Charcot`s This gentleman has non-alcoholic steatohepatitis
arthropathy in diabetes mellitus include tarso- (NASH) as evidenced by deranged liver function
metatarsal joint and metatatrso-phalangeal joint. tests and fatty infiltration on Ultrasound scan.
Patients present with swelling of the joint and NASH is commonly seen in Type 2 diabetic and
clinical examination reveals mildly raised associated with increased body mass index. It
temperature of the affected joint during the acute usually resembles alcoholic liver disease but the
phase. A MRI scan can aid in diagnosis and history of alcohol intake in such people is either
treatment includes complete contact cast to avoid absent or only modest. The diagnosis of this
development of deformities like rocker bottom condition is based on exclusion of common liver
foot and tarso-metatarsal dislocation. disorders like viral hepatitis, alcoholic liver
disease, drug induced and autoimmune liver
[ Q: 74 ] Question ID #681 disease (e.g. primary biliary cirrhosis). The
confirmatory test for diagnosis is liver biopsy. No
specific therapy for NASH exists and the patients
A newly diagnosed Type 2 Diabetic male is
should be encouraged to bring life style
referred to the clinic. His BMI is 28 kg/m2 and he
modification changes including regular exercise
drinks 4 units of alcohol every week. His blood
and weight loss.
pressure is 134/82 mmHg and examination is
pressure is 98 / 66 mm/hg and her pulse is 105 causes direct stimulation of carnitine acyl coA
bpm. transferase 1 and high glucagon levels inhibit the
formation of malonyl CoA, further stimulating
Investigations- transferase activity. Consequently there is a switch
NEFA resterification to oxidation.
Plasma Glucose 21.2 mmol/l (<7.8) Betahydroxybutyrate is produced in excess and is
subsequently metabolised to acetoacetate which
pH 6.9 (7.35 - 7.45) is detected in urine ketone analysis by dip stick
pCO2 3.2 kPa (4.7 - 6) testing. Glucagon levels also increase levels of
pO2 12.1 kPa (9.5 - 13) carnitine further enhancing fatty acid oxidation.
Cortisol and catecholamines directly stimulate
Serum bicarbonate 5.2 mmol/l (22 - 27) lipolysis. Glucagon is of particular importance as
haemoglobin 15.9 g / dl (12 - 16) ketone body levels rise early, with a continuous
wCC 21.1 X 109 (4 - 11) increase over 10 – 12 hours. Acidaemia, combined
with hyperglycaemia and hyperglucagonaemia
Which of the following is a feature of the promote intracellular loss of potassium. The final
serum potassium concentration largely depends
pathophysiology of diabetic ketoacidosis?
on the rate of urinary potassium loss. Platelet
(Please select 1 option) secretory activity is often increased in DKA, but
1- Acetoaceate is metabolized to beta aggregation decreased. Neutrophil count is also
hydroxybutyrate commonly raised and correlates with ketone body
levels, so does not necessarily imply underlying
2- Acidaemia is the primary cause of infection.
intracellular potassium loss
3- Hepatic glucose production reaches a [ Q: 77 ] Question ID #27
plateau within 4 hours of insulin withdrawal
A 21 year old lady with 10 year history of type 1
4- Hepatic glucose production rises gradually
diabetes has a most recent Hba1c of 7.8% ( < 6) is
over the first 6 - 8 hours following insulin
admitted to A+E following an overdose of her
withdrawal basal insulin. On examination she is conscious but
5- Insulin deficiency combined with cortisol and pale and sweaty, her blood pressure is 96 / 66
catecholamine excess is the main cause of mmHg, her pulse is 135 beats per minute and her
ketone body formation plasma glucose is 1.8 mmol/l.
2- Ketonuria correlates closely with degree of 2- Advise that the pump be re-sited at least
acidosis in diabetic ketoacidois 10cm from the current insertion site
3- Ketonuria resolves with correction of 3- Remove her pump and repeat the process of
acidosis in diabetic ketoacidosis drawing up the insulin and priming the
pump and using a new giving set and
4- Urine ketone stix measure acetoacetate in
cannula in case of an error
the urine
4- Remove the pump and revert to insulin via a
5- Urine ketone stix measure
pen until a new pump is able to be delivered
betahydroxybutyrate levels in the urine
urgently
Answer & Comments 5- Reduce all basal rates to 50% and halve the
insulin to carbohydrate ratio
Answer: 4- Urine ketone stix measure
acetoacetate in the urine
Answer & Comments
Urine ketone testing measures acetoacetate in Answer: 1- Advise her to come into hospital
urine which is produced by metabolism of immediately for assessment for urgent
betahydroxybutyrate. Ketoneuria persists despite delivery of her baby.
correction of acidaemia in patients with
ketoacidosis and does not correlate with the Placental insufficiency can present in diabetes as
degree of acidosis. Ketonuria is not a marker of severe recurrent unwarranted hypoglycaemia.
absolute insulin deficiency and may occur as a This should always be regarded as an emergency
normal response to exercise or fasting. in the third trimester as this may herald placental
insufficiency and prompt action and delivery may
[ Q: 81 ] Question ID #244 prevent intrauterine death. Only after placental
insufficiency has been excluded should other
A 33 year old woman is currently 34 weeks causes of hypoglycaemia be explored.
pregnant. She was commenced on continuous
subcutaneous insulin infusion at the end of the [ Q: 82 ] Question ID #3451
first trimester after developing severe recurrent
hypoglycaemic episodes. Her HbA1c has been You are called via the arrest bleep to a 43 year old
5.8% and she has had an occasional woman who is on the surgical ward. She had a
hypoglycaemic episode but generally these have laparoscopic cholecystectomy performed two
been mild and she has been able to manage these days ago and has since had nausea and vomiting
herself. She telephones your department as and has been receiving IV dextrose. It transpires
during the last 24 hours she has developed that she had a fit for which the nursing staff
recurrent severe hypoglycaemic episodes such requested the arrest team. She is now drowsy. Her
that she is continually taking lucozade to prevent past medical history is unremarkable.
this despite dramatically reducing her insulin
rates.
On examination, she is drowsy with a GCS of 13.
Her pulse is 88 bpm and blood pressure 132/88
How would you advise her? mmHg with oxygen sats of 100% on air. There is no
(Please select 1 option) focal neurology and her plantar responses are
1- Advise her to come into hospital equivocal.
immediately for assessment for urgent
Of note, her blood results reveal:
delivery of her baby.
Potassium 3.5 mmol/l (3.5-5) What is the target sodium to which you would
treat this patient over the next 1hr?
Urea 3.8 mmol/l (3-8)
(Please select 1 option)
Creatinine 70 micromol/l (50-100)
Glucose 4.1 mmol/l (3.5-6) 1- 116 mmol/l
2- 120 mmol/l
Which of the following is the most appropriate
initial treatment for this patient? 3- 125 mmol/l
Checking through his records, the previous days the last few days. She is lethargic, has had
his U+Es were normal. He had been allowed to eat occasional episodes of abdominal pain, and is
and drink normally and had stopped IV fluids 2 clinically dehydrated. She has a recent history of
days ago. He was on dexamethasone the dose of metastatic breast cancer with bony involvement,
which had been reduced to 4mg daily from 8 mg for which she is receiving palliative treatment. Her
daily over the last day but he was on no other results reveal:
treatment.
Sodium 144 mmol/l (134-144)
On examination, he had Glasgow Coma Scale of
Potassium 3.1 mmol/l (3.5-5.5)
15, though seemed confused with him being
disorientated. Oxygen saturations were 97% on air Calcium 4.1 mmol/l (2.2-2.6)
and his blood pressure was 110/80 with a pulse of
88 bpm. Which of the following would you expect to find on
the ECG?
Which of the following is the most likely diagnosis? (Please select 1 option)
(Please select 1 option)
1- Prolonged PR Interval
1- Acute hypoadrenalism
2- QT shortening
2- Cerebral Salt Wasting
3- ST elevation
3- Pituitary apoplexy
4- Tall tented T waves
4- Syndrome of inappropriate ADH secretion
5- Widened QRS complexes
5- Type IV renal Tubular acidosis
Answer & Comments
Answer & Comments
Answer: 2- QT shortening
Answer: 2- Cerebral Salt Wasting
The predominant abnormality here will relate to
The most likely diagnosis in this patient with head the markedly elevated calcium, not the mild
injury who then develops acute symptomatic hypercalcaemia. The QT interval is typically
hyponatraemia and grossly elevated urine sodium shortened in hypercalcaemia. Other features in
is cerebral salt wasting. The grossly elevated urine severe hypercalcaemia include J waves and bizarre
sodium (>100) is highly suggestive as the sodium is QRS appearance.
usually less with SIADH. Similarly with SIADH,
there is a dilutional element so urea and Some examples of ECG abnormalities in
potassium typically reflect this dilution being hypercalcaemia
themselves low normal in contrast to CSW where
there is hypovolaemia and hence higher
[ Q: 86 ] Question ID #3594
concentrations. Hypoadrenalism is an incorrect
answer as the patient is already on a high dose of
steroids – dexamethasone 4mg/d. A 53 year old man is admitted with a 3 day history
of left leg pain and fever. He has an 8 year history
Reference: of type 2 diabetes for which he takes metformin
Cerebral Salt Wasting Medscape and gliclazide together with ramipril and
simvastatin.
Answer: 1- IV flucloxacillin
Answer & Comments
This patient has a clinical class II to class III cellulitis Answer: 3- Omeprazole
with systemic upset associated with a
comorbidity. The vast majority of cases of cellulitis This patient has a hypocalcaemia associated with
are due to beta haemolytic strep or S Aureus. a severe hypomagnesaemia. This is likely due to
Therefore, Flucloxacillin is the treatment of choice the omeprazole, which results in increased GI
in this setting and recommended in guidelines as magnesium losses. The hypomagnesaemia impairs
IV treatment in Class II-III patients. In subjects the calcium sensing on the parathyroid cells;
allergic to beta lactams, hence, this results in an induced
clarithromycin/clindamycin can be used. hypoparathyroidism with hypocalcaemia. It is a
good diagnosis to make but is often not
Guidance on management of cellulitis in adults appreciated.
This is not due to vitamin D deficiency as with 3- Initial priming bolus dose of IV insulin
osteomalacia the 25OHD would be lower, ALP followed by sliding scale insulin
raised and the patient would likely have a proximal
myopathy. 4- Sliding scale IV insulin infusion
5- developed diabetic ketoacidosis as
PPI induced hypoparathyroidism evidenced by the low bicarbonate and pH.
Guidelines recommend the use of an IV fixed
[ Q: 88 ] Question ID #3767 rate insulin infusion regime consisting of
0.1iu/kg/hr and to continue the patient’s
A 30 year old female is admitted with a 2 day usual long acting insulin.
history of abdominal pains and general
deterioration. She has a background history of Answer & Comments
type 1 diabetes for which she takes basal bolus
insulin regime consisting of a long acting insulin Answer: 1- Fixed rate IV insulin infusion of
(20 iu) in the evening and thrice daily short acting 0.1iu/kg/hr
insulin (10-12 iu) with each meal. She has
continued to take her insulin but has become iller,
Reference:
being nauseous with vomiting associated with a
Joint British Scoieties Diabetes Care Group DKA
diffuse abdominal pain.
Guidelines
On examination, she appears dehydrated, has a
pulse of 98 bpm reg and a blood pressure of [ Q: 89 ] Question ID #3768
108/76 mmHg. Her Glasgow coma scale is 15 and
she has oxygen saturations of 98% on air. A 30 year old female is admitted with a 2 day
Abdominal examination reveals nothing overt history of abdominal pains and general
with just diffuse tenderness. deterioration. She has a background history of
type 1 diabetes for which she takes basal bolus
Her results reveal the following: insulin regime consisting of a long acting insulin
(20 iu) in the evening and thrice daily short acting
Glucose 23.7 mmol/l (3.5-5.5) insulin (10-12 iu) with each meal. She has
continued to take her insulin but has become iller,
Sodium 144 mmol/l (134-144)
being nauseous with vomiting associated with a
Potassium 3.0 mmol/l (3.5-5.5) diffuse abdominal pain.
Urea 8.3 mmol/l (3-8)
Creatinine Creatinine On examination, she appears dehydrated, has a
pulse of 98 bpm reg and a blood pressure of
pH 7.28 108/76 mmHg. Her Glasgow coma scale is 15 and
pO2 14.4kPa she has oxygen saturations of 98% on air.
PCO2 3.5 kPa Abdominal examination reveals nothing overt
with just diffuse tenderness.
Bicarbonate 15 mmol/l
Her results reveal the following:
Which of the following is the most appropriate
initial management for this patient’s Glucose 23.7 mmol/l (3.5-5.5)
hyperglycaemia?
Sodium 144 mmol/l (134-144)
(Please select 1 option)
Potassium 3.0 mmol/l (3.5-5.5)
1- Fixed rate IV insulin infusion of 0.1iu/kg/hr
Urea 8.3 mmol/l (3-8)
2- Give 15 units of SC short acting insulin Creatinine 105 micromol/l (50-100)
together with 10 units of long acting insulin
pH 7.28 90/66 mmHg and pulse 100 bpm reg. Her Glasgow
Coma Scale is 15 and there are no significant
pO2 14.4 kPa
findings on general examination.
PCO2 3.5 kPa
Bicarbonate 15 mmol/l Investigations reveal the following:
Which of the following is the most appropriate Glucose 9.9 mmol/l (3.5-5.5)
initial management for this patient? Sodium 129 mmol/l ((134-144)
(Please select 1 option) Potassium 5.5 mmol/l (3.5-5.5)
1- 100mls 8.4% sodium bicarbonate over 30 Urea 8.2 mmol/l (3-8)
mins, together with 1l Normal saline with 20 Ketones on meter 0.7 mmol/l (<1.5)
mmol potassium chloride over 1hr
2- 1l 5% dextrose given over 1 hr Which of the following would be the most
appropriate treatment for this patient?
3- 1l Hartmann’s solution given over 2 hrs
(Please select 1 option)
4- 1l Normal saline given over 2 hr
1- 100mcg Oral fludrocortisone
5- 1l Normal saline with 20mmol potassium
2- 1l N Saline over 1hr
chloride given over 1 hr
3- 100 mg IV Hydrocortisone as a blous
Answer & Comments 4- Fixed rate insulin infusion 0.1 iu/kg/hr
Answer: 5- 1l Normal saline with 20mmol 5- IV Insulin sliding scale
potassium chloride given over 1 hr
Answer & Comments
In this patient who has diabetic ketoacidosis, the
most appropriate treatment for immediate IV Answer: 3- 100 mg IV Hydrocortisone as a blous
resuscitation is 1l of Normal saline given over the
first hour and, in view of the low potassium, This patient appears to have a hypoadrenal crisis
potassium chloride should also be given as this will probably precipitated by the inability to retain the
fall quite abruptly when the insulin regime is oral hydrocortisone. In this case the glucose less
started. than 10 and the absence of ketones on the ketone
meter makes a diagnosis of DKA unlikely. The most
Reference: appropriate approach is to treat this patient with
Joint British Scoieties Diabetes Care Group DKA 100 mg IV hydrocortisone.
Guidelines
Reference:
[ Q: 90 ] Question ID #3769 Guidelines from the endocrine society
On Examination he appears ill and weak. His be stopped and in conjunction with the acute
temperature is 37 C and oxygen saturation is 97% kidney injury may be the precipitant.
on air. His blood pressure is 154/100 mmHg, heart
rate 92 bpm reg and respiratory rate is 22/min. Reference:
Renal Society Guidelines on treating
Results: hyperkalaemia
Sodium 141 mmol/l (134-144)
Potassium 7.6 mmol/l (3.5-5.5) [ Q: 92 ] Question ID #3779
Urea 19 mmol/l (3-8)
This is the ECG obtained on a 77 year old man with
Creatinine 350 micromol/l (50-100)
type 2 diabetes who was admitted with lethargy
pH 7.36 and tiredness.
Bicarbonate 20 mmol/l
1- IM Calcitonin
Answer & Comments
2- IM Denosumab
Answer: 5- 7
3- IV Hydrocortisone
4- IV N Saline – 1l over 2hrs initially The ABCD2 score is a risk assessment tool
designed to stratify TIAs according to future risk of
5- IV Pamidronate
stroke.
[ Q: 102 ] Question ID #94 A 32 year old female who has been treated with
cabergoline for a microprolactinoma attends clinic
and informs you that she is 8 weeks pregnant. She
A 24 year old female with a diagnosis of polycystic
has been continuing to take the cabergoline 0.25
ovarian syndrome is keen to start a family. She was
mg twice weekly and the pregnancy was
diagnosed with PCOs at the endocrine clinic
unexpected but she would like to continue with
following presentation with oligomenorrhoea and
the pregnancy.
mild hirsutes. She is overweight with a BMI of 27
kg/m2 and she has a menstrual cycle of
Which of the following would you advise?
approximately 6-8 weeks. She has been trying for
the last year to conceive but has been (Please select 1 option)
unsuccessful. Her partner has had semen analysis 1- Advise termination due to the teratogenic
performed by the GP which is normal.
risk with cabergoline
Investigations reveal:
2- Continue cabergoline at current dose
Estradiol 450 pmol/l (130 - 800) 3- Increase dose of cabergoline
LH 8.3 mu/l (2 - 10)
4- Stop cabergoline
FSH 2.2 mu/l (2 - 10)
5- Switch cabergoline to bromocriptine
Testosterone 2.2 pmol/l (<3)
Prolactin 380 mu/l (<450)
Answer & Comments and testosterone gel for the last 6 years and had
grown to an appropriate adult height and had
Answer: 4- Stop cabergoline normal secondary sexual characteristics.
Investigations reveal:
Pregnancy on patients taking dopamine agonist
therapy for prolactin secreting tumours is not IGF-1 42.3 nmol/l (28 - 50)
uncommon and there is no evidence that taking
Testosterone 15.5 nmol/l (9 - 30)
these agents is harmful to the fetus. However,
there is no need to take these agents during LH 1.2 mu/l (2 - 10)
pregnancy as the risks of tumour expansion are FSH 2.2 mu/l (2 - 10)
small (<1%). Thus, the patient should be advised to
Free T4 14.5 pmol/l (9 - 22)
stop cabergoline only.
TSH 2.2 mu/l (0.5 - 5)
[ Q: 104 ] Question ID #96 Prolactin 360 mu/l (50 - 450)
A 26 year old female with a microprolactinoma He is keen to continue GH therapy but is aware
falls pregnant after taking cabergoline. She is that if this is no longer necessary he is prepared to
advised to stop the cabergoline. stop it.
How would you continue to serially monitor the Which of the following is the most appropriate
patient during her pregnancy? action for this patient concerning his GH therapy?
(Please select 1 option)
(Please select 1 option)
1- Haemochromatosis
Approximately 20% of patients fail PDE-5
2- Kallmann syndrome
inhibitors such as sildenafil/Tadalafil. It is
important to ensure that testosterone 3- Obesity
concentrations are normal if one is to secure the
4- Pituitary microadenoma
adequacy of PDE-5 inhibitor therapy. In this case,
the testosterone concentration is adequate. 5- Testicular fibrosis
Under these circumstances, the most appropriate
treatment would be intracavernosal injections
Answer & Comments
with alprostidil – caverject. The patient should be
shown how to draw up, dose and inject the Answer: 3- Obesity
alprostidil and to seek medical advice should
priapism develop.
This patient's results reveal a low testosterone
with 'inappropriately' normal LH and
[ Q: 107 ] Question ID #107 FSH indicating a hypogonadotrophic
hypogonadism. His other pituitary function is
A 48 year old male presents with a history of normal and his CT of the pituitary is also reported
reduced libido and erectile dysfunction. His GP as normal. However, he is obese and has features
encloses the following results: of the metabolic syndrome with an elevated
fasting plasma glucose. With these results, the
most probable cause of his hypogonadism is unprotected sex for at least three years without
obesity. conception. Investigations reveal:
This is a well described, emerging phenomena and Estradiol 420 pmol/l (130 - 800)
the exact cause is ill-understood although is
LH 8.8 iu/l (3 - 10)
believed to directly relate to visceral fat mass with
increasing aromatisation of testosterone to FSH 2.2 iu/l (3 - 10)
oestradiol and then feedback on the Pituitary. Prolactin 580 mu/l (50 - 500)
TFTs Normal
[ Q: 108 ] Question ID #111
Which of the following would be the most
A 22 year old female presents with a long history appropriate agent for this patient?
of oligomenorrhoea, weight gain and hirsutism. (Please select 1 option)
A 55 year old male presents with problems with A 15 year old female with long standing obesity
erectile function. He has a poor libido and denies was referred with abdominal swelling and mild
any early morning erections. He has fathered two ankle oedema. On examination the blood pressure
children and his current problems date over the was 142/88 mmHg. Investigations reveal:
last two years. On examination, he appears a
phenotypically normal male, has a BMI of 38 kg/m2 Haemoglobin 10.5 g/dL (11.5-16.5)
and has normal male distribution of body hair.
Serum biochemistry Normal
Testicular examination reveals testes of 12-15 mls
bilaterally. Serum albumin 34 g/L (37-49)
Urine dipstick Proteinuria +
Investigations reveal:
Which of the following is the most appropriate
E+Us Normal investigation for this patient?
Fasting Plasma Glucose 6.1 mmol/l (3.5-5.5) (Please select 1 option)
Testosterone 6.6 nmol/l (9-30) 1- 24 hour urinary protein estimation
LH 2.8 iu/l (3-10) 2- Abdominal ultrasound
FSH 3.4 iu/l (3-10)
3- Protein electrophoresis
Prolactin 410 mu/l (<450)
4- Urinary albumin: creatinine ratio
Free T4 12.8 pmol/l (9-23)
TSH 3.2 mu/l (0.5-5) 5- Urinary B-human chorionic gonadotrophin
test (B-HCG)
Which of the following is likely to explain this
man’s results? Answer & Comments
(Please select 1 option)
Answer: 5- Urinary B-human chorionic
1- Craniopharyngioma gonadotrophin test (B-HCG)
2- Haemochromatosis
This young girl has been 'gained weight'with
3- Kallmann syndrome abdominal swelling and ankle oedema. She is
hypertensive, has proteinuria and is anaemic. This rise by no more than 0.5 ng/ml following
should raise a suggestion of a concealed testosterone treatment
pregnancy with pre-eclampsia. The most relevant
investigation would be a pregnancy test - urinary There is no evidence that testosterone causes
B-HCG. prostate cancer but, hypogonadism is associated
with prostate shrinkage. Hence, there is evidence
[ Q: 113 ] Question ID #423 suggesting that testosterone replacement may
actually unveil a prostate malignancy and one
should be vigilant with regard to a potential rise in
A 55 year old male is diagnosed with weight- PSA concentrations. 15% of men qith prostate
related hypogonadotrophic hypogonadism and is cancer have a PSA below 4.
treated with topical titratable testosterone 60mg
daily. Previous examination revealed a normal PR
examination. Original investigations revealed PSA [ Q: 114 ] Question ID #424
concentration 2.2 ng/ml, testosterone 7 nmol/l
and haematocrit 38%. A 72 year old male is referred by his GP who was
concerned about an elevated prolactin
On attendance at the clinic three months he feels concentration. He had been well up until 3 years
better in himself, with more energy and improved ago when he was diagnosed with prostate cancer
libido. Results reveal: and was treated with GnRH analogues. Since he
has been tired but his prostate cancer is well
Testosterone 13.8 nmol/l (8-30) controlled, being on 6 monthly appointments with
urology. His GP had checked a few results and
PSA 3.8 ng/ml (2.6-4)
found that his Prolactin was 980 mu/l (50-450),
Haematocrit 43% (40-50) precipitating referral. He currently receives
amlodipine, ramipril, simvastatin and
Which of the following is the most appropriate domperidone
approach to the management of this man's
testosterone He appears hypogonadal in appearance and his
(Please select 1 option)
blood pressure is 134/74 mmHg with a BMI of
32kg/m2.
1- Continue current dose of testosterone
2- Increase dose of testosterone Investigations reveal:
5- Testosterone replacement
This 16 year old has complete androgen
insensitivity syndrome as suggested by presence
Answer & Comments of primary amenorrhoea, scanty hair growth and
history of previous inguinal hernial repair pointing
Answer: 3- Refer to fertility clinic for evaluation
towards presence of ectopic testicular tissue
previously. It is an X linked recessive disorder due
This gentleman has primary gonadal failure based to mutation of androgen receptor gene located on
on his low testosterone levels and increased FSH/ Xq11-12, although a negative family history may
LH levels. His tall stature, bilateral gynaecomastia be present in more than one third cases.
and small testicular volume point towards
testosterone deficiency developing before he The diagnosis is usually suspected based on clinical
attained puberty, leading to eunuchoid physical profile including primary amenorrhoea, normal
appearance. It is likely that he suffers from a breast development, lack of axillary and pubic hair
genetic disorder like Klinefelter`s syndrome and and testis like masses in inguinal canal or labia. The
will need further karyotype analysis to establish testosterone levels are clearly higher than normal
the diagnosis. female range and oestrogen levels are low. LH
levels are elevated with normal FSH levels. The
These patients require lifelong replacement with management includes affirmation and
testosterone and are infertile. In contrast to reinforcement of female gender identity and
secondary hypogonadism which can be treated removal of testicular tissue (orchidectomy) after
with pulsatile GnRH treatment, the management attaining puberty as to avoid development of any
of infertility in primary hypogonadism usually malignancy in the ectopic gonadal tissue.
requires modalities like In vitro fertilisation (IVF)
or intra-cytoplasmic sperm injection (ICSI) and
[ Q: 118 ] Question ID #610
better achieved under specialist fertility clinic.
2- 11 β HSD 2- DHEAS
3- 11 β Hydroxylase 3- Prolactin
4- 17 α hydroxylase 4- Testosterone
Aldosterone and Deoxycorticosterone (DOC) are It is imperative that pregnancy is ruled out in a
the two most important mineralocorticoid female (in reproductive age group), presenting
hormones, with aldosterone binds weakly with with secondary amenorrhoea to avoid
corticosteroid binding globulin (CBG) and mostly unnecessary investigations and wastage of
circulates bound to albumin. In contrast DOC is resources. Further investigations (for example -
highly bound to CBG and usually undetectable in FSH, LH, Prolactin, TFT) are only warranted if the
urine. Both of these have equal affinity for initial urinary pregnancy test is negative
mineralocorticoid receptor but the former is more
important as a mineralocorticoid hormone due to
The following is a list of main causes for erectile LH 3.2 U/L (3-10)
dysfunction: Testosterone 8.1 nmol/L (9-28.5)
Prolactin 220 mU/L (45-400)
Medications such as diuretics, β blockers,
MAO inhibitors, opioids, glucocorticoids, LFT normal
tranquilizers, digoxin, antiandrogens, MRI Brain normal
gemfibrozil
What is the likely cause for his hypogonadism? [ Q: 125 ] Question ID #690
(Please select 1 option)
1- Anabolic steroid abuse A 22 year old lady, planning for a family in near
future, has been diagnosed to have a
2- Kallmann syndrome microprolactinoma based on raised prolactin
levels and MRI scan results.
3- Kleinfelter syndrome
4- Idiopathic hypogonadotrophic Which is the best management option in her case?
hypogonadism (Please select 1 option)
5- Weight related hypogonadotrophic 1- Bromocriptine
hypogonadism
2- Cabergoline
Which of the following is the likely diagnosis? Strontium appears to have a dual mechanism of
(Please select 1 option) action reducing bone reabsorption (like
1- Familial hypercalciuric hypercalcaemia bisphosphonates) and increasing bone formation
(like PTH).
2- Primary hyperparathyroidism
3- Pseudohypoparathyroidism
4- Secondary hyperparathyroidism
A 56 year old female is referred as she has been The patient originally attended the GP with vague
concerned about osteoporosis. She went through aches and pains which have been present for two
the menopause uneventfully at the age of 50 and months or so and she also admits to feeling rather
has no particular problems of note. She has a depressed. She is only taking Fluoxetine 20 mg
strong family history of osteoporosis. A DEXA scan daily at present. There is nil of note on
that she had obtained privately reveals a T score examination. Further investigations reveal:
of -2.5 at the hip and -2.0 at the lumbar spine. Her
investigations reveal: PTH 4.4 pmol/l (3 - 5)
Which of the following is the likely diagnosis? Which of the following is the most appropriate
(Please select 1 option) management of his DEXA result?
(Please select 1 option)
1- Drug-induced
1- Growth hormone therapy
2- Familial hypocalciuric hypercalcaemia
2- Increase dose of testogel
3- Milk-Alkali syndrome
3- No specific treatment required
4- Primary hyperparathyroidism
4- Reduce dose of thyroxine
5- Tertiary hyperparathyroidism
5- Weekly alendronate
Answer & Comments
Answer & Comments
Answer: 4- Primary hyperparathyroidism
Answer: 5- Weekly alendronate
This patient has mild hypercalcaemia, an
inappropriately normal PTH concentration and has The most appropriate approach to treating this
a normal/highish urine calcium excretion. The man’s osteoporosis, as demonstrated with a T
latter would argue against a diagnosis of FHH score of less than -2.5, is a bisphosphonate such as
where urine calcium excretion is low (less than 5 weekly alendronate. The causes of low BMD in
mmol/d) and so the likely diagnosis is primary association with hypopituitarism are multi-
hyperparathyroidism. factorial and GH deficiency may contribute.
Although this man is highly likely to have GH
[ Q: 132 ] Question ID #136 deficiency as demonstrated by his low IGF-1, he
does not appear to have specific symptoms of GH
deficiency that might merit treatment and NICE do
52 year old male is referred as his GP had not advocate the use of GH as a treatment of
organised a DEXA scan for him as he had been reduced BMD in adults hypopituitary patients.
concerned about his bone thickness. This concern Unlike bisphosphonates, where there is evidence
had arisen as he had read about reduced bone of reduced fracture risk, GH therapy has no data of
mineral density in relation to hypopituitarism. Ten reduced fracture risk.
years ago he had surgical removal of a non-
functional pituitary macroadenoma and had
developed hypopituitarism for which he receives [ Q: 133 ] Question ID #138
hydrocortisone 10mg bd, thyroxine 150
micrograms daily and testogel 50mg daily. His A 55 year old female presents with a one week
DEXA scan reveals T scores of -2.6 in some areas of history of sickness, lethargy and fatigue. She has a
the lumbar spine and -2.4 in the hip. two year history of metastatic breast carcinoma.
On examination, she appears dehydrated and
He is generally fit and quite well but, after reading unwell.
an article on bone strength in hypopituitarism is
concerned about osteoporosis. Her investigations reveal:
This young girl has a long history of muscle aches of a 'young' control. The Z score is the number of
and pains and has a low calcium concentration but standard deviations between the BMD of the
elevated phosphate. The latter is suggestive of a subject and that expected for an aged and sex
hypoparathyroidism rather than vitamin D matched control.
deficiency (where a phosphate would be expected
to be low). However, the patient has an elevated T scores greater than -1 are normal. T scores
PTH concentration and so this would suggest an between -1 to -2.5 reflect osteopenia and T scores
unresponsiveness to PTH and hence a diagnosis of less than -2.5 reflect osteoporosis. This DEXA
pseudo-hypoparathyroidism. The fact that she has report of the hip reveals T scores between +0.38
no skeletal abnormalities (short 4th Metacarpal and -2.0 which indicate osteopaenia at Ward's
for instance) would suggest a type 1b or type 2 triangle and the intertrochanteric region.
pseudohypoparathyroidism.
[ Q: 137 ] Question ID #457
[ Q: 136 ] Question ID #337
A 67 year old lady with newly diagnosed
The following results were obtained on a DEXA osteoporosis re-presents to clinic with intolerance
scan of the hip of a 55 year old female who was to Alendronate 70 mg weekly after 4 weeks of
concerned regarding her risk of osteoporosis since persistence. There is history of previous fracture
there was a strong family history of fragility at the left wrist when she fell off the fourth step of
fractures. a ladder 4 years ago. She is also taking calcium and
vitamin D supplementation. She underwent
BMD T score Z score menopause at age 40 and did not take hormone
replacement therapy. There is a family history of
Neck 0.74 -1.5 -0.29
osteoporosis and hip fracture in her late mother.
Trochanteric 0.73 +0.38 +1.1 Other secondary causes of osteoporosis have
Intertrochanteric 0.91 -1.29 -0.68 been excluded.
Total 0.83 -0.95 -0.11
Her DEXA scan T scores are:
Ward's triangle 0.50 -2.0 -0.21
AP Spine -3.9
Which of the following is the correct interpretation
of this DEXA scan? Total Hip -3.3
Alendronate is recommended as a first line According to the NIH consensus statement, the
treatment option for the secondary prevention of following parameters can be used as indications
osteoporotic fragility fractures in postmenopausal for parathyroidectomy in asymptomatic patients
women who are confirmed to have osteoporosis. with primary hyperthyroidism:
Risedronate and etidronate are recommended as
alternative treatment to alendronate, if it is Age < 50 years
contraindicated. In this case, the patient was T score of < -2.5 on bone mineral
intolerant of alendronate, is likely to be intolerant densitometry (BMD)
of other oral bisphophonates and so the next most Plasma calcium > 2.85 mmol/L
appropriate treatment would be strontium. IV Urinary calcium excretion of 10 mmol/d
Zolendronate is another possibility but arguably (or > 400 mg/day)
there is a lot less hassle providing the patient with Decrease in creatinine clearance by >30%
an oral preparation, so making it the preferable as compared to age matched normal
choice. subjects
Number of independent clinical risk factors for Other indications for parathyroidectomy in
fracture primary hyperparathyroidism include
25-OH Vitamin (desirable above score at the lumbar spine has fallen from -2.8 to -
32 nmol/L 3.5 and at the neck of femur it is stable at -3.6.
D 30)
7.2
PTH (1.0-6.0) Which of the following management options
pmol/L
would you select for the management of this
Which of the following results/ observations will patient?
go against the suggested diagnosis? (Please select 1 option)
2- Calcium excretion of < 50 mg/ day 2- Continue strontium ranelate unless fractures
again
3- History of renal stones
3- Start ibandronate monthly treatment and
4- Normal renal function repeat DEXA in 12 months
5- Patient on Thiazide diuretics 4- Start teriparatide treatment for 12 months
5- Start teriparatide treatment for 18 months
Answer & Comments
Answer: 2- Calcium excretion of < 50 mg/ day Answer & Comments
Answer: 5- Start teriparatide treatment for 18
Familial hypocalciuric hypercalcaemia (FHH) is an
months
autosomal dominant condition secondary to a
mutation of the calcium-sensing receptor leading
to a decreased excretion of calcium from the body, NICE guidelines recommend the use of
with an end result of mild hypercalcaemia. It is a teriparatide for secondary osteoporosis in women
benign disorder and an important differential who are unable to take bisphosphonates or have a
diagnosis for primary hyperparathyroidism (PHT). contraindication to or are intolerant of
The key distinguishing features include: alendronate and either risedronate or etidronate;
or who have had an unsatisfactory respond to
A family history of mild hypercalcaemia treatment with bisphosphonates and who are 65
No evidence of end organ damage years or older and have a T-score of - 4.0 SD or
(normal renal function, absence of below, or a T-score of - 3.5 SD or below plus more
nephrolithiasis, no evidence of bone than two fractures, or who are aged 55-64 years
disease) and have a T-score of -4 SD or below plus more
Decreased 24 hour urinary calcium than two fractures.
excretion (<50 mg/day or < 2.5 mmol/day)
or fasting calcium/creatinine excretion Reference:
ratio of < 0.01 http://www.nice.org.uk/nicemedia/live/11748/
42508/42508.pdf
[ Q: 140 ] Question ID #555
[ Q: 141 ] Question ID #617
A 67 year old woman comes to see you in
osteoporosis clinic. She has had 2 vertebral A 65 year old gentleman presents with deformity
fractures in the past. Her GP has given her and some discomfort in his tibia. His bloods show
alendronate and residronate but she developed raised alkaline phosphate with normal calcium and
oesophagitis with both. She was taking strontium phosphate levels. His skull and spinal X rays
ranelate which was withdrawn and now continues confirm the diagnosis of Paget`s disease.
on raloxifene together with calcium and vitamin D.
Her DEXA scan was repeated and shows her T-
Which one of the following is the most appropriate Answer & Comments
treatment for his underlying condition?
Answer: 5- X-ray of tibia
(Please select 1 option)
4- MRI tibia
Teriparatide is recommended for treatment of
5- X-ray of tibia severe osteoporosis (osteoporosis in presence of
1 or more fragility fractures) in patients in whom
treatment with bisphosphonates has been
unsatisfactory. The high cost associated with this
agent restricts their use to only a selective group OPG in contrast binds to RANKL and inhibits its
of patients. Teriparatide is contraindicated in interaction with RANK receptor. Therefore
presence of renal impairment (eGFR < 30 inhibition of RANKL factor leads to down
mL/minute/ 1.73 m2), severe hepatic impairment, regulation of osteoclast production.
hypercalcaemia, metabolic bone disease (Paget`s
disease, primary hyperparathyroidism) and
[ Q: 145 ] Question ID #638
unexplained rise in alkaline phosphatase levels.
1- Lining cells
This gentleman has two clinical risk factors (CRFs)
2- Marrow cells for development of osteoporosis- rheumatoid
3- Osteoblasts arthritis and usage of steroids. Both of these are
moderate risk factors for the development of
4- Osteoclasts osteoporosis in contrast with smoking and alcohol
intake which are considered weak CRFs. Parental
5- Osteocytes
history of hip fracture and previous fragility are
generally classified as strong risk factors for
Answer & Comments osteoporosis development. According to the
national guidelines for diagnosis and management
Answer: 3- Osteoblasts
of osteoporosis1, treatment with bisphosphonates
should be considered if the T score on BMD
The communication between Osteoblasts and assessment is < -1 in presence of 2 or more CRFs
Osteoclasts seems to occur as a result of locally and strongly recommended if T score is < -2.5.
produced signals which include osteoclast
differentiation factor called RANK ligand (RANKL) Reference:
and osteoprotegerin (OPG). RANKL is a protein Guideline for the diagnosis and management of
derived from OSTEOBLASTS, which binds to the osteoporosis in postmenopausal women and men
RANK receptor and activates NF-?B on the surface from the age of 50 years in the UK, Produced by JA
of OSTEOCLAST precursors leading to increased Kanis, J Compston, A Cooper, C Cooper, R
osteoclast production. Francis,D Marsh, EV McCloskey, D Reid, P Selby
and M Wilkins, on behalf of the National
Osteoporosis Guideline Group (NOGG).
Pseudohypoparathyroidism (PHP) is a
heterogeneous group of disorders characterized This lady appears to have sub-optimal thyroid
by low calcium, elevated phosphate levels and hormone replacement which is likely to contribute
resistance to PTH activity. These individuals have to her dyslipidaemia. Hypothyroidism is a
short statue, round face and shortened fourth recognised risk factor for lipid lowering therapy
metacarpals. associated myoptoxicity, therefore prior to
escalating her specific lipid lowering medications
it is important to correct her thyroid profile.
[ Q: 147 ] Thyroid - Question ID #35
[ Q: 148 ] Question ID #50
A 53 year old lady with a 3 year history of type 2
diabetes is reviewed in clinic; she has a previous
history of hypothyroidism but has no other A 42 year old South Asian gentleman is referred to
medical history of note. Her current medication the diabetic clinic by his GP. He has a 5 year history
includes metformin 500mg three times a day, of type 2 diabetes. He complains of feeling tired
simvatatin 20 mg at night, ramipirl 2.5 mg once a and has a previous history of ischemic heart
day and thyroxine 100 mcg once a day. disease.
On examination her BMI is 32 kg /m2 blood On examination BMI 34 kg m2, pulse 52 bpm,
pressure 135/85 mmHg and her pulse is 62 beats blood pressure 148/78 mm/Hg. Current
per minute. medication consists of 'Novomix 30/70' mixed
insulin - 42 IU twice daily, metformin 500 mg twice
Sensitivity is a measure of how likely a test is to both TPO and TSH Receptor antibodies are
pick up the presence of a disease in a person who negative. The most appropriate way of
has it. Sensitivity is calculated by dividing the determining if there is a functional adenoma is
number of test positives, by the test positives plus through a RAI uptake scan. An ultrasound of the
the test negatives. Therefore, in this case thyroid really provides no useful information in
sensitivity = 40 ÷ (40 + 10) = 0.8, therefore the this scenario.
answer is 80%.
[ Q: 154 ] Question ID #98
Reference:
http://courses.ncssm.edu/math/Stat_Inst/Stats
2007/Stat%20and%20Calc/Sensitivity%20an A 28 year old female presents with anxiety and
d%20Specificity.pdf tremor. On examination she has a pulse of 88 bpm,
a fine tremor of the outstretched hands and a
modest goitre with a faint bruit. There are no eye
[ Q: 153 ] Question ID #97 signs. Her investigations reveal:
A 33 year old female presents with a 3 month Free T4 38.9 pmol/l (9 - 22)
history of agitation. On examination, she has a fine
TSH 2.1 mu/l (0.5 - 5)
tremor of the out-stretched hands and lid lag.
There is no palpable goitre. Her investigations TSH receptor Antibody 1.1 mu/l (<2)
reveal:
An MRI of the pituitary reveals a 0.9 cm adenoma
Free T4 15.5 pmol/l (9 - 23) of the pituitary.
TSH <0.02mu/l (0.5 - 5)
Which of the following would be the most
Free T3 9.9 pmol/l (3 - 6) appropriate initial treatment for this patient?
TSh Receptor Antibody 1.1 u/l (<2) (Please select 1 option)
TPO antibodies 105 u/l (<140)
1- Ablative radioactive iodine
1- Leave dose of thyroxine unchanged What is the most appropriate investigation for this
2- Reduce dose of thyroxine to 100 patient?
micrograms/d (Please select 1 option)
PRL and GH typical of a mutation of the The most appropriate first line treatment for this
transcriptional activator - pit-1. Prop-1 mutations patient who has recurrent thyrotoxicosis following
(Prophet of Pit-1) are the most common defects in anti-thyroid treatment would be radioactive
combined childhood pituitary hormone defiencies iodine. She should ensure that she does not
and are characterised by LH/FSH deficiency become pregnant for six months after treatment
together with GH, TSH and Prolactin. Later, ACTH but she is already taking the OCP. There is no
deficiency may ensue. Imaging may reveal a small report of eye disease associated with the Graves’
or indeed normal appearance of the pituitary and so concomitant steroids is probably unnecessary.
does tell us nothing of the functionality. Following radioiodine treatment, approximately
80% of patients become hypothyroid.
[ Q: 159 ] Question ID #109
[ Q: 160 ] Question ID #110
A 24 year old female has received a six month
course of carbimazole for Graves disease. She is a A 28 year old female who is one month post-
non-smoker, with no family history of thyroid partum develops tiredness. Thyroid function tests
dysfunction. She recently stopped treatment and reveal:
attends two months later for subsequent review
and complains of recurrence of her previous Free T4 25.6 pmol/l (9 - 22)
symptoms. On examination, she has a modest
TSH <0.02 mu/l (0.5 - 5)
goitre with pulse of 80 bpm and has a fine tremor
of the outstretched hands. There are no eye signs TPO antibodies 550 iu/l (<140)
associated with Graves disease. The results of her
thyroid function tests reveal: When she is seen 2 months later she is still tired
and emotionally labile. The results of TFTs reveal:
Free T4 32.3 pmol/l (9 - 22)
TSH <0.02 mu/l (0.5 - 5) Free T4 8.5 pmol/l (9 - 22)
TSH 12 mu/l (0.5 - 5)
She informs you that she is keen on getting
pregnant after about 6 months time (as she is due She is treated with thyroxine 50 micrograms daily
to get married in the summer). She is currently and her thyroid function tests improve. She is
taking the oral contraceptive, microgynon. reviewed regularly and you see her one year later
where she is well and has with the following
Which of the following would be the most thyroid function tests:
appropriate action for this patient?
(Please select 1 option) Free T4 21.5 pmol/l (9 - 22)
1- Continue with further six months TSH 0.3 mu/l (0.5 - 5)
propylthiouracil
Which of the following is the most appropriate
2- Continue with long term propylthiouracil action for this patient?
3- Continue to treat with long term carbimazole (Please select 1 option)
5- Stop thyroxine and reassess thyroid function This patient appears to have developed episodic
and profound muscle weakness. The potassium is
normal on admission but the patient has
Answer & Comments
recovered fully. The history is compatible with
Answer: 5- Stop thyroxine and reassess thyroid hypokalaemic periodic paralysis yet the
function tachycardia and recent weight loss is quite
compatible with thyrotoxicosis. Therefore, a
diagnosis of thyrotoxic periodic paralysis should
This patient is likely to have post-partum
be considered. During attacks, hypokalaemia
thyroiditis as suggested by the initial
typically occurs and the condition is best treated
hyperthyroidism with subsequent hypothyroidism
with restoration of euthyroidism.
occurring post-partum. Approximately 30% have
persistent hypothyroidism but the majority
recover. There is a strong chance of further thyroid [ Q: 162 ] Question ID #126
dysfunction in subsequent pregnancies. The most
appropriate approach therefore, is to stop A 35 year old female presented with a three
thyroxine and reassess thyroid function in month history of 3kg weight loss and anxiety. On
approximately one month. examination, she had mild tremors of the
outstretched hands, proptosis and lid lag.
[ Q: 161 ] Question ID #113 Examination of the neck reveals an asymmetrically
enlarged goitre with a 3.5cm irregular nodule
palpable in the left lobe. Investigations reveal:
A 21 year old male is admitted with a one hour
history of profound muscle weakness such that he
was immobile and unable to move his arms or legs. Free T4 24 pmol/l (9-23)
He was otherwise well, had noted some weight TSH <0.22 mu/l (0.5-5)
loss of late but denied use of illicit substances. He TPO antibodies >1500 mu/l (<150)
had one similar episode two months ago from
which he recovered fully. TSH-R antibodies 3.3 iu/l (<1)
Calcium 2.65 mmol/l (2.2 -2.6)
2
On admission, his BMI was 21 kg/m , his pulse was U+Es Normal
102bpm, blood pressure was 112/80 mmHg and
he had full muscle strength. Investigations reveal:
The patient was initiated on carbimazole therapy.
This woman has Graves’ disease and also has a left This patient has mild hypercalcaemia, an
sided discrete nodule. In any nodule one should inappropriately normal PTH concentration and has
consider the possibility of a carcinoma and the a normal/highish urine calcium excretion. The
most appropriate initial investigation would be a latter would argue against a diagnosis of FHH
FNA which would be diagnostic. This is not likely to where urine calcium excretion is low (less than 5
be a toxic adenoma as the patient has positive TSH mmol/d) and so the likely diagnosis is primary
receptor antibodies indicating Graves’ disease. hyperparathyroidism.
Therefore, radioiodine uptake scan would merely
reveal increased uptake and the nodule may be
[ Q: 164 ] Question ID #128
cold or hot – this procedure would not be
diagnostic. Ultrasound would add little diagnostic
information. The slight elevation of calcium is not A 32 year old female presents with a six month
unusual in thyrotoxicosis. history of weight loss and anxiety. She does not
report any visual symptoms and her visual acuity
is normal. She is diagnosed with Graves’ disease
[ Q: 163 ] Question ID #127 based on confirmatory thyroid function tests and
a positive TSH receptor antibody. She is treated
A 45 year old woman is referred by her GP for with carbimazole and thyroxine in a block and
investigation of her hypercalcaemia. He noted the replacement regime for approximately six
following investigations: months. Treatment is then stopped. She re-
attends clinic three months after stopping
U+Es Normal treatment and complains of once again feeling
rather anxious and having poor sleep. She is a non-
Calcium 2.8 mmol/l (2.2 - 2.6)
smoker and drinks little alcohol.
Phosphate 1.0 mmol/l (0.8 - 1.4)
On examination, she has a fine tremor of the
The patient originally attended the GP with vague outstretched hands, lid lag and a goitre. Her
aches and pains which have been present for two thyroid function tests reveal:
months or so and she also admits to feeling rather
depressed. She is only taking Fluoxetine 20 mg Free T4 38.2 pmol/l (9 - 23)
daily at present. There is nil of note on
TSH <0.05 mu/l (0.5 - 5)
examination. Further investigations reveal:
A 42 year old female is seen due to anxiety and 3- Previous treatment with thionamides
weight loss. She is found to have Graves disease 4- Smoker of 10 cigarettes per day
with positive TSH receptor antibodies. The
physician elects to treat her with a block and 5- Use of the oral contraceptive
replacement regime consisting of carbimazole and
thyroxine for six months. He explains at that stage Answer & Comments
treatment will be withdrawn and thyroid function
tests monitored for relapse. The patient enquires Answer: 4- Smoker of 10 cigarettes per day
as to what are the chances of relapse after this
treatment. Factors recognised to contribute to the
development/deterioration of Graves’
Which of the following most accurately reflects the Ophthalmopathy include smoking, rise in TSH and
relapse rate after treatment with antithyroid undertreatment of hypothyroidism together with
drugs? high TSH receptor antibodies.
(Please select 1 option)
2- 20%
A 45 year old female is referred by her GP with
3- 30% deranged thyroid function tests. She has been
complaining of occasional palpitations hence her
4- 50% GP checked investigations and found the
5- 70% following:
This is a relatively standard approach to the TSH Receptor Antibody 0.2 iu/l (<1.5)
treatment of Graves' disease - give a block and
replace regime for between 6-12 months. Studies There is nothing of note on examination with no
suggest that this approach is associated with an palpable thyroid goitre.
approximate 50% long term remission rate.
Which of the following would be the most Which of the following is the likely diagnosis?
appropriate investigation for this patient? (Please select 1 option)
(Please select 1 option)
1- Hashimoto's encephalopathy
1- Request alternative TFT assay at another
2- Lymphocyctic hypophysitis
centre
3- Limbic encephalomyeltis
2- Thyroid FNA
4- MELAS
3- Thyroid microsomal antibodies
5- POEMS
4- Thyroid ultrasound
5- Thyroid uptake scan Answer & Comments
Answer: 1- Hashimoto's encephalopathy
Answer & Comments
Answer: 5- Thyroid uptake scan This patient has a short history of confusion yet
the neurological investigations are normal. The
This patient has subclinical hyperthyroidism and likely diagnosis is therefore Hashimoto's
this is most likely due to a toxic impalpable encephalopathy - considered to be part of an
adenoma. The most appropriate investigation autoimmune encephalitis. The condition was
would be a thyroid uptake scan which may be described in the 1960s by a Prof Brain! The
expected to reveal the toxic adenoma. Then, condition can be progressive or relapsing
treatment with radio-iodine would be remitting and the treatment is steroids. Often, the
appropriate. condition presents prior to the development of
hypothyroidism and patients can be entirely
euthyroid yet with quite profound neurological
[ Q: 168 ] Question ID #301 dysfunction.
1- 50-100 MBq
Pendred syndrome is an autosomal recessively
inherited condition characterised by deafness, 2- 200-400 MBq
goitre, hypothyroidism and is due to defective
3- 500-800 MBq
iodine binding.
4- 900-1100 MBq
[ Q: 171 ] Question ID #348 5- 1200-1500 MBq
The recommended dose of RAI is typically Into which of the following classifications does this
between 500 - 800 MBq with the frequent dose fit?
being 555 MBq. (Please select 1 option)
1- Thy 1
[ Q: 173 ] Question ID #360
2- Thy 2
A 68 year old female presents with recurrent 3- Thy 3
Graves' disease after having received one course
of block and replacement therapy with 4- Thy 4
carbimazole and thyroxine. After discussion, she 5- Thy 5
elects to receive radioiodine treatment. Prior to
receiving treatment she is rendered euthyorid
again with carbimazole. Answer & Comments
Answer: 1- Thy 1
At least how many days prior to her radioactive
iodine should she stop her carbimazole?
The classification of thyroid biopsy samples is
(Please select 1 option)
based upon five criteria:
1- 7 days
Insufficient sample or unable to report
2- 14 days Thy 1:
due to heavy blood staining.
3- 21 days Non-neoplastic; normal thryoid,
Thy 2:
4- 28 days thryoiditis, colloid
In this case, the cytology reports a thyroiditis - Which of the following is the correct classification
benign THY2.
of this report?
(Please select 1 option)
[ Q: 176 ] Question ID #364
1- Thy 1
A 28 year old female undergoes an FNA of an 2- Thy 2
isolated 2cm thyroid nodule This is reported as:
3- Thy 3
'...a follicular neoplasm'.
4- Thy 4
Which of the following is the correct classification
5- Thy 5
of this report?
(Please select 1 option)
Answer & Comments
1- Thy 1
Answer: 4- Thy 4
2- Thy 2
3- Thy 3 The classification of thyroid biopsy samples is
4- Thy 4 based upon five criteria
The classification of thyroid biopsy samples is Thy 3: Neoplasm possible; samples suggesting
based upon five criteria: follicular neoplasm
The thyroid gland is normally located beneath the What is the most appropriate step in her
cricoid cartilage midway between the apex of management?
thyroid cartilage and the supra-sternal notch. It (Please select 1 option)
normally weighs about 10-20 g. The growth of
thyroid gland usually occurs in a downward and 1- FNAC
posterior direction as the anterior/upward growth
2- Observation
of the gland is limited by attachment of sterno-
thyroid muscle to the thyroid cartilage. As a result 3- Radioiodine ablation
an enlarging thyroid gland can expand and grow in
4- Thyroidectomy
retrosternal space compressing the surrounding
structures like trachea and oesophagus. 5- Thyroxine (low dose)
A detectable Tg level in such a scenario is What is the most appropriate modality which can
indicative of the presence of thyroid remnants or be used for his definitive management?
recurrence of the tumour. (Please select 1 option)
Haemoglobin 12.8 g/dL (12-16) although have been irregular for 6 months. On
examination she has BMI of 21 and general
Ferritin 302 pmol/L (30-330)
physical as well as systemic examination is
unremarkable. Her blood show:
What is the most appropriate course of action in
her management for her overall improvement?
Haemoglobin 9.4 g/dL (12-16)
(Please select 1 option)
139 (135-145
1- Increase the thyroxine dose to 275 µg/ day Sodium
mmol/L mmol/L)
[ Q: 188 ] Question ID #478 The following is a list of the major causes leading
to persistently elevated TSH levels despite
adequate thyroxine therapy:
A 24 year old with Type I DM and autoimmune
thyroiditis, presents to the endocrine clinic with
Compliance
history of 6 Kg of weight loss over last 4-6 months.
Drugs such as omeprazole, ferrous
She denies going on any diet or indulging in
excessive exercise. She denies any issues sulphate, calcium carbonate
Malabsorption syndromes like coeliac
regarding compliance with insulin injections and/
or thyroxine tablets. Her appetite is normal and disease
Nephrotic syndrome
denies any nausea and vomiting. Her bowel habits
In her case history of weight loss, alteration in Hashimoto's thyroiditis (increased risk of
bowel habit, low Hb and low corrected calcium lymphoma)
should raise the suspicion of associated coeliac Family or personal history of thyroid
disease. adenoma
Cowden`s syndrome
Familial adenomatous polyposis
[ Q: 189 ] Question ID #479
Familial thyroid cancer
Which of the following is suggestive of the
A thyroid nodule is more likely to be malignant if:
suspicious nature of the histology/radiology of a
thyroid nodule? The affected individual is a child or an
(Please select 1 option) elderly male
It is a firm, solitary, non toxic nodule
1- Epithelial cells
Associated hoarseness of voice and/or
2- Haemorrhage palpable cervical lymph node
In presence of micro-calcification on FNAC
3- Lymphocytes
4- Macrophages [ Q: 190 ] Question ID #481
5- Micro-calcification
A 32 year old lady with known papillary carcinoma
Answer & Comments of thyroid (which was treated with total
thyroidectomy and post operative radio-iodine
Answer: 5- Micro-calcification ablation 3 years ago), presents to the GP for a
routine check up. Presently she is on Thyroxine 75
Ultra-sound features suggestive of thyroid µg/day and otherwise well in herself. Her
malignancy: examination is normal with evidence of a healed
thyroid scar and no palpable cervical lymph nodes.
Her biochemical results are as shown below:
>1cm sized nodule which is taller than it is wide
According to the guidelines issued by British What is the most appropriate next step in her
thyroid Association on management of thyroid management?
cancer, the following are considered to be risk (Please select 1 option)
factors for thyroid cancer:
1- Arrange Ultrasound of the neck
History of neck irradiation in childhood 2- Check Calcitonin levels
Endemic goitre
3- Decrease thyroxine dose
What is the most appropriate next step in her 4- Increase in 31 deiodinase Type 1 levels
management? 5- Increase in TBG levels
(Please select 1 option)
The presence of inappropriately normal or Enlargement of the thyroid gland (goitre) can be
elevated TSH levels in the presence of elevated T3 secondary to physiological states of increased
and T4 levels is indicative of either a TSH secreting iodide demand such as puberty and pregnancy,
pituitary adenoma or Thyroid hormone resistance. congenital defects of thyroid hormone synthesis,
Rarely such a biochemical profile is a result of the iodide deficiency, autoimmune thyroiditis
presence of heterophile antibodies leading to (Hashimoto`s thyroiditis), drug induced
spurious results. TSH secreting tumours usually (amiodarone, lithium, interferon), post partum
present with features of mild thyrotoxicosis and thyroiditis etc.
goitre. The TSH a-subunit secretion is increased in
such tumours and is used to distinguish from In the developed world iodine deficiency goitre is
Thyroid hormone resistance. A molar ratio of a- uncommon as the optimum Iodide requirement of
subunit to TSH of > 5.7 is considered diagnostic in 150 µg per day (according to WHO
such cases. A TRH stimulation test can occasionally recommendations) is easily met through diet. This
be used to distinguish between the two conditions patient has been on a world tour and visited Iodide
but is hardly performed these days. depleted areas such as Himalayas making her
susceptible to Iodide deficiency goitre. As more
In contrast thyroid hormone resistance can either than 95% of dietary Iodide is excreted in urine, a
be generalised, selective pituitary or selective 24 hour urinary excretion of iodide is an excellent
peripheral (resistance to hormone action). Most of index of dietary iodine intake and can unmask a
the resistance syndromes are familial in origin. iodide deficiency state.
Patients are generally clinically euthyroid except in
cases of pituitary resistance to thyroid hormone.
[ Q: 197 ] Question ID #631
The mutation is usually at thyroid hormone
receptor level.
A 75 year old gentleman presents to the Endocrine
clinic with an 8 week history of weight loss,
[ Q: 196 ] Question ID #615 palpitations and tremors. He has past medical
history of ischaemic heart disease and AF. He is
A 32 year old has decided to go on a strict vegan currently taking Aspirin, Ramipril, Metoprolol and
diet for a year. She has recently returned from a Rosuvastatin. He was on Amiodarone which was
world tour and visited sub-Saharan countries and stopped recently by the cardiologist (1 week ago).
went on an expedition in the Himalayas. She His blood results are as shown below:
presents with a diffuse smooth goitre. Her bloods
show a euthyroid status and negative anti TPO TSH < 0.01mU/L (0.35-5.5 mU/L)
antibodies.
Free T4 28 pmol/L (11.5-22.7 pmol/L)
What is the next investigation to establish the Free T3 7.9 nmol/L (3.5-6.5 pmol/L)
likely diagnosis? Anti TPO negative
(Please select 1 option) Uptake < 1%
1- CT neck scan uptake
2- FNAC
What is the most appropriate next therauptic for the last 1 year as far as symptoms of
option for his management? palpitations are concerned. His recent routine
(Please select 1 option)
blood tests show:
(Please select 1 option) What should be the most appropriate step in her
1- Carbimazole management?
(Please select 1 option)
2- Lugols iodine
1- Lobectomy
3- Propanolol
2- Reassure and discharge
4- Propylthiouracil
3- Radioiodine ablation
5- Radioiodine
4- Refer for total thyroidectomy
Answer & Comments 5- Repeat FNAC in 3-6 months
Answer: 4- Propylthiouracil
Answer & Comments
The patient is likely to have Graves` thyrotoxicosis Answer: 5- Repeat FNAC in 3-6 months
as suggested by features of hyperthyroidism and
associated strong family history of autoimmune
The guidelines from the British Thyroid
thyroid disease. In view of her pregnant state,
Association (BTA) recommend repeating cytology
treatment with lowest dose of Propylthiouracil,
with repeat FNAC in those nodules where THY2
which renders her euthyroid, should be initiated.
results are obtained at 3-6 month interval.
The following are few of the salient points to be
remembered as far as pregnancy and thyroid
dysfunction is concerned: [ Q: 201 ] Question ID #692
first and, in the vast majority of cases after US, the 4- Oral carbimazole
only test required for the evaluation of a solitary
5- Oral propylthiouracil
thyroid nodule.
Answer: 1- Drug-induced
A 23 year old female presents with a two year
history of failure to conceive and
This patient is taking a dopamine antagonist – oligomenorrhoea. She is investigated, found to
olanzapine which will cause hyperprolactinaemia. have a prolactin of 1015 mu/l (50-500) and is
The fact that the menstrual function is normal and diagnosed with a microprolactinoma. She explains
estradiol is normal would argue against the that she is keen to conceive.
hyperprolactinaemia being pathological due for
instance to a microprolactinoma. Which of the following is the most appropriate
treatment for this patient?
Reference:
(Please select 1 option)
Drug-induced hyperprolactinaemia
1- Bromocriptine
[ Q: 204 ] Question ID #68 2- Pituitary surgery
3- Pergolide
A 72 year old female is diagnosed with acromegaly
due to a macroadenoma of the pituitary and 4- Radiotherapy
underwent surgery. Surgical resection was 5- Somatostatin analogues
unsuccessful, no further surgery was
recommended and she had received radiotherapy
2 years ago. However, her growth hormone Answer & Comments
concentrations on day profile and IGF-1
Answer: 1- Bromocriptine
concentrations remain elevated despite the use of
octreotide LAR 30 mg every 4 weeks.
Bromocriptine is the treatment for
Which of the following is the most appropriate hyperprolactinaemia that is licensed for use in
management for this patient’s ongoing patients wishing to conceive. Quinagolide is also
acromegaly? licensed. Surgery is rarely required for the
treatment of prolactinomas.
(Please select 1 option)
amenorrhoeic. Her most recent MRI scan reveals Answer & Comments
that the tumour has shrunk since the original scan
of 3 months ago but she still has a 2 cm pituitary Answer: 1- GH receptor antagonist
tumour which is just touching the chiasm. Her
visual fields are full to confrontation. Pegvisamont is a GH receptor blocker and results
in a decline in IGF-1 concentrations in active
Which of the following is the most appropriate acromegaly. It is generally reserved for the
treatment for this patient? treatment of active acromegaly that is resistant to
(Please select 1 option)
somatostatin analogues.
A 72 year old female is receiving treatment with Which of the following is the most appropriate
pegvisamont for acromegaly which is management strategy for this patient?
unresponsive to somatostatin analogues. (Please select 1 option)
What is the most appropriate management for 3- Refer for pituitary surgery
this patient? 4- Treat with testosterone only and repeat MRI
(Please select 1 option) in 3 months
1- Refer for pituitary radiotherapy 5- Treat with somatostatin analogue
2- Refer for pituitary surgery
5- Refer for intracytoplasmic sperm injection these agents is harmful to the fetus. However,
(ICSI) there is no need to take these agents during
pregnancy as the risks of tumour expansion are
small (<1%). Thus, the patient should be advised to
Answer & Comments
stop cabergoline only.
Answer: 3- Stop testosterone and commence HCG
injections [ Q: 214 ] Question ID #96
What is the most appropriate investigation for this On examination he has a BMI of 37.5 kg/m2, a
patient? blood pressure of 136/88 mmHg and a pulse of 68
(Please select 1 option) bpm. No abnormalities are found in
cardiovascular, chest or abdominal examination.
1- Request KAL-1 mutational analysis
Further results are obtained which reveal:
2- Request MENIN mutational analysis
3- Request MRI of the pituitary and olfactory U+Es Normal
bulbs Fasting Plasma Glucose 6.1 mmol/l
most probable cause of his hypogonadism is She now presents to clinic and explains that she is
obesity. 8 weeks pregnant.
This is a well described, emerging phenomena and What do you advise regarding her management?
the exact cause is ill-understood although is (Please select 1 option)
believed to directly relate to visceral fat mass with
increasing aromatisation of testosterone to 1- Advise surgery
oestradiol and then feedback on the Pituitary. 2- Continue somatostatin analogue therapy at
current dose
[ Q: 219 ] Question ID #123
3- Increase dose of somatostatin analogue
therapy
A 44 year old male is diagnosed with a pituitary
macroadenoma. He undergoes trans-sphenoidal 4- Stop depot somatostatin analogue therapy
resection of this lesion and the histology is
5- Switch to tds short acting analogue
reported as a chromophobe adenoma.
somatostatin therapy.
Which of the following would be the likely cause of
this histological description? Answer & Comments
(Please select 1 option) Answer: 4- Stop depot somatostatin analogue
1- ACTH secreting adenoma therapy
2- GH secreting tumour
SMS analogue therapy is not licensed for
3- Non-functional adenoma pregnancy and should be stopped. In this scenario,
with the patient being effectively cured with SMS
4- Prolactinoma
therapy in the presence of a microadenoma, it
5- TSH secreting adenoma would seem most appropriate to stop analogue
therapy. However, many experts, particularly in
the context of a macroadenoma may take the
Answer & Comments
approach of continuing treatment as no ill effects
Answer: 3- Non-functional adenoma have been reported. This is quite an unusual
scenario as surgery would have been considered
first line treatment for acromegaly, yet many
A chromophobe adenoma does not take up any
young women who may wish to begin a family,
dye – chromo-phobe. Hence this is typical of a
may elect SMS analogues in the first instance as
non-functional pituitary tumour. Eosinophilic
the risks of hypopituitarism after surgery are quite
adenomas are typically GH or Prolactin producing
high.
whereas basophilic describe ACTH tumours.
fields are normal to confrontation. Her results His other investigations reveal:
reveal:
IGF-1 10.8 nmol/l (28-42)
Prolactin 1040 mu/l (50-450) Testosterone 14.5 nmol/l (9-30)
TFTs Normal FreeT4 18.2 nmol/l (9-23)
Answer: 2- Acromegaly and impaired glucose A 44 year old female presents with amenorrhoea,
tolerance weight gain and ease of bruising. She is diagnosed
with Cushing’s syndrome based on elevated 24hr
This patient has a paradoxical rise in GH urine free cortisol concentrations of 880 nmol/d
concentrations during the OGTT and this indicates (<250) and a cortisol of 88 nmol/l after low dose
acromegaly as the GH concentrations should dexamethasone test. An ACTH is 108 ng/l (20-60).
suppress below 2 mu/l. With regard to the glucose An MRI is performed of the pituitary and is
concentrations, the glucose at 120 mins is 8.8 reported as normal.
mmol/l indicating impaired glucose tolerance.
Which of the following is the most appropriate
next step for this patient?
[ Q: 224 ] Question ID #140
(Please select 1 option)
ACTH would clinch the diagnosis and sensitivity is treatment is prednisolone and beta blockade and
improved following CRF. if possible withdrawal of amiodarone. This
contrasts with AIT type 1 where the patient
High dose dexamethasone was once considered to typically has pre-existent thyroid disease or
differentiate between Cushing’s disease and positive thyroid auto-antibodies. Thyroid uptake
ectopic but this has proven not to be the case and scan may be useful in differentiating between the
generally the test adds little to results of Low dose two as in AIT type 1 uptake may be increased and
testing and so is used infrequently. CT of adrenals low in type 2, but this is not often the case as
would not provide diagnostic information as it is uptake is typically very low due to competition of
ACTH dependent disease. radio-iodine with the iodine in amiodarone.
A 66 year old male with a past history of ischaemic A 28 year old male with hypopituitarism due to a
heart disease and episodes of ventricular non-functioning pituitary tumour now wishes to
tachycardia has been treated with amiodarone start a family. He underwent surgery at the age of
200 mg for two years. His VT is well controlled on 20 and since has been treated with Testostrerone
this treatment but latterly he presents with gel 50mg daily, hydrocortisone 10mg bd,
fatigue, anxiety and weight loss. thyroxine 100 micrograms daily and growth
hormone 1.5 mg daily. He has normal erectile
On examination, he has a fine tremor of the function but a most recent semen analysis
outstretched hands, a pulse rate of 88 bpm but is demonstrates azoospermia.
receiving beta blockers, has lid lag but no goitre is
palpable. His Thyroid function tests reveal: Which of the following is the most appropriate
approach to fertility on this man?
Free T4 55 pmol/l (10-23) (Please select 1 option)
TSH <0.02 (0.5-5) 1- Continue testosterone and commence HCG
Thyroid Antibodies Negative injections
2- Refer for ICSI
Which of the following is the likely diagnosis?
(Please select 1 option) 3- Refer for sperm donation
Answer & Comments therapy and the prior history of fits is a contra-
indication to IST. Therefore, the most appropriate
Answer: 1- AGHDA quality of life score test would be GHRH/Arginine.
NICE recommend the monitoring of the Although IGF-1 concentrations may be low in GHD,
effectiveness of GH therapy with the AGHDA they may still reside within the normal range and
score. Patients should begin with an AGHDA score are not used diagnostically.
above 11 which reflects a poor quality of life –
(normal scores are 4 or less). [ Q: 232 ] Question ID #156
Free T4 8.1 pmol/l (9.8 – 23) To what does the sensitivity of the new test
approximate?
TSH 1.5 mu/l (0.35-5)
(Please select 1 option)
Prolactin <50 (50-450)
Estradiol <100 (130 – 850) 1- 50%
Which of the following is most likely the diagnosis? Answer & Comments
(Please select 1 option) Answer: 4- 80%
1- Factitious results
Assuming that the MRI is the gold standard here,
2- Lymphocytic hypophysitis
sensitivity of the test is the number of true
3- Non-functional pituitary tumour positives (80)/ number of true positives (80) +
false negatives (20). In this case, sensitivity is 80%.
4- Post-partum thyroiditis
In the case of specificity, this relates to the number
5- Sheehan syndrome of true negatives (800)/ the number of true
negatives (800) + false positives (100). This is
approximately 90% (88.9%).
Plasma
Day Sodium Potassium Urea 4.5 Creatinine [ Q: 238 ] Question ID #344
1 131 3.6 mmol/l mmol/l 95 µmol/L
mmol/l
A 38 year old female presents with tiredness,
Plasma
Day Sodium Potassium Urea 4.2 Creatinine
lethargy and weight gain. Five years previously she
2 126 3.4 mmol/L mmol/L 88 µmol/L had undergone trans-sphenoidal hypophysectomy
mmol/L for a non-functional pituitary tumour. She had a
Plasma complicated post-operative course having
Day Sodium Potassium Urea 5 Creatinine developed a CSF leak which required re-
3 123 3.5 mmol/L mmol/L 88 µmol/L exploration and then seizures for which she
mmol/L
remains on sodium valproate. She was
Plasma demostrated to be hypopituitary for which she
Day sodium Potassium Urea 5 Creatinine
receives hydrocortisone 10mg bd, thyroxine 100
4 120 3.5 mmol/L mmol/L 88 µmol/L
mmol/L micrograms/d and combined HRT. You suspect
that a growth hormone deficiency is likely to be
responsible for her current symptoms.
On day 4 she was commenced on a fluid restriction
of 1 litre per day. The following were noted at that
stage: Plasma osmollity 258 mosmol/l (275-305) Which of the following investigations would be the
urine osmolality 688 mosmol/l with urine sodium most appropriate test to confirm growth hormone
84 mmol/l. deficiency in this patient?
(Please select 1 option)
Which of the following is the most likely diagnosis
1- Clonidine test
to explain the hyponatraemia?
(Please select 1 option) 2- Combined GHRH/Arginine test
[ Q: 243 ] Question ID #647 What is the likely advice you will give?
(Please select 1 option)
A 65 year old male with an inoperable ACTH- 1- Continue the same dose of pegvisomant
secreting pituitary adenoma, undergoes bilateral
adrenalectomy. 2- Decrease the dose
3- Repeat radiotherapy
The follow up for this gentleman should include the
following investigations? 4- Start him on octreotide
Pegvisomant is a genetically modified analogue of consistent with pregnant state and needs further
growth hormone which acts as a highly selective confirmation with urinary pregnancy test /
growth hormone receptor antagonist. It is licensed Ultrasonography.
for treatment of patients with acromegaly who
have undergone incomplete/ inadequate surgical [ Q: 246 ] Question ID #677
resection and are unresponsive to somatostatin
analogue therapy. It leads to normalisation of IGF-
1 levels in more than 90% patients although the A 23 year old man, with muscular build, presents
GH levels need to be monitored as it may show an to the clinic with reduced libido. He visits the gym
increase. regularly and denies usage of any substance of
abuse. His BMI is 30kg/m2 and normal secondary
Usage of pegvisomant is limited due to the high sexual characteristic development with some acne
cost of therapy. The above mentioned patient is on back. His blood results are as shown below:
showing good response to Pegvisomant therapy as
reflected by normalisation of IGF-1 levels so the FSH 1.8 U/L (1-10)
therapy should be continued at the same dose LH 3.2 U/L (3-10)
with regular monitoring of liver function tests.
Testosterone 8.1 nmol/L (9-28.5)
Prolactin 220 mU/L (45-400)
[ Q: 245 ] Question ID #666
LFT normal
A 26 year old is referred to Endocrinology clinic MRI Brain normal
with a 3 months history of amenorrhoea (but no
galactorrhoea) due to a suspected What is the likely cause for his hypogonadism?
microprolactinoma. Her investigations show; (Please select 1 option)
the aryl- hydrocarbon Receptor Interactive Protein HLA DR3 / DR4 on chromosome 6 are common
(AIP) within the 11q13 region. The mechanism genetic markers for type 1 diabetes. Diabetes
underlying this propensity to pituitary tumour associated with polyglandular endocrinopathy
formation is unknown but recognition of the (Schmidt syndrome) preferentially occurs in
diagnosis s important in identification of at risk middle age women and is strongly associated with
kindreds. HLA-DR-3. Type 1 diabetes has a higher prevelance
in northern European and Scandanavian countries
and relatively low prevalence in Asia. The
[ Q: 249 ] Other - Question ID #10
histological features of islets from patients with
late onset type 1 diabetes are similar to those from
A 22 year old law student is referred to clinic with patients with classical type1 diabetes, although
a 2 month history of progressive polydypsia and there may be differences in major
polyuria and also complains of unplanned weight histocomapatability complex expression, with
loss in the region of 5 kg over the last 3 months. fewer islets being effected in late onset compared
She has a 25 year old sister diagnosed with type 1 with classical type 1 diabetes.
diabetes 5 years previously. On examination her
BMI is 20 kg / m2 her body weight is 64 kg, her
blood pressure is 112 / 66 mm/hg and visual acuity [ Q: 250 ] Question ID #11
6/ 9 in both eyes.
A 33 year primary school teacher is referred to
Investigations reveal: clinic by her GP. She has recently moved in to the
area having been diagnosed with diabetes 5 years
Fasting plasma glucose 14.9 mmol/l (3.5 - 5.6) previously and commenced immediately on
insulin. She however denies any history of weight
Hba1c 13.1% (<6) loss or osmotic symptoms prior to initiation on
13.9 pmol/l (9.3 - insulin. She has a younger brother recently
T4
23.4) diagnosed with diabetes. Her glycaemic control
TSH 0.51 mu/l (0.32 - 5) has always been good and she describes
hypoglycaemic symptoms once to twice weekly.
Which of the following is a recognized feature of On examination her BMI is 25 kg / m2 her body
weight is 64 kg, her waist circumference is 69 cm,
type 1 diabetes?
her blood pressure is 120 / 65 mmHg and her
(Please select 1 option) visual acuity is 6 / 6 in both eyes. Her current
1- Association with a single susceptibility locus medication includes insulin glargine 8 units at
night combined with novorapid 4 units prior to
2- HLA genes on chromosome 6 are closely lunch and dinner.
linked to immune modulation
3- High prevalence in Asia Investigations reveal:
4- More Islet cell destruction in late onset type Hba1c 6.3% (<6)
1 diabetes compared with classical type1
Islet cell
diabetes. Negative
antibodies
5- Malnutrition in utero may be linked with GAD antibodies Negative
increased risk of developing Type 1 diabetes
TSH 0.36 mu/l (0.32 - 5)
>99 ml/min 1.73m2 (90 -
Answer & Comments eGFR
110)
Answer: 2- HLA genes on chromosome 6 are
closely linked to immune modulation What is the genetic basis of this lady’s diabetes?
(Please select 1 option)
This patient has Maturity onset diabetes of the 1- Apo E2 / E2 genotype with increased LDL and
young (MODY) based on the clinical presentation. chylomicron concentrations.
The diagnosis of MODY is supported the following 2- Apo E2 / E 4 genotype with increased LDL and
clinical characteristics chylomicron concentrations.
Mild to moderate hyperglycemia (typically 3- Apo E4 / E4 genotype with increased LDL and
130-250 mg/dl, or 7-14 mM) discovered chylomicron concentrations.
before 30 years of age.
4- Apo E2 / E2 genotype synthesis with
increased IDL and chylomicron
A first degree relative with a similar
degree of diabetes. concentrations.
5- Apo E 4/ E4 genotype with increased VLDL
Absence of positive antibodies or other and chylomicron synthesis.
autoimmune conditions (e.g thyroiditis) in
patient and family.
Answer & Comments
Persistence of a low insulin requirement Answer: 4- Apo E2 / E2 genotype synthesis with
(e.g., less than 0.5 u/kg/day) past the increased IDL and chylomicron
usual quot;honeymoon" period.
concentrations.
Absence of obesity (though obese people
can get MODY), or other problems Type III hyperlipoproteinaemia occurs as a
associated with type 2 diabetes or consequence of elevated chylomicron and IDL
(intermediate density lipoprotein) concentrations.
Cystic kidney disease in patient or close Also known as broad beta disease or
relatives. dysbetalipoproteinemia, the most common cause
for this form is the presence of Apo E2/E2
Non-transient neonatal, or apparent type genotype. It is due to cholesterol-rich VLDL (β-
1diabetes with onset before 6 months of VLDL). Prevalence is 0.02% of the population.
age. Apolipoprotein E (APOE) is an apoprotein found in
chylomicron and IDL particles that binds to a
Inheritance is autosomal dominant with specific receptor on liver cells and peripheral cells.
penetrance of 40 – 90%, thus most patients will It is essential for the normal catabolism of
have other family members with diabetes. triglyceride-rich lipoprotein constituents. APOE
was initially recognized for its importance in
lipoprotein metabolism and cardiovascular
disease. More recently, it has been studied for its
role in several biological processes not directly
related to lipoprotein transport, including
Fasting plasma
Turbid Plasma 6.8 mmol/l (3.5- 5.6)
glucose
Total Cholesterol 7.1 mmol/l (<5)
Total cholesterol 8.1 mmol/l ( < 5)
Plasma Triglyceride 8.9 mmol/l (0.9 - 2.2)
Plasma
Apo E2 / E2 Genotype Positive 9.5 mmol/l (0.9 - 2.2)
Triglyceride
TSH 2.1 mu / l (0.32 - 5.5) TSH 2.1 mu / l ( 0.32 - 5.5)
AST 89 iu / l (5 - 45)
What would be the drug of choice to treat this
patient’s dyslipidaemia? ALP 112 iu / l (30 - 115)
(Please select 1 option) Gamma GT 96 iu / l (8 - 70)
A 32 year old man is referred to the lipid clinic [ Q: 255 ] Question ID #38
following a routine fasting lipid profile carried out
by his GP which reveals a total cholesterol level of
A 48 year old lady is referred to the lipid clinic
8.3 mmol/l (<4) and a plasma triglyceride level of
following a fasting lipid profile checked by her GP.
18.6 mmol/l ( <1.7). He has no family history of
She has no family history or past medical history
note, denies any alcohol excess and other
of note. She is a non-smoker, is receiving no
investigations include a fasting plasma glucose of
regular medication and drinks alcohol
5.3 mmol/l (<5.6), normal thyroid function tests,
infrequently. On examination her blood pressure
liver function tests and renal function. On
is 160 / 85 mmHg, her BMI is 33 kg / m2 her waist
examination his blood pressure is 135 / 75 mm/hg,
his BMI is 27 kg / m2 and no tendon xanthomata circumference is 92 cm and she has no evidence of
tendon xanthomata.
can be found.
Investigations reveal:
Which of the following is a feature of this
dyslipidaemia?
Fasting Plasma Glucose 6.8 mmol/l (3.5 - 5.6)
(Please select 1 option)
2- Diagnosis is based on LDL and total The most recent addition to the database of genes
cholesterol levels above the 75th centile in which defects cause FH is one encoding a
combined with appropriate family history, member of the proprotein convertase family,
tendon xanthomata and xanthelasma PCSK9. Rare dominant gain-of-function mutations
in PCSK9 cosegregate with hypercholesterolemia,
3- Heterozygous FH has a population and one mutation is associated with a particularly
prevalence of 1:5000 in most countries severe FH phenotype. Expression of PCSK9
4- LDL receptor mutations account for 90% of normally down-regulates the LDL-receptor
cases of FH. pathway by indirectly causing degradation of LDL-
receptor protein, and loss-of-function mutations
5- Thickening of the Achilles tendon is a in PCSK9 result in low plasma LDL levels.
characteristic physical sign associated with
FH The diagnosis of FH is primarily based on clinical
grounds and may be confirmed by genetic testing.
Typical clinical features include cholesterol
Answer & Comments
deposition in the tendons result in the formation
Answer: 5- Thickening of the Achilles tendon is a of tendon xanthomata, with thickening of the
characteristic physical sign associated with FH Achilles tendon being an early classical
coetaneous manifestation of FH. Such soft tissue
manifestations combined with LDL-cholesterol
Familial hypercholesterolemia (FH) is levels above the 95th centile and a family history
characterized by raised serum LDL cholesterol of premature cardiovascular disease which
levels, which result in excess deposition of classically demonstrates intra-familial
cholesterol in tissues, leading to accelerated concordance of onset suggests a diagnosis of FH.
atherosclerosis and increased risk of premature Subsequent genetic testing for LDL-receptor and
coronary heart disease. In most populations Apo B mutations reveal mutations in between 50
studied, heterozygous FH occurs in about 1:500 and 80% of cases; those without a mutation often
people, but not all develop symptoms. have higher triglyceride levels and may in fact
Homozygous FH occurs in about 1:1,000,000. FH have other causes for their high cholesterol, such
results from defects in the hepatic uptake and as combined hyperlipidemia due to metabolic
degradation of LDL via the LDL-receptor pathway, syndrome.
commonly caused by a loss-of-function mutation
in the LDL-receptor gene (LDLR) on the short arm FH is usually treated with statins. Statins act by
of chromosome 19 or by a mutation in the gene inhibiting the enzyme hydroxymethylglutaryl CoA
encoding apolipoprotein B (APOB). FH is primarily reductase (HMG-CoA-reductase) in the liver. In
an autosomal dominant disorder with a gene- response, the liver produces more LDL receptors,
dosage effect. An autosomal recessive form of FH
which remove circulating LDL. Statins effectively is connected to the vena cava thus by-passing liver
lower cholesterol and LDL levels, although metabolism. Inhibition of the microsomal
sometimes add-on therapy with other drugs is triglyceride transfer protein, for example with the
required, such as bile acid sequestrants investigational drug AEGR-733, and infusion of
(cholestyramine or colestipol), nicotinic acid recombinant human apolipoprotein A1 are being
preparations or fibrates. Control of other risk explored as medical treatment options, with gene
factors for cardiovascular disease is required, as therapy also as a possible future alternative.
risk remains somewhat elevated even when
cholesterol levels are controlled. Professional Given that FH is present from birth and
guidelines recommend that the decision to treat atherosclerotic changes may begin early in life, it
an FH patient with statins should not be based on is sometimes necessary to treat adolescents or
the usual risk prediction tools (such as those even teenagers with agents that were originally
derived from the Framingham Heart Study), as developed for adults. Due to safety concerns,
they are likely to underestimate the risk of many doctors prefer to use bile acid sequestrants
cardiovascular disease; unlike the rest of the and fenofibrate as these are licensed in children.
population, FH have had high levels of cholesterol Nevertheless, statins seem safe and effective, and
since birth, probably increasing their relative risk. in older children may be used as in adults.
The addition of Ezetimibe may also be a
therapeutic option. Although controversy persists A multidisciplinary expert panel in 2006 advised
around the role of ezetemibe due to the absence on early combination therapy with LDL apheresis,
of a significant effect of combination ezetmibe / statins and cholesterol absorption inhibitors in
statin compared with statin monotherapy on children with homozygous FH at the highest risk.
carotid intima media thickness in patients with FH.
[ Q: 258 ] Question ID #41
There are no interventional studies that directly
show mortality benefit of cholesterol lowering in
FH patients. Rather, evidence of benefit is derived A 45 year old Asian man is reviewed by his GP as
from a number of trials conducted in people who part of a cardiovascular risk factor screening
have polygenic hypercholesterolemia (in which programme. He is a non-smoker with the only
heredity plays a smaller role). Still, an family history of note being his father who has
observational study of a large British registry been diagnosed with type 2 diabetes at the age of
showed that mortality in FH patients had started 68 following admission to hospital with an episode
to improve in the early 1990s, when statins were troponin negative chest pain. On examination his
introduced. blood pressure is 165 / 75 mmHg, his BMI is 28 kg
/ m2 and his waist circumference is 89 cm
Homozygous FH is harder to treat. The LDL
receptors are minimally functional, if at all. Only Investigations reveal:
high doses of statins, often in combination with
other medications, are modestly effective in 5.5 mmol/l (3.5 -
Fasting Plasma Glucose
improving lipid levels. If medical therapy is not 5.6)
successful at reducing cholesterol levels, LDL Fasting Total Cholesterol 5.6 mmol/l (<5)
apheresis may be used; this filters LDL from the
plasma in a process reminiscent of dialysis. Very Fasting Plasma 1.7 mmol/l (0.9 -
severe cases may be considered for a liver Triglyceride 2.2)
transplant leading to rapid improvement in 0.8 mmol/l (0.9 -
HDL-Cholesterol
cholesterol levels, but at the risk of complications 1.8)
from any solid organ transplant. Other surgical LDL-Cholesterol 3.8 mol/l (< 3)
techniques include partial ileal bypass surgery, in
which part of the small bowel is bypassed to 2.1 mu/l (0.32 -
TSH
decrease the absorption of cholesterol, and 5.5)
portacaval shunt surgery, in which the portal vein
How would you manage this gentleman’s lipid 4- Fasting Total and LDL-Cholesterol levels
profile?
5- Fasting LDL and HDL-cholesterol levels
(Please select 1 option)
Answer & Comments On examination, his weight is 142 kg and his BMI
is 51 kg/m2. His blood pressure is 136/84 mmHg
Answer: 2- Continue Orlistat at current dose and his pulse is 72 bpm.
This patient has lost 5.8 kg of weight over three Investigations reveal:
months equating to roughly a 5% weight loss
which is the figure recommended by NICE U+Es Normal
supporting continued treatment. The maximal
Fasting Glucose 8.2 mmol/l
Orlistat dose is 120mg tds and there is no value in
increasing this. NICE do not recommend the
combination of sibutramine and Orlistat as there Which of the following would be the most
is no evidence to support the combined appropriate approach to his weight management?
treatment. (Please select 1 option)
1- Combined Noradrenergic/Serotinergic
Answer & Comments
uptake inhibitor
2- Endocannabinoid receptor antagonist Answer: 2- Recommend for bariatric surgery
any treatment. Her weight is 86kg and she has a Answer & Comments
BMI of 27.8 kg/m2. Her pulse is 72 bpm and her
blood pressure is 126/78 mmHg. She is keen to Answer: 2- Autosomal recessive
receive some kind of intervention to help with her
weight loss. Laurence-Moon-Biedl syndrome is an autosomal
recessive disorder that is typified by severe
Which of the following would you recommend? obesity, hypogonadism, hyperphagia and optic
(Please select 1 option) atrophy.
1- Bariatric surgery
[ Q: 265 ] Question ID #52
2- Continue with education, diet and exercise
3- Fluoxetine A 51 year old gentleman is referred by his GP for
review of his glucose control and insulin dosage.
4- Orlistat He has a 7 year history of type 2 diabetes. He has
5- Sibutramine a previous history of ischemic heart disease having
been commenced on insulin following his
myocardial infarction 3 years previously.
Answer & Comments
Answer: 2- Continue with education, diet and On examination, body weight is 144 kg, BMI 51 Kg
exercise / m2 pulse 67 bpm, blood pressure 160/88 mmHg.
He has already tried an exercise regime and other
weight loss measures recommended by his doctor
This young female has a BMI indicating that she is with little effect. His current mediation includes
overweight. The NICE criteria recommends metformin 1 gram twice daily, simvastatin 40 mg
pharmacological intervention in patients with a daily, Novomix 30/70 mixed insulin 114 units in
BMI above 27 who have other co-morbidities the morning, 120 units evening time, ramipril 10
including diabetes, hypertension etc. The mg daily, bisoprolol 5 mg daily and Aspirin 75 mg
presence of PCOs does not fulfill this criteria and daily having been previously intolerant of orlistat.
the patient should be counselled and advised on
adhering to a hypocaloric diet which should be Investigations reveal
complemented by an appropriate exercise regime.
Hba1c 9.8% (<6)
[ Q: 264 ] Question ID #48
Total Cholesterol 4.1 mmol/l (<5)
Plasma triglyceride 2.5 mmol/l (0.9 - 2.2)
A 16 year old male with severe obesity is
diagnosed with Laurence-Moon-Biedl syndrome. TSH 2.9 mu l (0.32 - 5.5)
58 ml / min / 1.73 m2 (90
Which of the following is the mode of inheritance eGFR
- 110)
of this condition? albumin creatinine
5.5 mg / mmol (<2.5)
(Please select 1 option) Ratio
1- Autosomal dominant
Which of the following is the most appropriate
2- Autosomal recessive therapy option for this patient?
3- Mitochondrial (Please select 1 option)
From which of the following is Glucagon-like An 18 year old female who presents with
Peptide 1 (GLP-1) produced? oligomenorrhoea is found to have non-classical
(Please select 1 option) congenital adrenal hyperplasia. She enquires as to
the risk of any of her future children suffering the
1- Body of stomach disorder.
2- Ileum
Which of the following is correct concerning this
3- Liver risk?
4- Pancreas (Please select 1 option)
5- Pylorus 1- Virtually 0%
2- 1%
Answer & Comments
3- 5%
Answer: 2- Ileum
4- 25%
Which of the following is the total daily energy ESR 10 mm/hr (0 - 15)
expenditure for an average male?
(Please select 1 option) Which of the following is the most likely diagnosis?
1- 1000 kcal (Please select 1 option)
FBC Normal
A 44 year old male presents with a three month
Sodium 144 mmol/l (134 - 144) history of profuse watery diarrhoea. No specific
Potassium 3.9 mmol/l (3.5 - 5) abnormalities are noted on examination and
colonoscopy and stool culture are negative.
urea 10.2 mmol/l (3 - 8)
Investigations reveal the following results:
140 micromol/l (50 -
Creatinine
100) Polypeptide P 210 pmol/l (<300)
Chloride 122 mmol/l (95 - 105) VIP 350 pmol/l (<30)
Fasting Plasma Gastrin 33 pmol/l (<40)
7.8 mmol/l (3.5 - 5.6)
Glucose
Glucagon 25 pmol/l (<50)
pH 7.12 (7.35 - 7.45)
Standard bicarbonate 11 mmol/l (22 - 28) Which of the following is the most appropriate
pCO2 4.2 kPa (4.5 - 6) investigation?
pO2 14.5 kPa (10 - 13) (Please select 1 option)
3- Methanol consumption
Answer & Comments
4- Olanzapine induced acidosis
Answer: 2- MRI abdomen
5- Topiramate induced acidosis
This patient appears to have a VIPoma – Werner-
Answer & Comments Morrison syndrome. This is due to a pancreatic
neuroendocrine tumour in over 90% of cases and
Answer: 5- Topiramate induced acidosis
is associated with profuse watery diarrhoea and
fasting VIP concentrations typically 10 times
The patient has a hyperchloraemic acidosis with a elevated. Imaging the pancreas would be the most
normal anion gap {sodium – appropriate step here with and would be expected
(bicarboinate+chloride) = 11 – NR 8-16}. In this to localize the tumour effectively as they are
case the normal anion gap would exclude all the typically larger than 2cm. Surgical resection is the
typical causes as found with high anion gap – lactic treatment of choice but somatostatin analogues
acidosis, DKA, methanol etc (A useful mnemonic would be used to control symptoms prior to
for high anion gap metabolic acidosis is surgery.
MUDPILES). Loss of bicarbonate ions is often a
cause of normal anion gap acidosis – renal tubular
acidosis. In our case, Topiramate causes a type 2
RTA and loss of bicarbonate ions. Acetazolamide,
Which of the following is the likely explanation? The typical rash associated with glucagonoma is a
(Please select 1 option) necrolytic migratory erythema which is often
located in the perineal region. The underlying
1- Craniopharyngioma histology is inflammation and separation of the
2- Drug induced superficial layers of the epidermis.
The most likely explanation is pancreatic exocrine A 55 year old male attends for an insurance
insufficiency, a side effect of the octreotide medical. He is generally fit and has no medical
therapy and caused by inhibition of pancreatic history of note. He is a non smoker and drinks
enzyme secretion. This is well described and may approximately 10 units of alcohol each week. His
be treated with pancreatic enzyme family history includes his father having died at the
supplementation such as Creon. age of 54 of a heart attack.
You are asked to review a patient on the Cholesterol 5.8 mmol/l (<5)
neurosurgical ward. He has had an aneurysm
clipped following subarachnoid haemorrhage but Triglyceride 1.5 mmol/l (0.8- 2.0)
the neurosurgeons are concerned that his sodium HDL 1.0 mmol/l (0.9 - 1.9)
concentration has fallen. Currently his sodium Plasma Glucose 5.4 mmol/l (3.5 - 5.6)
concentration is 118 mmol/l. Further
investigations reveal: He is advised a low cholesterol diet only. By how
much would this be expected to lower his
Urea 4 mmol/l (3-8) cholesterol concentration?
Potassium 4.4 mmol/l (3.5-5) (Please select 1 option)
Plasma Osmolality 254 mosmol/l (275-295) 1- 10%
Urine Sodium 50 mmol/l
2- 20%
Urine Osmolality 440 mosmol/l
3- 30%
Which of the following is the most likely 4- 40%
explanation of this patient’s hyponatraemia?
5- 50%
(Please select 1 option)
Reference:
British Heart Society - Cardiovascular Risk
Prediction Chart
Investigations reveal:
Answer: 4- His Waist circumference, blood LDL Cholesterol 3.2 mmol/l (< 3)
pressure and HDL concentration
Which of the following criteria would suggest a
diagnosis of metabolic syndrome in this gentleman
This gentleman meets the criteria for the according to the International Diabetes
metabolic syndrome, as defined by the
Federation?
International Diabetes Federation, since he has
central obesity (waist circumference ≥94 cm), a (Please select 1 option)
plasma triglyceride ≥1.7 mmol/l, blood pressure 1- BMI, LDL-C concentration and blood
≥130 / 85 mm Hg and a HDL-cholesterol < 1.04 pressure
mmol/l.
2- BMI, fasting plasma glucose and plasma
The IDF definition of the metabolic syndrome is triglyceride concentration
based on the presence of central obesity (waist
3- Waist circumference, HDL-cholesterol
circumference in Europeans is ≥94 cm in men and
concentration and blood pressure
≥80 cm in women) combined with any two of the
following: 4- Waist circumference, triglyceride
concentration and LDL-cholesterol
Plasma concentration
≥1.7 mmol/l
Triglyceride
5- Waist circumference, HDL-cholesterol and
HDL LDL-cholesterol concentration
< 1.04 ( Men)
Cholesterol
HDL
< 1.29 (Women) Answer & Comments
Cholesterol
Fasting Answer: 3- Waist circumference, HDL-cholesterol
Plasma ≥ 5.5 mmol/l concentration and blood pressure
Glucose
Blood ≥ 130/85 mm Hg or recieving This gentleman meets the criteria for the
Pressure Anti-Hypertensive Treatment metabolic syndrome, as defined by the
International Diabetes Federation, since he has
central obesity (waist circumference ≥94 cm), a Which of the following treatments is associated
plasma triglyceride ≥1.7 mmol/l, blood pressure with the greatest reduction in future
≥130 / 85 mm Hg and a HDL-cholesterol < 1.04 morbidity/mortality for this man?
mmol/l.
(Please select 1 option)
Non-HDL cholesterol includes VLDL, IDL, LPa apart Analysis of a 24hour urine collection reveals:
from LDL Cholesterol.
Sodium 140 mmol/L (135-145)
[ Q: 284 ] Question ID #340 Potassium 2.9 mmol/L (3.5-5)
Urea 5.0 mmol/L (3-8)
A 63 year old female is admitted with a two day Bicarbonate 36 mmol/L (20-28)
history of being unwell and deteriorating
confusion. Arterial blood gas analysis reveals: Random Glucose 4.5 mmol/L (3.0-6.0)
Potassium 266 mmol/24h (25-100)
pH 7.65 (7.36-7.44) Sodium 92 mmol/24h (100-250)
pO2 8 kPa (11.3-12.6) Serum Magnesium 0.8 mmol/L (0.7-1.2)
pCO2 4.5 kPa (4.7-6.0)
Which of the following is the likely diagnosis?
Standard Bicarbonate 35 mmol/L (20-28)
(Please select 1 option)
Which of the following is the correct interpretation 1- Bartter syndrome
of these blood gas results?
2- Conn's syndrome
(Please select 1 option)
The patient has a hypokalaemic metabolic What is the most appropriate next step in his
alkalosis with urinary potassium wasting, a normal management?
blood pressure and in the context of poor (Please select 1 option)
academic performance is likely to have Bartter
1- Fluid restriction of 1.5 l/day
syndrome. The normal blood pressure makes
Conn's, fibromuscular dysplasia and Liddle's 2- Hypertonic normal saline 1 litre over 6 hours
unlikely. Bartter's syndrome usually presents in and repeat electrolytes
childhood with polyuria, nocturnal enuresis and
growth retardation. Bartter's syndrome is 3- Normal saline 1 litre over 6 hours and repeat
associated with hyperplasia of the juxtaglomerular electrolytes
apparatus. 4- Start demeclocycline
5- Start Tolveptan
[ Q: 286 ] Question ID #486
A 24 year old who was involved in a car crash 3 Answer & Comments
days ago, is presently being nursed in intensive Answer: 3- Normal saline 1 litre over 6 hours and
care unit. His general physical and systemic repeat electrolytes
examination is unremarkable. The recent
biochemistry results are as shown below:
This 24 year old appears slightly hypovolaemic
(with raised urea and creatinine but has normal
119
Sodium (135-145) physical examination) and has hyponatraemia in
mmol/L
presence of normal thyroid and adrenal function
3.8 (TFT/short synacthen test normal). The serum
Potassium (3.5-5.5)
mmol/L osmolality is low with low levels of sodium
18.3 excretion from the urine (in contrast to SIADH
Urea (3-8) which is associated with low serum osmolality
mmol/L
along with excessive urinary sodium loss- >20
115
Creatinine (60-115) mmol/L). As a result patient may benefit from fluid
µmol/L
replacement and rechecking of his electrolyte
2.65 levels.
Calcium (2.2-2.6 mmol/L)
mmol/L
TSH 2.24 mU/L (0.35-5.5 mU/L) [ Q: 287 ] Question ID #487
16.9 (11.5-22.7
Free T4
pmol/L pmol/L) According to NICE guidelines, which of the
Prolactin 480 mU/L following oral hypoglycaemic agent is safe to use
6.3 in women with gestational diabetes in pregnancy?
Glucose
mmol/L (Please select 1 option)
5- Turner syndrome
[ Q: 290 ] Question ID #630
1- Copper 2- CT abdomen
2- Vitamin A 3- CT angiography
Vitamin A and D are fat soluble vitamins which Hypoglycaemia in non-diabetics is traditionally
may be deficient in patients who undergo bariatric classified as post-absorbative (fasting)
surgery and develop malabsorption during post hypoglycaemia or post-prandial (reactive)
operative phase. The shortening of length of
hypoglycaemia. However, whether a patient
bowel and dumping syndrome contribute to appears to be ill or well, is now regarded as a
process of malabsorption. better factor in assessing the cause of
hypoglycaemia. In a well appearing individual,
Vitamin A deficiency is associated with night causes of hypoglycaemia include:
blindness, xerophthalmia and occasionally
complete blindness (Vitamin A deficiency being Drugs - insulin or insulin secretagogue;
one of the leading preventable causes of blindness alcohol; other drugs
in developing countries!). Night blindness usually Accidental, surreptitious, or malicious
manifests as inability in adjustment to dimmed hypoglycaemia
light and is an early feature of Vitamin A Endogenous hyperinsulinism - insulinoma;
deficiency. The production of eye pigment - functional beta cell disorders
Rhodopsin (found in retina), which is responsible nesidioblastosis); noninsulinoma
for sensing low light situations is impaired in pancreatogenous hypoglycaemia;
Vitamin A deficiency. postgastric bypass hypoglycaemia; insulin
autoimmune hypoglycaemia; anti-insulin
[ Q: 294 ] Question ID #660 antibody; anti-insulin receptor antibody
Idiopathic postprandial hypoglycaemia
A 44 year old woman is admitted for further
investigations of her hypoglycaemic episodes. Her In this non-diabetic patient with hypoglycaemia,
72 hour fasting blood results are shown below: her fasting blood results reveal a raised insulin
level with a normal c-peptide level, which is
suggestive of an exogenous insulin source.
(3.5-
Blood sugar 2.2 mmol/L
5.5)
The biochemical pattern for different causes of
Insulin 155 µU/L (<10) hypoglycaemia in non-diabetics are as follows:
C Peptide 10.2 pmol/L (<500)
4- Lipoprotein lipase deficiency identified. She has been advised that on this basis,
‘cascade testing’ of her family should be
5- Polygenic Hypercholesterolaemia
undertaken.
Answer & Comments She seeks your advice about cascade testing and
what should be done now:
Answer: 2- Familial Combined Hyperlipidaemia
(Please select 1 option)
Familial combined hyperlipidaemia (FCH) is a 1- Cascade testing using DNA is not appropriate
commonly encountered hyperlipidaemia and is as LDL receptor mutations are very rare and
associated with premature cardiovascular disease. full gene sequencing of the son is required
Unlike familial hypercholesterolaemia, the
genetics of the condition are not clear and the 2- Offer a cholesterol test to her son and her
presentation much more variable. Whilst obesity other family members to assess if
and insulin resistance are more commonly hypercholesterolaemia is present in them
associated with FCH, this is not invariably the case. before offering a DNA test
The commonest laboratory findings are raised
3- Offer a DNA test to adult family members
cholesterol and triglycerides (i.e. combined),
and a cholesterol test in family members
together with a raised concentration of
apolipoprotein B. This is the surface protein aged between 8 and 16 years old
present on both very low density lipoproteins 4- Offer a DNA test to adult family members
(VLDL) as well as low density lipoprotein (LDL) and only to assess if the identified mutation is
is a good marker of atherosclerotic risk. Both present in them before offering a
familial and polygenic hypercholesterolaemia are
cholesterol test
less commonly associated with
hypertriglyceridaemia in the absence of diabetes 5- Offer a DNA test to her son and her other
mellitus, obesity or alcohol excess. Anabolic family members to assess if the identified
steroid misuse usually causes low HDL cholesterol. mutation is present in them before offering
Lipoprotein lipase deficiency is very rare, is usually a cholesterol test
associated with a more severe dyslipidaemia and
is inherited in an autosomal recessive manner,
which is not consistent with his family history. Answer & Comments
Answer: 5- Offer a DNA test to her son and her
Reference:
other family members to assess if the
Heart UK Familial Combined Hyperlipidaemia
identified mutation is present in them before
(FCH)
offering a cholesterol test
Larry A. Weinrauch, LA (MD) MedlinePlus
Medical Encylcopedia (2014) Familial
combined hyperlipidemia Individuals who have a high clinical probability of
having FH should be offered genotyping. This can
confirm the diagnosis, but importantly can
[ Q: 300 ] Question ID #3351 facilitate cascade testing. Once the DNA mutation
has been identified in the presenting patient
A 15 year old boy attends clinic with his mother. (known as the ‘index’ patient), testing for the
The mother has just been diagnosed with familial presence or absence of this specific mutation can
hypercholesterolaemia (FH). She has a strong be undertaken in family members. This is carried
family history of premature cardiovascular disease out in a step wise fashion, such that first degree
and was found to have significant relatives i.e. parents, children and siblings are
hypercholesterolaemia. She recently had a genetic tested first and subsequent relatives thereafter.
test carried out and has been advised that a For example if the index patient’s brother does not
mutation in the LDL receptor causing FH has been have the mutation, testing in his children is not
required. If the index patient’s sister does have the The following clinical findings are relevant for the
mutation, her children will be offered testing. diagnosis of familial hypercholesterolaemia:
(Please select 1 option)
Offering DNA testing, rather than initial
cholesterol testing in family members is 1- Premature corneal arcus is pathognomonic
recommended, as it provides a definitive answer, of FH and confirms the diagnosis
particularly in younger people when the
2- Premature marked xanthelasma are
cholesterol results may be harder to interpret.
pathognomonic of FH and confirm the
Cholesterol testing should be carried out in family
members who have the family mutation, but in diagnosis
those who do not; this may not be required, 3- Tendon xanthomata are pathognomonic of
particularly for children. DNA testing should be FH and confirm the diagnosis
offered to children by the age of 10. LDL receptor
mutations are the commonest cause of FH. Less 4- The clinical findings are not relevant as the
commonly mutations are found in the apo B or LDL is >7.9 mmol/L, which confirms the
PCSK9 genes. diagnosis
5- The diagnosis may only be confirmed with a
Reference:
Heart UK Children and familial DNA test
hypercholesterolaemia
Answer & Comments
[ Q: 301 ] Question ID #3352 Answer: 3- Tendon xanthomata are
pathognomonic of FH and confirm the
A 39 year old man previously fit and well is diagnosis
admitted to hospital with an acute myocardial
infarction (MI). He is successfully treated with
Tendon xanthomata are pathognomonic of FH and
percutaneous coronary intervention. The
confirm the diagnosis. These may manifest as
admitting doctor notes that his father sustained a
thickening and/or hardening of tendons, usually
fatal MI at the age of 52 and his paternal
associated with distinct nodularity. They are
grandfather sustained a fatal MI at the age of 48.
commonly found in the Achilles’ tendons and
On examination he is found to have xanthomata in
sometimes in other extensor tendons, such as
both Achilles’ tendons, bilateral corneal arcus and
those of the hands. The presence of xanthelasma
marked xanthelasma. His lipid profile is measured
and premature arcus are suggestive, but are not
on admission:
specific for FH. Xanthelasma may be found in the
absence of hypercholesterolaemia, but its finding
Reference should prompt measurement of a lipid profile.
Results
interval Corneal arcus becomes more common with
Total 11.2 advancing age, but it’s presence in a person under
cholesterol mmol/L the age of 40 is suggestive of a lipid disorder. The
HDL 1.3 probability of FH increases with higher LDL
>1.0 cholesterols, but there is no definitive diagnostic
cholesterol mmol/L
cut-off above which the diagnosis is confirmed. A
1.5 DNA test may confirm the diagnosis, but the
Triglycerides <2.0
mmol/L patient may be described as having definite FH in
9.2 the presence of the above profile with xanthoma.
Calculated LDL
mmol/L
5.2 Reference:
Fasting glucose 3.0-6.0 NICE guidelines CG71 (2008) Identification and
mmol/L
management of familial
Thyroid profile normal hypercholesterolaemia
Which of the following is the most likely diagnosis? A 37 year old man attends for evaluation due to
his family history of premature vascular disease
(Please select 1 option)
and is advised that he should have his lipid profile
1- Familial hypercholesterolaemia checked. He asks if fasting will make a difference
to the results of the test.
2- Hypercholesterolaemia secondary to diet
3- Hyperlipidaemia secondary to myocardial What is the relevance of fasting status in the
ischaemia interpretation of a lipid profile?
4- Hypothyroidism (Please select 1 option)
calculated LDL value can still be helpful for the 3- Hyperlipidaemia secondary to skeletal
assessment of familial hypercholesterolaemia. muscle damage
Larry A. Weinrauch, LA (MD) MedlinePlus Individuals who have a high clinical probability of
Medical Encylcopedia (2014) Familial having FH should be offered genotyping. This can
combined hyperlipidemia confirm the diagnosis, but importantly can
facilitate cascade testing. Once the DNA mutation
[ Q: 310 ] Question ID #3361 has been identified in the presenting patient
(known as the ‘index’ patient), testing for the
presence or absence of this specific mutation can
A 15 year old boy attends clinic with his be undertaken in family members. This is carried
mother. The mother has just been diagnosed out in a step wise fashion, such that first degree
with familial hypercholesterolaemia (FH). She relatives i.e. parents, children and siblings are
has a strong family history of premature tested first and subsequent relatives thereafter.
cardiovascular disease and was found to have For example if the index patient’s brother does not
significant hypercholesterolaemia. She has have the mutation, testing in his children is not
recently had a genetic test carried out and has required. If the index patient’s sister does have the
been advised that a mutation in the LDL mutation, her children will be offered testing.
receptor causing FH has been identified. She
has been advised that on this basis, ‘cascade Offering DNA testing, rather than initial
testing’ of her family should be undertaken. cholesterol testing in family members is
recommended, as it provides a definitive answer,
She seeks your advice about cascade testing and particularly in younger people when the
what should be done now: cholesterol results may be harder to interpret.
Cholesterol testing should be carried out in family
(Please select 1 option)
members who have the family mutation, but in
1- Cascade testing using DNA is not appropriate those who do not; this may not be required,
as LDL receptor mutations are very rare and particularly for children. DNA testing should be
full gene sequencing of the son is required offered to children by the age of 10. LDL receptor
mutations are the commonest cause of FH. Less
2- Offer a cholesterol test to her son and her commonly mutations are found in the apo B or
other family members to assess if PCSK9 genes.
hypercholesterolaemia is present in them
before offering a DNA test Reference:
Heart UK Children and familial
3- Offer a DNA test to adult family members
hypercholesterolaemia
and a cholesterol test in family members
aged between 8 and 16 years old
[ Q: 311 ] Question ID #3362
4- Offer a DNA test to adult family members
only to assess if the identified mutation is A 39 year old man previously fit and well is
present in them before offering a admitted to hospital with an acute myocardial
cholesterol test infarction (MI). He is successfully treated with
5- Offer a DNA test to her son and her other percutaneous coronary intervention. The
admitting doctor notes that his father sustained a
family members to assess if the identified
fatal MI at the age of 52 and his paternal
mutation is present in them before offering
grandfather sustained a fatal MI at the age of 48.
a cholesterol test On examination he is found to have xanthomata in
both Achilles’ tendons, bilateral corneal arcus and
Answer & Comments marked xanthelasma. His lipid profile is measured
on admission:
Answer: 5- Offer a DNA test to her son and her
other family members to assess if the
Reference
identified mutation is present in them before Results
interval
offering a cholesterol test
2- He and the mother should be reassured that which he takes omeprazole. Physical examination
the lipids are unremarkable is normal with a blood pressure of 130/78 mmHg.
You organise investigations with the following
3- Offer annual testing of the lipid profile results:
through puberty
4- The lipid profile should be re-checked when Reference
Results
he is an adult interval
likely. Total cholesterol is often lower following a LDL cholesterol= total cholesterol- HDL
myocardial infarction. cholesterol- (triglycerides/2.2) Triglycerides can
increase in the non-fasting state and therefore
Reference: also affect calculated LDL cholesterol. Current
NICE CG71 (2008) Identification and NICE guidance recommends using non-fasting
management of familial parameters such as non-HDL cholesterol (i.e. total
hypercholesterolaemia cholesterol -HDL cholesterol) as these parameters
Heart UK (2015) What is Familial are less affected by fasting status. Using a
Hypercholesterolaemia (FH)? calculated LDL value can still be helpful for the
assessment of familial hypercholesterolaemia.
FH Wales- information from a service which
offers comprehensive clinical and family
The above equation is not applicable when the
testing triglycerides are >4.4 mmol/L
AS04 : H·E·A·R·T UK Advice sheetDiagnosing FH-
Simon Broome Criteria Reference:
Joint British Society (2014) guidelines on
[ Q: 314 ] Question ID #3365 prevention of cardiovascular disease 3
NICE guideline CG181 (2014) Lipid modification:
A 37 year old man attends for evaluation due to cardiovascular risk assessment and the
his family history of premature vascular disease modification of blood lipids for the primary
and is advised that he should have his lipid profile and secondary prevention of cardiovascular
checked. He asks if fasting will make a difference disease
to the results of the test.
[ Q: 315 ] Question ID #3366
What is the relevance of fasting status in the
interpretation of a lipid profile? A 42 year old lady attends clinic complaining of
(Please select 1 option) persistent lethargy. She works as a teacher and is
generally well. She has no relevant medical history
1- All the parameters within a lipid profile are
and takes no regular medication. Her mother
significantly affected by fasting status
developed angina at the age of 75 and her father
2- Fasting is only relevant for the assessment of has no medical problems at the age of 78. On
an apolipoprotein profile examination her pulse is 56/min, blood pressure is
122/72 mmHg, BMI of 26 kg/m2 with no other
3- The equation used to calculate triglycerides obvious findings. Investigations reveal:
is not applicable if the patient is not fasting
4- Total cholesterol and HDL cholesterol only Reference
Results
are affected by fasting status interval
Total 11.0
5- Triglycerides and calculated LDL cholesterol
cholesterol mmol/L
only may be affected by fasting
HDL 1.2
>1.2
cholesterol mmol/L
Answer & Comments
1.1
Triglycerides <2.0
Answer: 5- Triglycerides and calculated LDL mmol/L
cholesterol only may be affected by fasting 9.3
Calculated LDL
mmol/L
Most laboratories measure total cholesterol, HDL 4.8
cholesterol and triglycerides and calculate LDL Fasting glucose 3.0-6.0
mmol/L
cholesterol by using the following formula (all
Creatine kinase 1350 u/L 25-200
parameters in mmol/L):
Which of the following is the most likely diagnosis? pressure is 110/68 mmHg. At 7 weeks gestation
(Please select 1 option) her weight was 54 kg, BMI 22 kg/m2.
1- Familial hypercholesterolaemia
Reference
Results
2- Hypercholesterolaemia secondary to diet interval
[ Q: 317 ] Question ID #3368 Lipid lowering with a statin is recommended for all
patients who have established cardiovascular
A 64 year old man is reviewed 3 months following disease (secondary prevention). In an individual
admission with a myocardial infarction. He who has never experienced side-effects, he should
required coronary artery bypass surgery but has be reassured that statins are safe and effective at
made a good recovery and is asymptomatic. He is reducing the risk of recurrent cardiovascular
a life long non-smoker, has had treated events. Risk factor calculators are not appropriate
hypertension for 10 years and plans to return back for use in secondary prevention. Dietary advice is
an important adjunct to drug therapy in this
to work on a part-time basis. His lipid profile prior
to his MI reveals: situation, but not as the only intervention. Drug
therapies, other than statins should not be offered
at this stage as monotherapy.
Reference
Results
interval
Reference:
Total 5.7
Hypercholesterolaemia- secondary prevention
cholesterol mmol/L
HDL 1.0 NICE CG 181
>1.0
cholesterol mmol/L
0.8 [ Q: 318 ] Question ID #3369
Triglycerides 0.1-2.0
mmol/L
4.3
Calculated LDL A 52 year old man attends for a cardiovascular risk
mmol/L
assessment. He is not concerned about his health,
4.8
Fasting glucose 3.5-5.5 but his wife has mentioned that he has gradually
mmol/L
put on weight over the last few years and should
have a check up. He is generally well with no family
He has never taken a statin because of concerns history of premature vascular disease. He carries
about side-effects and has not started the out little physical activity and is a non-smoker. The
atorvastatin recommended by his cardiac following are results from your assessment.
surgeon.
Results Reference interval
Which of the following actions is appropriate?
Blood pressure 160/95
(Please select 1 option)
Weight 105 kg
1- Advise that dietary measures alone are
appropriate and refer to a dietitian 33
BMI
kg/m2
2- Advise that statin therapy is safe in nearly all
Total
people and is recommended for all patients 7.4
cholesterol
following CABG
HDL cholesterol 0.8 >1.0
3- Recommend a bile acid sequestrant Triglycerides 2.4 0.1-2.0
4- Recommend starting a fibrate Calculated LDL 5.5
5- Recommend starting ezetimibe Fasting glucose 5.4 3.5-5.5
HbA1c 36 20-42
Answer & Comments
His calculated 10 year cardiovascular risk (QRISK 2)
Answer: 2- Advise that statin therapy is safe in
is 34%. He is started on atorvastatin 20mg once
nearly all people and is recommended for all daily, which is gradually up titrated to 80 mg once
patients following CABG daily, and ramipril 5 mg once daily. Twelve months
later his parameters are as follows:
Answer: 5- Reinforce diet and lifestyle advice 1- Advise that the diagnosis of familial
hypercholesterolaemia can only be made
This gentleman is offered treatment in the context with a DNA test
of primary prevention. Pharmacological 2- He and the mother should be reassured that
intervention is appropriate, but there are no the lipids are unremarkable
specific ‘lipid targets’. The current guidance for
lipid modification from NICE recommends offering 3- Offer annual testing of the lipid profile
intervention if CV risk is greater than 10% over 10 through puberty
years. His hypertriglyceridaemia and low HDL
4- The lipid profile should be re-checked when
cholesterol are related to obesity and lifestyle
he is an adult
intervention will provide the greatest opportunity
for health improvement. Clinical trials are 5- The results suggest familial
underway with specific HDL raising drugs to hypercholesterolaemia and a specialist
ascertain their safety and potential utility. referral is appropriate
Reference:
Joint British Society guidelines on prevention of Answer & Comments
cardiovascular disease 3 (2014) Answer: 5- The results suggest familial
NICE CG 181 hypercholesterolaemia and a specialist
referral is appropriate
Cholesterol is lower in children than in adults and Which of the following actions would be the most
the results in the context of the history are appropriate?
suggestive of familial hypercholesterolaemia (FH). (Please select 1 option)
Assessment within a specialist paediatric setting is
appropriate. In FH, ideally the diagnosis should be 1- Introduce no new medication
made by testing for the known family DNA
2- Prescribe a fibrate
mutation, but if this is neither known nor
available, lipid testing is appropriate. Deferring the 3- Prescribe a statin
diagnosis with such a concerning family history is
4- Prescribe ezetimibe 10 mg once daily
undesirable. Statins may be used in children from
the age of 10. 5- Prescribe omega 3 fatty acids g once daily
Reference:
Answer & Comments
NICE guidelines for familial
hypercholesterolaemia Answer: 2- Prescribe a fibrate
HEART UK- the cholesterol charity- FH and
children Hypertriglyceridaemia can cause pancreatitis, and
FH Wales- website for the FH service in Wales, in this situation, drug therapy to reduce
which offers a comprehensive clinical and triglycerides is appropriate. The most effective
family testing service group of agents for this are the fibrates. Statins
and ezetimibe have a minimal effect on
hypertriglyceridaemia. High dose (4g/day) of
[ Q: 320 ] Question ID #3371
omega 3 fatty acids can be helpful, often in
conjunction with a fibrate; however at a low dose
A 33 year old woman is discharged from the the impact on triglyceride metabolism is minimal.
surgical ward following acute admission with She is likely to have an underlying defect in
abdominal pain and a diagnosis of acute triglyceride metabolism which will require further
pancreatitis, this settled with conservative assessment.
management. She drinks no alcohol, has no
relevant medical history and takes no medication. Reference:
Her hepatic ultrasound shows no evidence of Management of hypertriglyceridaemia
gallstones. On admission the surgical team had
checked a fasting lipid profile and ask you to follow
up on the results below: [ Q: 321 ] Question ID #3372
A 62 year old woman with acute symptomatic Her blood results reveal the following:
hyponatraemia likely due to SIADH and associated
with drowsiness is considered to be appropriate
for immediate treatment with hypertonic saline. Sodium 118 mmol/l (134-144)
Her serum sodium is 115 mmol/l. Potassium 3.8 mmol/l (3.5-5.5)
Urea 4.5 mmol/l (3-8)
What is the target sodium to which you would
Creatinine 80 micromol/l (50-100)
treat this patient over the next 1hr?
Glucose 4.8 mmol/l (3.5-5.5)
(Please select 1 option)
Reference: Reference:
Cerebral Salt Wasting Medscape Resuscitation Council Guidelines on Anaphylaxis
An 18 year old female is admitted as an emergency An 18 year old student is admitted after having
with acute facial swelling and difficulty breathing. had a strange reaction where she became acutely
She is not known to have any allergy but does unwell after going to a Chinese restaurant. She
suffer from mild asthma for which she occasionally became light headed and short of breath and she
uses a salbutamol inhaler. She was out having a thinks she momentarily passed out. An ambulance
picnic with her friends, eating various foods, when was called and she was given oxygen and
she suddenly became unwell, feeling particularly recovered. She has a history of eczema but
anxious and itchy, while developing facial/mouth nothing else. On examination, there is nothing to
swelling with difficulty breathing. Her friends find and she seems fine. Blood results are normal.
called 999.
Which of the following tests could be used to
On examination, she has swelling of the face assess if this were an anaphylactic reaction?
particularly around the eyes and lips. There are (Please select 1 option)
wheals around the neck. Pulse is 100 bpm and
blood pressure 128/80 mmHg. She has an audible 1- Alpha-1 antitrypsin
wheeze and oxygen saturations are 96% on air.
2- C1-esterase
Which of the following is the most appropriate 3- C4 complement
treatment for this patient? 4- C-reactive protein
(Please select 1 option)
5- Mast cell tryptase
1- 0.5 mg adrenaline IV
2- 0.5 mg adrenaline IM Answer & Comments
3- 10mg chlorpheniramine IM Answer: 5- Mast cell tryptase
4- 100mg hydrocortisone IV
Mast cell tryptase is the molecule to measure in
5- 40mg oral prednisolone
suspected anaphylaxis as the mast cells release
this in abundance in anaphylaxis with
Answer & Comments concentrations rising abruptly within 30 minutes
to 1 hour. C1 esterase deficiency is a cause of
Answer: 2- 0.5 mg adrenaline IM repeated episodes of anaphylaxis but this is not
the molecule to measure. As in this case, the
This patient has developed angioedema, urticarial measurement of mast cell tryptase may
and breathing difficulties typical of anaphylaxis. differentiate between a faint and anaphylaxis
The patient should be treated with 0.5 mg IM where doubt exists.
adrenaline as the initial treatment as per the
resuscitation council guidelines. Other treatments Reference:
include IM/IV hydrocortisone and IM Resuscitation Council Guidelines anaphylaxis
chlorpheniramine but the IM adrenaline and fluids
is the initial treatment. The use of IV adrenaline is [ Q: 333 ] Question ID #3775
reserved for severe cases in whom expert
understanding of IV adrenaline is understood.
A 72 year old man with a 12 year history of
diabetes being treated with metformin is seen as
Which of the following is the likely cause of this Which of the following tumour markers would be
man’s hyperkalaemia? most helpful in this patient?
(Please select 1 option) (Please select 1 option)
2- Bisoprolol 2- CA 125
This patient has a long history of hypertension and This patient presents with a classical history of
hyperkalaemia. He has a mild hyponatraemia flushes and diarrhoea together with metastatic
associated with the hyperkalaemia and the most deposits in the liver suggesting metastatic
likely diagnosis is type 4 renal tubular acidosis or neuroendocrine tumour (carcinoid syndrome).
hyporeninaemic hypoaldosteronism. The most Useful tumour markers in this condition include
likely drug to cause this is the beta blocker and this chromogranin A and B and urinary 5-HIAA with
should be stopped. Other drugs implicated include 85% sensitivity and 95% sensitivity.
ACE inhibitors and NSAIDs.
Chromogranin A: Is it a useful tumour marker in
Reference: NETs
Hyporeninemic hypoaldosteronism
[ Q: 335 ] Question ID #4134
[ Q: 338 ] Question ID #6
A 45 year old lady recently diagnosed with type 2 Which of the following is the most appropriate
diabetes is managed by diet and exercise alone initial advice for this lady with respect to adjusting
and has an HbA1c of 6.5% (<6). Her BMI is 32 kg / her prandial insulin dose?
m 2 and her blood pressure is 144 / 74 mmHg. (Please select 1 option)
Answer: 3- 1 in 10
The DAFNE patient education program suggests
that patients should initially adjust prandial insulin
The incident diabetes risk in siblings and offspring doses by 1 unit according to each 10 gram of
of patients with type 2 diabetes is approximately dietary carbohydrate. This initial ratio may
10%. The background population risk for type 2 subsequently be modified according to response
diabetes is around 2-5%, while the risk of incident and the influence of other factors such as exercise.
type 2 diabetes in a monozygotic twin of a patient
with type 2 diabetes is around 1 in 3.
[ Q: 339 ] Question ID #7
[ Q: 341 ] Question ID #9 The most likely diagnosis for this patient is MODY,
suggested by the presence of a first degree
A 35 year old school teacher is referred to clinic for relative presenting with diabetes < 25 years
assessment of and management of his diabetes. coupled with the fact that this patient has no
He was diagnosed with diabetes 3 years previously evidence of ketosis and has corrected fasting
on the basis of elevated fasting blood glucose blood glucose for at least 2 years without insulin.
levels and has subsequently experienced good The diagnosis of MODY in this case is also
glycaemic control on oral hypoglycaemic therapy. supported by the absence of features of the
metabolic syndrome, obesity and negative GAD
He denies any weight loss, has never experienced
ketosis and has an elder sister who was diagnosed serology.
with diabetes 12 years previously at the age of 24.
On examination his blood pressure is 135/75 [ Q: 342 ] Question ID #12
mmHg, his body weight is 75 kg and his BMI is 25.5
kg / m2. His current medication includes metfomin A 51 year old man with recently identified type 2
1 gram twice daily and gliclazide 80 mg twice daily. diabetes currently managed with lifestyle
modification alone undergoes a mixed meal
Investigations reveal: tolerance test with measurement of glucose,
insulin and glucagon profiles over a 4 hour period.
HbAlc 7.5% (<6)
Fasting Plasma Which of the following glucose, insulin and
6.9 mmol/l (<6) glucagon profiles would you expect to occur in this
Glucose
Total cholesterol 4.5 mmol/l (<4) gentleman compared to an age matched healthy
volunteer following a mixed meal tolerance test?
TSH 0.1 mu/l (032 - 5.5)
(Please select 1 option)
TSH receptor
Negative 1- Increased plasma glucose, reduced early
antibodies
phase insulin secretion, reduced glucagon
96 ml/min/1.73 m2 (90 -
eGFR secretion
110)
GAD antibodies Negative 2- Increased plasma glucose, reduced early
phase insulin secretion, elevated glucagon
Which of the following is the most likely correct secretion
diagnosis for this young man?
3- Increased plasma glucose, exaggerated early
(Please select 1 option) phase insulin secretion, reduced glucagon
1- Latent Autoimmune Diabetes in Adults secretion
(LADA) 4- Increased plasma glucose, delayed early
2- Maturity onset diabetes of the young phase insulin secretion, no difference in
(MODY) glucagon secretion
Beta and alpha cell dysfunction are characteristic together with insulin in a 1:1 molar ratio. Beta cells
features of the pathophysiology of type 2 account for between 70 – 80% of the endocrine
diabetes. Beta cell function is reduced by up to cell mass of the pancreas and contain
70% according to data from UKPDS at the point of chromogranin A which is an acidic 48KDa protein
type 2 diabetes diagnosis. The earliest which has a putative function as a pro-hormone.
manifestation of beta cell dysfunction occurs in Delayed, but exaggerated early phase insulin
the form of reduced and delayed post-prandial secretion is a feature of early glucose intolerance,
early phase insulin secretion. Alpha cell as glucose intolerance progresses to type 2
dysfunction is manifest as elevated plasma diabetes, with associated progressive beta cell
glucagon levels compared to healthy subjects dysfunction, early phase postprandial insulin
which remain inappropriately elevated following secretion becomes both delayed and blunted.
meal ingestion.
[ Q: 344 ] Question ID #17
[ Q: 343 ] Question ID #15
A 59 year old lady with a 4 year history of type 2
A 45 year old man with a 6 month history of diet diabetes is reviewed in clinic. On examination her
controlled type 2 diabetes undergoes a 2,500 kcal BMI is 33 kg / m2 and her blood pressure is 145 /
4 hour mixed meal tolerance test as part of a 65 mmHg. She is generally well but is concerned
research program. Peak insulin concentrations are about the possibility of cardiovascular disease
measured after 2 hours with a level of 48 pmol/l since her father suffered with a myocardial
being achieved with a peak glucose concentration infarction at the age of 61 years and she has
of 14. 8 mmol/l being achieved after 90 minutes recently read an article outlining the link between
diabetes and cardiovascular disease.
Which of the following is a feature of pancreatic
beta cell function? Investigations reveal:
(Please select 1 option)
total Cholesterol 4.8 mmol/l (<5)
1- Beta cells account for 30% of the Endocrine
cell mass. LDL-Cholesterol 2.8 mmol/l (< 3)
HDL-Cholesterol 1.1 mmol/l (0.9 - 1.9)
2- C-Peptide is secreted with insulin in a 2:1
molar ratio Hba1c 7.1% (<6)
Plasma Triglyceride 1.9 mmol/l (0.9 - 2.2)
3- C-peptide is a by-product of insulin
proteolysis TSH 1.9 mu/l (0.32 - 5)
4- Early phase insulin secretion is delayed and Which of the following parameters is the most
diminished in people with impaired glucose important coronary heart disease risk factor in this
tolerance lady?
5- Insulin is produced by proteolytic cleavage (Please select 1 option)
from pro- insulin 1- Hba1c
2- HDL-Cholesterol
Answer & Comments
3- LDL-Cholesterol
Answer: 5- Insulin is produced by proteolytic
cleavage from pro- insulin 4- Total cholesterol
5- Systolic Blood pressure
Insulin is produced in the pancreatic beta cell by
proteolytic cleavage from pro-insulin resulting in a
by-product, namely c-peptide which is secreted
Answer & Comments Using clinical diabetes as the criterion, most twin
Answer: 2- HLA genes on chromosome 6 are studies have demonstrated a concordance rate for
monozygotic twins of between 45 – 95% and for
closely linked to autoimmune process
dizygotic twins between 3 and 37%. When glucose
tolerance tests are performed in ‘non-diabetic’
Multiple HLA loci on chromosome 6 are linked monozygotic co-twins available data suggests that
with the pathophyisology of type 1 diabetes. the concordance rate is usually over 70%.
Obesity is not a contributory factor to the
pathophysiology of type 1 diabetes, however Concordance rate for type II diabetes mellitus in
there is an increasing incidence of obesity in monozygotic twins: actuarial analysis.
patients with type 2 diabetes as consequence of In this scenario, the addition of ezetimibe to a
hyperinsulinaemia related to insulin resistance. simvastatin is supported by the IMPROVE-IT study
This hyperinulinaemia is not however appropriate and are included in guidance. Current guidelines
to the level of glycaemia. Identical twin do not advocate the use of fibrate therapy as an
concordance is high (45 – 95%) in twin studies of add on to statin therapy. ( < 2mmol/l).
patients with type 2 diabetes.
Lipid Modifying Drugs NICE
[ Q: 349 ] Question ID #34
[ Q: 350 ] Question ID #35
A 49 year old man attends clinic for review. He has
a 2 year history of type 2 diabetes which was A 53 year old lady with a 3 year history of type 2
diagnosed following admission to hospital with a diabetes is reviewed in clinic; she has a previous
troponin positive cardiac event. His current history of hypothyroidism but has no other
medication includes simvasatin 40 mg at night, medical history of note. Her current medication
metformin 500 mg three times daily, aspirin 75 mg includes metformin 500mg three times a day,
once daily, Perindopril 4 mg once daily and simvatatin 20 mg at night, ramipirl 2.5 mg once a
Bisoprolol 2.5 mg once daily. day and thyroxine 100 mcg once a day.
On examination, his BMI is 30 kg /m2, his blood On examination her BMI is 32 kg /m2 blood
pressure is 141/79 mmHg and his pulse is 62 beats pressure 135/85 mmHg and her pulse is 62 beats
per minute. per minute.
This lady appears to have sub-optimal thyroid This lady’s primary concern with respect to insulin
hormone replacement which is likely to contribute therapy is minimising hypoglycaemia risk. Her BMI
to her dyslipidaemia. Hypothyroidism is a is also elevated thus a secondary consideration
recognised risk factor for lipid lowering therapy would relate to the minimization of any weight
associated myoptoxicity, therefore prior to gain associated with exogenous insulin therapy.
escalating her specific lipid lowering medications Basal insulin analogues such as Insulin Detemir
it is important to correct her thyroid profile. and Glargine are associated with reduced
hypoglycaemia risk compared with NPH insulin,
while results of the 4T study demonstrated that
[ Q: 351 ] Question ID #49
insulin initiation with Detemir in patients receiving
on-going metformin and sulphonylurea
A 65 year old lady is referred to the clinic for combination therapy was associated with reduced
insulin initiation. She has a 7 year history of type 2 hypoglycaemia and less weight gain compared
diabetes with deteriorating glycaemic control over with either TDS pre-prandial novorapid or twice
the last 2 years resulting in a most recent Hba1c daily novomix 30 regimens.
measuring 9.1%. Her current medication includes
metformin 1g BD, Gliclazide 16 mg BD, Ramipril
5mg OD, furosemide 40mg OD, simvastatin 40mg [ Q: 352 ] Question ID #50
od and bisoprolol 1.25mg OD. She has a previous
history of ischaemic heart disease having been A 42 year old South Asian gentleman is referred to
admitted to hospital 8 months previously with an the diabetic clinic by his GP. He has a 5 year history
episode of heart failure. of type 2 diabetes. He complains of feeling tired
and has a previous history of ischemic heart
On examination, her BMI measures 35 kg/m2, her disease.
blood pressure 140/80 mmHg and she has
evidence of background retinopathy on retinal On examination BMI 34 kg m2, pulse 52 bpm,
screening. She has been previously intolerant of blood pressure 148/78 mm/Hg. Current
exenetide due to nausea and had no significant medication consists of 'Novomix 30/70' mixed
response when commenced on sitagliptin. You insulin - 42 IU twice daily, metformin 500 mg twice
contemplate insulin initiation for her but she daily, simvastatin 20 mg od, aspirin 75 mg od,
informs you that she is worried about enalapril 20 mg od, atenalol 50 mg od.
commencing insulin as she lives alone and is
worried about hypoglycaemia when commenced Investigations –
on insulin.
Hbalc 7.9% (<6)
Which of the following insulin regimens would be
Total Cholesterol 6.9 mmol/l (<5)
the most appropriate for this lady in terms of
hypoglycaemia avoidance? LDL-Cholesterol 4.5 mol/l (< 3)
(Please select 1 option) Triglyceride 2.9 mmol/l (0.9 - 2.2)
A 69 year old lady with a 4 year history of type 2 A 51 year old man with a 12 month history of diet
diabetes attends the clinic for annual review. She controlled type 2 diabetes has an Hba1c of 7.4%
is currently taking metformin 1 g bd together with (<6). On examination his BMI is 33 kg/m2 and his
lisinopril 20mg daily. She is well and little is noted blood pressure is 134/88 mmHg. He is
on examination other than a BMI of 31.8 kg/m2 consequently commenced on metformin at a
and a blood pressure of 144/84 mmHg. She has an staring dose of 500mg BD.
HbA1c of 58 mmol/mol (7.5% (<6)).
Which of the following is a recognized feature of
Which of the following would be the most metformin therapy?
appropriate approach to the further management (Please select 1 option)
of her hyperglycaemia?
1- Metformin improves hepatic glucagon
(Please select 1 option)
sensitivity
1- Add Exenatide
2- Metformin undergoes both hepatic and
2- Add Gliclazide renal clearance
3- Add Pioglitazone 3- Metformin decreases splanchnic glucose
4- Add Liraglutide turnover
Direct renin inhibitors, such as aliskiren, act to pancreatitis in the general population (0.44- 0.47
block the action of renin in converting cases per 1000 patient years). Neutralizing
angiotensinogen to angiotensin I. antibodies to exenatide have been infrequently
identified, with a maximum incidence of around
[ Q: 361 ] Question ID #59 3%. Anti-exenatide antibodies are however
identified in up to 40% of patients, the clinical
relevance of this finding is unclear.
A 56 year old man with a 5 year history of type 2
diabetes has a Hba1c of 8.2% (<6), a BMI of 36 kg/
m2 and blood pressure of 155/78 mmHg. His [ Q: 362 ] Question ID #77
current medication includes metformin 850 mg
tds, glicalzide 160 mg bd, simvaststin 20 mg daily, A publication reports the outcome of a new statin
ramipril 5 mg daily and bendrofluazide 2.5 mg therapy in a placebo controlled primary
daily. Based on current NICE guidelines he is prevention of ischaemic heart disease in a diabetic
commenced on exenatide at a starting dose of 5µg population. 1000 patients were randomised to
bd. receive the new therapy and 1000 allocated to
placebo. The study was completed over a five year
Which of the following is a feature of the period. In the placebo group there were 150
therapeutic profile of exenatide? myocardial infarcts and in the group treated with
the new statin there were 100 myocardial infarcts.
(Please select 1 option)
1- Exenatide has over 90% homology to human What is the number needed to treat with the new
GLP-1 statin versus the conventional statin to prevent
2- Exenatide undergoes hepatic and renal one MI over the course of this study?
clearance (Please select 1 option)
Exenetide is an exendin derived peptide with 53 % In the new statin group, there were 100 MIs in a
homology to human GLP-1 and is consequently 1000 patients treated. In the conventional statin
resistant to degradation by human DPP-IV. It is group there were 150 MIs in a 1000 treated
predominantly cleared via renal excretion and is patients. Therefore treating 1000 patients one
therefore not recommended in patients with an would have 50 fewer MIs. Thus the Number
eGFR < 30. Exenatide has a half-life of around 2.5 needed to treat with the new statin to prevent 1
hours and therefore is given twice daily by MI is 1000/50 = 20.
subcutaneous injection. No causal link between
exenatide and pancreatitis has to date been [ Q: 363 ] Question ID #78
identified, although a higher incidence of
pancreatitis has been reported in association with A new antiplatelet agent has been proven to
exenatide (0.49 cases per 1000 patient years) reduce the risk of stroke in a year from 10% in
compared with the background rate of
A 54 year old male attends the diabetic clinic for [ Q: 365 ] Question ID #115
routine review. He has had diabetes for ten years
and is currently receiving metformin 1g bd
A 62 year old male, who has been newly diagnosed
combined with biphasic insulin – novomix 30. He
with diabetes by his GP, is referred to clinic. He is
also takes simvastatin 40 mg daily. Investigations
on a diet, but currently taking no medication. His
reveal:
blood pressure is 136/82 mmHg, his BMI is 32
kg/m2 and his investigations reveal:
87 mmol/mol (10.1%)
HbAlc
(<6)
Fasting Plasma Glucose 7.6 mmol/l (3.5 - 5.5)
U+Es Normal
HbA1c 6.65 (<6)
Total Cholesterol 5.5 mmol/l (<5)
Total Cholesterol 5.0 mmol/l (<5)
hdL 0.8 mmol/l (0.9 - 1.9)
HOMA is one of the most widely used and easiest Answer & Comments
techniques of measuring insulin
sensitivity/resistance. It generally correlates well Answer: 2- Ileum
with insulin clamp studies.
GLP-1 is a hormone produced by the L-cells of the
[ Q: 369 ] Question ID #119 ileum and has effects on insulin and glucagon
secretion by the pancreatic islets, as well as having
central effects regulating caloric intake.
‘Healthy fasted subjects are given a 50g
carbohydrate meal and glucose concentrations
are measured every 15mins for the first hour and [ Q: 371 ] Question ID #121
every 30 mins up to 2 hours. These glucose
concentrations are then compared with values Which of the following effects are mediated by
after the consumption of 50g of glucose’. GLP-1?
(Please select 1 option)
To which of the following does this methodology
refer? 1- Delayed gastric emptying time
(Please select 1 option) 2- Increased jejunal transit time
She is generally well and has a history of diabetes, Which of the following would be a reason for
depression and hypertension for which she is screening this patient for gestational diabetes?
receiving gliclazide 80mg bd, metformin 500 mg (Please select 1 option)
tds, Lisinopril 20 mg daily, amlodipine 10mg daily
and citalopram 20mg daily. Her hyponatraemia is 1- Her age
confirmed and the following results are returned: 2- Her BMI
3- Her family history of diabetes
Urine sodium 22 mmol/l
Urine osmolality 610 nmosmol/l 4- Her history of polycystic ovarian syndrome
5- The previous weight of her infant
Which of the following is the likely cause of her
hyponantraemia?
Answer & Comments
(Please select 1 option)
Answer: 2- Her BMI
1- Citalopram
2- Gliclazide Screening for gestational diabetes is hotly debated
3- Hypoadrenalism with the general acceptance of case selection
being advocated. NICE advise testing for GDM in
4- Lisinopril the following scenarios:
1st degree relative with diabetes (her Which of the following foods has the highest GI
cousin is not a first degree relative) value?
Previous pregnancy with macrosomia (Please select 1 option)
(>4.5kg)
Previous gestational diabetes 1- Apples
BMI above 30
2- Baked Potatoes
Family origin with a high prevalence of
diabetes 3- Boiled Potates
4- Peaches
[ Q: 374 ] Question ID #165
5- Spaghetti
Which of the following foods should he specifically The Glycaemic Index (GI) relates solely to
avoid? carbohydrates and is a measure of by how quickly
a food raises plasma glucose compared with
(Please select 1 option)
glucose itself. Carbohydrates can be scored from 0
1- Apples to100 where glucose has a GI of 100. Thus, the GI
index provides information on whether a food
2- Grapefruits raises blood sugar levels dramatically, moderately
3- Oranges or minimally - high, moderate or low GI values.
Apples, peaches Oranges and even Chocolate are
4- Peaches considered low GI (less than 55). Yet, through
5- Pineapple different preparation, the GI can alter –mashed
potatoes (70) and baked potatoes (85) have a high
GI (above 70) whilst boiled potatoes have a
Answer & Comments moderate GI of 58. Foods only appear if they
Answer: 3- Oranges contain carbohydrate hence meats, eggs and fish
do not appear in the GI index. The GI is important
in diabetes and conditions related to post-prandial
Hyperkalaemia is a common biochemical hypoglycaemia or dumping syndrome. Generally,
abnormality in chronic renal failure and a low the lower the GI index the ‘better’ the
potassium diet is frequently required. Foods with carbohydrate.
a high potassium content which should be avoided
include Avocado, Banana, Tomato, Oranges,
Orange juice and Potato. Also canned fruits have a [ Q: 376 ] Question ID #167
high potassium content.
A 52 year old male is seen 6 weeks after being
admitted to hospital with an inferior myocardial
[ Q: 375 ] Question ID #166
infarction. He was diagnosed with type 2 diabetes
two years ago and was taking metformin 500 mg
A 23 year old type 1 diabetic man has noticed tds, ramipril 10mg daily and simvastatin 40mg
some problems in relation to his post-prandial daily. He had received thrombolysis and made an
glucose excursions after eating certain foods. He uneventful recovery. He was a smoker of 10
has read about the glycaemic index (GI) of foods cigarettes per day but has since given up. After his
and its impact on post-prandial glucose MI, he was also prescribed atenolol 25 mg daily
concentrations. and aspirin 75mg daily.
This gentleman meets the criteria for the Which of the following criteria would suggest a
metabolic syndrome, as defined by the diagnosis of metabolic syndrome in this gentleman
International Diabetes Federation, since he has according to the International Diabetes
central obesity (waist circumference ≥94 cm), a Federation?
plasma triglyceride ≥1.7 mmol/l, blood pressure (Please select 1 option)
≥130 / 85 mm Hg and a HDL-cholesterol < 1.04
mmol/l. 1- BMI, LDL-C concentration and blood
pressure
The IDF definition of the metabolic syndrome is 2- BMI, fasting plasma glucose and plasma
based on the presence of central obesity (waist triglyceride concentration
circumference in Europeans is ≥94 cm in men and
≥80 cm in women) combined with any two of the 3- Waist circumference, HDL-cholesterol
following: concentration and blood pressure
4- Waist circumference, triglyceride
Plasma
≥1.7 mmol/l concentration and LDL-cholesterol
Triglyceride
concentration
HDL
< 1.04 ( Men) 5- Waist circumference, HDL-cholesterol and
Cholesterol
LDL-cholesterol concentration
Answer & Comments Which of the following is associated with the use
of CGM during pregnancy?
Answer: 3- Waist circumference, HDL-cholesterol
concentration and blood pressure (Please select 1 option)
In accordance with NICE guidelines, which of the Which of the following methodologies is outlined
following would be the most appropriate above?
management with respect this man’s glycaemic (Please select 1 option)
control?
1- Homeostatic Model Assessment (HOMA)
(Please select 1 option)
2- Hyperinsulinaemic euglycaemic clamp
1- Add Gliclazide 80 mg once a day
3- Intravenous glucose tolerance test
2- Add pioglitazone 30 mg once daily
4- Quantitative insulin sensitivity check index
3- Add vildagliptin 50 mg twice daily
5- Hyperglycaemic clamp
4- No other intervention required
5- Start once daily insulin glargine Answer & Comments
Answer: 2- Hyperinsulinaemic euglycaemic clamp
Answer & Comments
Answer: 1- Add Gliclazide 80 mg once a day This is the classical example of the
hyperinsulinaemic euglycaemic clamp used as the
This man has poor glucose control with associated gold standard to assess insulin sensitivity. Clamp
symptoms. According to current NICE guidance a techniques are relied upon in research studies as
sulphonylurea would eb the most appropriate well as phase 3 and 4 interventions to assess the
additional therapy (May 2009) Vildagliptin would impact of pharmacological agents and
be an appropriate alternative is problemtaic interventions impact on insulin sensitivity.
hypoglycaemia should occur. He is not a candidate
for Pioglitazone since he has a previous history of [ Q: 383 ] Question ID #175
heart failure, even though there is no current
clinical evidence of heart failure.
A 48 year old Asian lady is referred to the diabetes
clinic. She has a 4 year history of type 2 diabetes
[ Q: 382 ] Question ID #174 and speaks very little English. Her current
medication includes Metformin 1 gram twice
The following description summarizes a daily, Gliclazide 160 mg twice daily, Lisinopril 10
standardized technique outlined in a research mg daily and Simvastatin 20 mg daily. On
proposal that you are submitting for local ethical examination, her BMI is 28 kg/m2, her body weight
approval: is 71 kg and her blood pressure is 136/76 mmHg.
Answer & Comments The DAFNE (Dose Adjustment For Normal Eating)
patient education programme is designed for
Answer: 2- Conversation mapping people with type 1 diabetes. It is a course running
over 5 consecutive days primarily focusing diet
composition and insulin dose changes.
Conversation mapping would be the most
appropriate for this lady, as it is a primarily visually
based patient education course and as such it can [ Q: 385 ] Question ID #177
be effectively delivered with the support of
interpreters. It is probably the most useful of the A 56 year old man is admitted to hospital with a
listed educational techniques for patients with lower respiratory tract infection. He is noted on
limited English speaking capacity. admission to have a random plasma glucose of
15.6 mmol/l (3.5-5.5). He is treated with oral
[ Q: 384 ] Question ID #176 antibiotic therapy, 3 months following his
discharge he undergoes a 75 gram oral glucose
tolerance test, the results of which are outlined
A 27 year old lady with a 5 year history of type 1
below:
diabetes is seen in the clinic. She is generally well
and expresses a wish to be referred to a structured
education programme as a means of improving Fasting plasma glucose 7.5 mmol/l (< 5.6)
her blood glucose control. On examination her
BMI is 25 kg/m2, her body weight is 68 kg and her 2-hour post challenge plasma glucose 13.1 mmol/l
blood pressure is 125/75 mmHg. Her current (<11.1)
mediation includes Novorapid and Levemir at a
total daily dose of 84 units. Which of the following education programmes
would be most appropriate for this man?
Investigations reveal: (Please select 1 option)
1- BERTIE
Hba1c 8.9% (<6)
2- Conversation mapping
Total 4.1 mmol/l (<4)
Plasma Triglyceride 1.1 mmol/l (0.9 - 2.2) 3- DAFNE
Thyroid Function Normal 4- DESMOND
5- X-PERT
Which of the following structured patient
education programmes would be the most
appropriate for this lady? Answer & Comments
(Please select 1 option) Answer: 4- DESMOND
1- BERTIE
This man has a new diagnosis of type 2 diabetes
2- Conversation mapping based on his oral glucose tolerance test. The
Which of the following is the most appropriate months ago due to progressive weight gain; he has
method of managing this lady’s blood glucose also been intolerant of orlistat due to abdominal
during this period? discomfort and diarrhoea. On examination, his
BMI is 43 kg/m2 and his blood pressure is 166/88
(Please select 1 option)
mmHg.
1- Continue with intravenous sliding scale
insulin Investigations reveal:
2- Commence sub-cutaneous sliding scale
Hba1c 8.9% (<6)
insulin
Total Cholesterol 4.9 mmol/l (<5)
3- Introduce sub-cuatneous basal insulin
regimen Plasma Triglyceride 3.1 mmol/l (0.9 - 2.2)
TFTs Normal
4- Introduce a subcutaneous GLP-1 analogue
5- Re-introduce gliclazide in association with Which of the following treatments is associated
her enteral feed with the greatest reduction in future
morbidity/mortality for this man?
Answer & Comments (Please select 1 option)
glicalzide 160 mg twice daily, Warfarin as per INR, exists, however, GLP-1 has been shown to exert
atorvastatin 40 mg daily, perindopril 4 mg daily beneficial effects on left ventricular function.
and bisoprolol 5 mg daily.
Reference:
On examination his blood pressure is 155/75 Renal failure and exenatide
mmHg, his BMI is 38 kg/m2 and he has no evidence
of cardiac failure. The cardiology team are thus
[ Q: 391 ] Question ID #184
considering the possibility of exenatide, but are
unsure of the any specific contraindications.
A 56 year old man with a body mass index of 34
Further investigations reveal: kg/m2 is found to have a random plasma glucose
of 12.1 mmol/l during a recent admission to
hospital with a lower respiratory tract infection.
Hba1c 8.9% (<6)
One month later he has his fasting plasma glucose
Total Cholesterol 4.9 mmol/l (<5) measured on two occasions over a 5 day period
Plasma revealing fasting plasma glucose levels of 6.7
3.9 mmol/l (0.9 - 2.2) mmol/l and 6.9 mmol/l respectively.
Triglyceride
55 ml/min/1.73m2; (90-
eGFR Which of the following would be the most
110)
appropriate approach regarding this patient's
glucose concentrations?
Trans thoracic echo LV ejection fraction of 35%
with an LV mural thrombus (Please select 1 option)
and his BMI is 33 kg/m2. Urinalysis reveals 3+ Which of the following is the most appropriate
protein and 1+ glucose and a 24 hour urinary approach to improving this young man's glucose
collection is subsequently arranged. control?
(Please select 1 option)
Which of the following urine albumin
concentrations signifies overt proteinuria? 1- Administer correction bolus at 7 am
(Please select 1 option) 2- Increase basal insulin rate between 11 pm
1- 20 mg / 24 hours and 8 am
5- 300 mg / 24 hours
Answer & Comments
Answer & Comments Answer: 3- Increase basal rate between 5am and 8
am
Answer: 5- 300 mg / 24 hours
This man's glucose profile is typical of the dawn
Microalbuminuria = Urine albumin: 20- phenomenon with progressive increase in early
200 mg/g creatinine (men); 30-300 mg/g morning plasma glucose levels in the absence of
creatinine (women). nocturnal hypoglycaemia. This phenomenon is
primarily related to hepatic gluconeogenesis and
Overt albuminuria = Urine albumin: >300 glycogenolysis, hence the correct management
mg/day. would be to increase the basal insulin infusion rate
Overt proteinuria = Urine total protein: during this period in order to suppress hepatic
glucose release.
=300 mg/day.
The X-PERT (and DESMOND) patient programme is This lady is at an increased risk of hypoglycaemia
6 week educational programme specifically during the month of Ramadan, therefore glucose
designed for patients with type 2 diabetes. It has independent insulin providing therapies such as
been shown to be effective in improving a number exogenous insulin and sulphonylureas do not
of parameters in diabetes care in trained represent the most appropriate treatment option.
individuals. Exenatide’s mechanism of action utilises the
incretin effect, resulting in glucose dependent
[ Q: 395 ] Question ID #188 stimulation of beta cell insulin release and would
thus be associated with a reduced risk of
hypoglycaemia, particularly in the absence of
A 55 year old Asian lady with a 5 year history of
concomitant sulphonylureas therapy. Exenatide is
type 2 diabetes is reviewed in clinic. Her current
however associated with appetite suppression
medication includes metformin 1 gram twice daily,
and is therefore not acceptable during Ramadan.
pioglitazone 30 mg once daily, simvastatin 20 mg
Sitagliptin is a DPPIV-enzyme inhibitor and thus
once daily and perindopril 4 mg once daily. On
also utilises the incretin effect to produce glucose
examination her blood pressure is 144/78 mmHg,
reduction, but does not have an effect on appetite
her BMI is 36.8 Kg/m2 and her body weight is 75.8
suppression and is thus acceptable.
kg. She is a devout Muslim and wishes to fast
during daylight hours during the month of
Ramadan. [ Q: 396 ] Question ID #189
18th birthday and requests some advice on the On examination his body weight is 88 kg, his BMI
effect of alcohol on his diabetes management. is 28 kg / m2 and his blood pressure is 110 / 70
mmHg. His most recent Hba1c was 65 mmol/mol
Which of the following is the most appropriate (8.1%) (<6) and his main current concern is how to
advice for this young man? optimally manage his diabetes in relationship to
his sporting activities, since he is a keen rugby
(Please select 1 option)
player having recently been included in the
1- Alcohol increases blood glucose levels and national under 21 squad. As his level of exercise
insulin doses should be increased has increased, he has noticed increased variability
in his blood glucose profile and an increased
2- Alcohol produces reductions in blood
frequency of both elevated and low blood sugars,
glucose in the short term so that insulin which is starting to adversely affect his athletic
doses should be reduced performances.
3- Alcohol reduces blood glucose levels for
several hours after and insulin doses should Which of the following is the most appropriate
be 75% advice for this young man?>
(Please select 1 option)
4- Spirits have less effect on blood glucose than
beer 1- Consume energy drinks immediately before
and during exercise
5- Alcohol produces reductions in blood
glucose levels for several hours after 2- Increase his dose of prandial insulin with his
drinking a pre bed meal is advised in meal following playing rugby to facilitate
conjunction with a modest reduction in normalisation of elevated blood glucose
insulin dose levels which may have occurred as a
consequence of exercis
Answer & Comments 3- Omit his dose of prandial insulin with his
meal prior exercise or playing a rugby game,
Answer: 5- Alcohol produces reductions in blood
while increasing his basal insulin dose on the
glucose levels for several hours after drinking
evening following playing rugby a game of
a pre bed meal is advised in conjunction with
rugby
a modest reduction in insulin dose
4- Reduce his dose of basal insulin on the
Alcohol produces short term elevations in blood evening following playing a game of rugby
glucose followed by longer term reduction in and also reduce his dose of prandial insulin
blood glucose levels after drinking. It is therefore with his meal prior to playing rugby
important to advise patients of this possibility and
5- Reduce his dose of basal insulin on the
the need for pre bed food in conjunction with a
evening before he plays a rugby game
modest dose reduction following drinking.
A 27 year old lady attends clinic for review. She has [ Q: 399 ] Question ID #192
a 10 year history of type 1 diabetes and her
current insulin regimen includes once daily insulin A 38 year old man is referred to you for
glargine at a dose of 34 units at night along with assessment following his recent diagnosis of
variable doses of pre-prandial novorapid. Her diabetes, based on two fasting plasma glucose
most recent Hba1c is 57 mmol/mol (7.4%) (<6), levels of 7.9 mmol/l and 8.1 mmol/l. On
but she is keen to achieve an HbA1c of < 48 examination his blood pressure is 146/76 mmHg,
mmol/mol (6.5%) prior to attempting to conceive his BMI is 31 kg /m2 , his body weight is 90 kg and
since during a first unplanned pregnancy 2 years his waist circumference is 97 cm. He is currently
previously she had experienced a miscarriage receiving no medication; he is a non-smoker and
which she attributed to her poor blood glucose drinks alcohol infrequently. He has no family
control at that time. She conducts regular blood history of note and his main complaint is that of
glucose monitoring and is motivated to modify her feeling a little lethargic with some loss of libido.
insulin doses according to her blood glucose
profile. HbA1c 6.8% (<6)
Which of the following is the most appropriate Total Cholesterol 4.2 mmol/l (<5)
advice for this lady in order to achieve a reduction Plasma Triglyceride 1.5 mmol/l (0.9 - 2.2)
in her Hba1c? TSH 0.98 mu/l (0.32 - 5.5)
(Please select 1 option) Testosterone 8.1 nmol/l (9 - 30)
1- Focus on reducing fasting blood glucose Prolactin 175 mu/l (50-450)
levels AST 92 iu/l (5-40)
2- Focus on reducing pre-meal blood glucose Alkaline Phosphatase 295 iu/l (50-110)
levels
Which of the following is the most appropriate
3- Focus on reducing 2 hour post prandial blood
investigation for this gentleman?
glucose levels
(Please select 1 option)
4- Focus on reducing pre-bed blood glucose
levels 1- Islet cell antibody serology
5- Focus on reducing pre bed and fasting blood 2- GAD antibody serology
glucose levels 3- Serum ferritin
4- Transferrin saturation
5- Overnight dexamethasone suppression test
The glycemic index of a food is defined as the area or without additions, there is still a higher blood
under the two hour blood glucose response curve glucose curve after white bread than after a low GI
(AUC) following the ingestion of a fixed portion of bread.
carbohydrate (usually 50 g). The AUC of the test
food is divided by the AUC of the standard (either The glycemic index can only be applied to foods
glucose or white bread, giving two different with reasonable carbohydrate content, as the test
definitions) and multiplied by 100. relies on subjects consuming enough of the test
food to yield about 50 g of available carbohydrate.
The average GI value is calculated from data Many fruits and vegetables (but not potatoes)
collected in 10 human subjects. Both the standard contain very little carbohydrate per serving and
and test food must contain an equal amount of thus the GI is negligible. This also applies to
available carbohydrate. The result gives a relative carrots, which were originally and incorrectly
ranking for each tested food. reported as having a high GI. Alcoholic beverages
have been reported to have low GI values, but it
The current validated methods use glucose as the should be noted that beer has a moderate GI.
reference food, giving it a glycemic index value of Recent studies have shown that the consumption
100 by definition. This has the advantages that it is of an alcoholic drink prior to a meal reduces the GI
universal and it results in maximum GI values of of the meal by approximately 15%. Moderate
approximately 100. White bread can also be used alcohol consumption more than 12 hours prior to
as a reference food, giving a different set of GI a test does not affect the GI
values (if white bread = 100, then glucose ≈ 140).
For people whose staple carbohydrate source is The GI concept has been criticized for a variety of
white bread, this has the advantage of conveying reasons –
directly whether replacement of the dietary staple
with a different food would result in faster or The glycemic index does not take into
slower blood glucose response. The disadvantages account other factors besides glycemic
with this system are that the reference food is not response, such as insulin response, which
well-defined, and the GI scale is culture is measured by the insulin index and can
dependent. A low GI food will release glucose be more appropriate in representing the
more slowly and steadily. A high GI food causes a effects from some food contents other
more rapid rise in blood glucose levels and is than carbohydrates.
suitable for energy recovery after endurance The glycemic index is significantly altered
exercise or for a person with diabetes by the type of food, its ripeness,
experiencing hypoglycemia. processing, the length of storage, cooking
methods, and its variety (white potatoes
The glycemic effect of foods depends on a number are a notable example, ranging from
of factors such as the type of starch (amylose vs moderate to very high GI even within the
amylopectin), physical entrapment of the starch same variety)
molecules within the food, fat and protein content The glycemic response is different from
of the food and organic acids or their salts in the one person to another, and even in the
meal — adding vinegar for example, will lower the same person from day to day, depending
GI. The presence of fat or soluble dietary fibre can on blood glucose levels, insulin resistance,
slow the gastric emptying rate thus lowering the and other factors.
GI. Unrefined breads with higher amounts of fiber High-glycemic-index foods may have a low
generally have a lower GI value than white breads. carbohydrate content, and low-glycemic-
Many brown breads, however, are treated with index foods may have a high carbohydrate
enzymes to soften the crust, which makes the content. This criticism can be addressed
starch more accessible (high GI). by taking into account the glycemic load
Low GI foods (GI range <55) include most
While adding butter or oil will lower the GI of a fruit and vegetables (except potatoes,
meal, the GI ranking does not change. That is, with watermelon), grainy breads, pasta,
Which of the following is the most appropriate the portal vein of the liver. The islets are then
information for this lady regarding islet cell infused slowly through the catheter into the liver.
transplantation? Specialized enzymes are used to remove islets
from the pancreas of a deceased donor. Because
(Please select 1 option)
the islets are fragile, transplantation occurs soon
1- 50% of patients who receive islet cell after they are removed. Typically a patient
transplantation will demonstrate insulin receives at least 10,000 islet “equivalents” per
independence 2 years following their final kilogram of body weight, extracted from two
islet cell infusion donor pancreases. Patients often require two
transplants to achieve insulin independence
2- Around 10% of patients remain free of
insulin injections five years after Rejection is the biggest problem with any
transplantation. transplant, the use of immunosuppressive
corticosteroid based immunosuppression may
3- islet cell transplantation requires an open
have a detrimental metabolic effect, due to
surgical procedure increased insulin resistance. Currently the
4- Long term post-transplantation Edmonton protocol of immunosuppressive drugs
corticosteroid based immunosuppressant is used, which includes including daclizumab,
therapy is required sirolimus, and tacrolimus.
A 53 year old man with a 4 year history of type 2 A 27 year old lady has developed gestational
diabetes who is currently self-employed as a taxi diabetes during her first pregnancy. She has no
driver is reviewed in clinic. His current medication family history of diabetes. She has required insulin
includes metformin 1 g od, rosiglitazone 4 mg od, therapy from 28 weeks gestation in the form of a
simvastatin 20 mg od and enalpril 5 mg od. On basal bolus regimen with an insulin requirement of
examination, his blood pressure is 146/76 mmHg, 70 units over 24 hours. Following an uneventful
his BMI is 33 kg/m2, his visual acuity is 6/9 in the delivery, which of the following should she be
right eye and 6/6 in the left eye. His metabolic advised to do?
control is good with his most recent HbA1c being
6.9% (<6). His main concern relates to the effect
diabetes may have on his eye sight and in (Please select 1 option)
particular how this may influence his ability to
drive. 1- Continue with her present insulin
2- Commence low dose once daily basal insulin
When assessing a patient’s fitness to drive which
of the following is most accurate? 3- Stop insulin and commence metformin
(Please select 1 option) 4- Stop Insulin and commence on a
sulphonylurea
1- The best corrected visual acuity must be
6/12 or better in one eye 5- Stop all treatment
2- The best corrected visual acuity should be
6/9 or better in both eyes Answer & Comments
3- The patient may not drive if there is a visual Answer: 5- Stop all treatment
field defect in one eye
4- Proliferative retinopathy is an automatic bar Gestational diabetes generally resolves once the
baby is born. Based on different studies, the
to driving
chances of developing GDM in a second pregnancy
5- The patient should be able to read a number are between 30 and 84%, depending on ethnic
plate at 25 meters background and presence of other risk factors for
diabetes such as obesity, metabolic syndrome and
family history. This patient should be re-screened
with a fasting plasma glucose measurement at the
6-week postnatal health check and annually
thereafter, as she is at a high risk of developing Diagnose gestational diabetes if the woman has
subsequent diabetes since women who required either:
insulin treatment or who have positive glutamate
decarboxylase (GAD) or islet cell antibodies (ICA) a fasting plasma glucose level of 5.6
are at higher risk of future diabetes. mmol/litre or above
a 2- hour plasma glucose level of 7.8
Reference: mmol/litre or above.
NICE clinical guidelines 63: Diabetes in
pregnancy Offer a trial of changes in diet and exercise to
women with gestational diabetes who have a
[ Q: 407 ] Question ID #205 fasting plasma glucose level below 7 mmol/litre at
diagnosis.
A 24 year old Asian lady with a strong family Offer metformin[2] to women with gestational
history of type 2 diabetes undergoes an oral diabetes if blood glucose targets are not met using
glucose tolerance test at 24 weeks gestation changes in diet and exercise within 1–2 weeks.
during her first pregnancy. The results of her 75 g
oral glucose tolerance test are outlined below. Offer insulin instead of metformin to women with
Fasting and 2 hour self blood glucose monitoring gestational diabetes if metformin is
over the subsequent 2 weeks reveal an average contraindicated or unacceptable to the woman.
fasting glucose of 5.4 mmol/l and 2-hour value of Offer addition of insulin to the treatments of
6.7 mmol/l. changes in diet, exercise and metformin[2] for
women with gestational diabetes if blood glucose
Fasting Blood Glucose 5.8 mmol/L (5.3 targets are not met.
Level mmol/l)
1 Hour Blood Glucose 10.1 mmol/l (<10 [ Q: 408 ] Question ID #206
Level mmol/L)
2 Hour Blood Glucose 9.3 mmol/l (<8.6 A 23 year old lady with a previous history of
Level mmol/L) gestational diabetes during her first pregnancy 18
3 Hour Blood Glucose 8.2 mmol/l (<7.8 months previously undergoes a 75 gram oral
Level mmol/L) glucose tolerance test at week 24 of her second
pregnancy. This reveals a 2hr post OGTT glucose of
Which is the most appropriate initial management 14.5 mmol/l. She is subsequently commenced on
for this lady? a basal bolus insulin regimen at an initial total daily
dosage of 64 units.
(Please select 1 option)
Reference: 5.3
NICE clinical guidelines 63: diabetes in pregnancy Fasting Plasma Glucose
mmol/l
1-hour post challenge plasma 8.1
[ Q: 409 ] Question ID #207 glucose mmol/l
2-hour post challenge plasma 7.7
A 32 year old Asian lady with a 2 year history of glucose mmol/l
type 2 diabetes is referred by her GP following a
positive pregnancy test subsequent to a two Which of the following is the correct interpretation
month period of ammenorrhea. Her glycaemic of these data?
control has been good, with a most recent HbA1c
(Please select 1 option)
of 6.3% (<6). She is currently only taking
metformin 500 mg twice daily and on examination 1- Gestational diabetes
her BMI is33 kg /m2 and her blood pressure is
132/70 mm/hg. 2- Impaired glucose tolerance
3- Impaired fasting plasma glucose
Which of the following is the most appropriate
4- Normal Glucose tolerance
initial approach for managing this lady’s diabetes?
(Please select 1 option) 5- Combined impaired fasting glucose and
impaired glucose tolerance
1- Commence Basal Insulin
2- Commence Basal-Bolus Insulin Answer & Comments
3- Commence Bihpasic Insulin Answer: 4- Normal Glucose tolerance
4- Continue Metformin
5- Commence Sulphonylurea Diagnose gestational diabetes if the woman has
either:
A fasting plasma glucose level of 5.6 Depending on the population studied, the
mmol/litre or above or diagnostic criteria and the length of follow-up, the
risk can vary enormously. The risk appears to be
A 2-hour plasma glucose level of 7.8 highest in the first 5 years, reaching a plateau
mmol/litre or above. thereafter. In populations with a low risk for type
2 diabetes, in lean subjects and in patients with
Diabetes in pregnancy NICE guidelines (2015)
auto-antibodies, there is a higher rate of women
NG3, gestational diabetes: risk assessment,
developing type 1 diabetes.
testing, diagnosis and management
Reference:
[ Q: 411 ] Question ID #209 Impact of gestational diabetes on the risk of
diabetes following pregnancy among Chinese
A 21 year Asian lady has developed gestational and South Asian women.
diabetes during her first pregnancy. She has no
family history of diabetes. She has required insulin [ Q: 412 ] Question ID #210
therapy from 28 weeks gestation in the form of a
basal bolus regimen with an insulin requirement of
A 35 year old lay with a 3 year history of type 2
68 units over 24 hours. 2 weeks subsequent to an
diabetes is referred to ante-natal clinic at 14
uneventful delivery she no longer requires insulin
weeks gestation. Her current medication includes
and has a fasting plasma glucose of 5.5 mmol/l and
metformin 500 mg three times a day only. On
random self measured capillary glucose levels no
examination, her blood pressure is 136/74 mmHg,
greater than 7.9 mmol/l.
her BMI is 35 kg/m 2 and she has no clinical
evidence of microvascular complications.
What is the risk of this lady progressing to develop
diabetes over the next 5 years?
HbA1c 7.5% (<6)
(Please select 1 option)
TSH 0.98 mu/l (0.32 - 5.5)
1- 1 %
U+E Normal
2- 5 - 10% eGFR 96 ml/min/1.73m² (90-110)
3- 10 - 20%
Which of the following would be the most
4- 25 - 35% appropriate method of managing this lady’s
5- 80% glycaemic control?
(Please select 1 option)
Answer & Comments 1- Increase metformin dose
Answer: 4- 25 - 35% 2- Add GLP1 analogue
3- Add Thiazolidinedione
Women diagnosed with gestational diabetes have
an increased risk of developing diabetes mellitus 4- Commence basal-bolus insulin
in the future. The risk is highest in women who 5- Commence twice daily biphasic insulin
needed insulin treatment, had antibodies
associated with diabetes (such as antibodies
against glutamate decarboxylase and islet cell Answer & Comments
antibodies), women with more than two previous
Answer: 4- Commence basal-bolus insulin
pregnancies, and women who were obese and
exhibiting features of the metabolic syndrome (in
order of importance). Women requiring insulin to This lady has suboptimal glycaemic control on
manage gestational diabetes have a 25-35% risk of metformin monotherapy. An optimal level of
developing diabetes within the next five years. glucose control should be defined by an HbA1c
target < 6.5%. This would be best achieved with Answer & Comments
conversion to a basal bolus insulin regimen. GLP1
and Thaizolidinediones are not recommended Answer: 1- Diabetes
during pregnancy, while biphasic insulin would not
represent the most effective means of managing Based on this lady’s average glucose of 8.6 mmol/l
both fasting and post-prandial glucose levels. she is likely to have a HbA1c of around 7 – 7.5%
Analogue basal and prandial insulins may be safely based on the A1c derived average glucose
used in such a regimen. Current evidence suggests calculation (Av Glucose = 1.583 X A1c – 2.52). She
that continuing with metformin in combination is at high risk of subsequent diabetes following her
with insulin may be appropriate to reduce insulin pregnancy since she required around 0.8 u / kg of
requirements as there is no evidence of a insulin, although she has no family history of
detrimental effect of metformin on pregnancy diabetes and is GAD negative. Positive family
outcome. history, GAD positivity and features of the
metabolic syndrome further increase the risk of
[ Q: 413 ] Question ID #211 incident diabetes following gestational diabetes.
Based on this glucose profile alone it is unclear
whether this lady has type 1 or type 2 diabetes.
A 24 year old lady attends clinic 6-8 weeks
following the birth of her first baby. The delivery Reference:
was uncomplicated, but during her pregnancy she NICE clinical guidelines 63: Diabetes in
developed gestational diabetes and was pregnancy
commenced on basal bolus insulin at 22 weeks
requiring a total of 54 units daily. On examination,
her blood pressure is 122/66 mmHg, her BMI is 29 [ Q: 414 ] Question ID #212
kg/m2 her body weight is 65 kg and her waist
circumference is 81 cm. She was found to be GAD A 25 year old lady attends ante-natal clinic at 14
antibody negative when she initially developed weeks gestation, having recently been
gestational diabetes and has no family history of commenced on basal bolus insulin following a
diabetes. She is concerned about the possibility of diagnosis of gestational diabetes. She has no
developing diabetes in the future and since family history of diabetes, but was diagnosed with
discontinuing insulin after her delivery she has gestational diabetes during her first pregnancy 2
been monitoring her blood glucose levels two to years previously which resulted in the successful
three times daily. On review of her meter, she has birth of a 5.3 kg baby boy. On examination her
an average glucose level of 8.9 mmol/l. She blood pressure is 135/75 mmHg, her BMI is 31
returns to clinic the next morning and her fasting kg/m2 her waist circumference is 84 cm and
plasma glucose is taken and found to be 7.6 fundoscopy reveals no evidence of retinopathy
mmol/l.
What should be the HbA1c target for this lady
What is the current diagnosis for this lady based during her pregnancy?
on her self monitored glucose profile?
(Please select 1 option)
(Please select 1 option)
1- 5.5%
1- Diabetes
2- 6.1%
2- Impaired glucose tolerance
3- 6.5%
3- Impaired fasting glucose
4- 7%
4- Combined impaired glucose tolerance and
5- Do not measure Hba1c.
impaired fasting glycaemia
5- Normal glucose tolerance
Investigations reveal:
Answer & Comments
Answer: 2- Duloxetine 60 mg daily HbA1c 8.3% (<6)
Total Choleseterol 4.7 mmol/l (<4)
In accordance with NICE guidelines 173, for people Plasma TG 1.9 mmol/l (0.9 - 2.2)
with painful diabetic neuropathy, offer oral
duloxetine 60mg, amitriptyline, pregabalin or 78 ml/min/1.73 m2;
eGFR
gabapentin. If satisfactory pain reduction is still (0.9 - 2.2)
not achieved with maximisation then switch to Urine
1.8 mg/mmol creat
another 1st line agent. In this case duloxteine is Albumin:Creatinine
(<2.5)
the right answer as the dose for gabapentin is too Ratio
high for initiation.
What is the most likely cause of this man’s
Tight control of blood glucose can reverse the symptoms?
changes of diabetic neuropathy, but only if the
(Please select 1 option)
neuropathy and diabetes is recent in onset.
Conversely, painful symptoms of neuropathy in 1- Diabetic peripheral neuropathy
uncontrolled diabetics tend to subside as the
2- Cauda equina syndrome
disease and numbness progress. Of course, these
uncontrolled patients are at great risk for diabetic 3- Mononeuritis multplex
foot ulcers and amputation.
4- Peripheral vascular disease
Reference: 5- Prolapsed intervetebral disc
NICE clinical guidelines 173: Neuropathic pain
Answer & Comments
[ Q: 419 ] Question ID #229
Answer: 5- Prolapsed intervetebral disc
A 65 year old gentelman with a 6 year history of
type 2 diabetes is reviewed in clinic and complains Mononeuritis multiplex involves damage to at
of a dead feeling in his left foot a that over the last least 2 separate nerve areas. This condition can
few weeks has become increasingly painful with become progressively worse over time. The
episodes of stabbing pain and electric shock like damage to the nerves involves destruction of the
symptoms, which do not appear to be related to axon, thus interfering with nerve conduction.
exercise. He also complains of low back pain. His Common causes of damage include a lack of
Corda equina syndrome results from acute loss of Which of the following is the most appropriate
function of the neurologic elements (nerve roots) approach for the management of this gentleman’s
of the spinal canal below the termination (conus) anemia?
of the spinal cord, with signs arising due loss of (Please select 1 option)
function related to the injured nerves.
1- EPO should not be considered until his eGFR
Diabetic peripheral neuropathy usually produces is < 30 ml/min/1.73 m2
bilateral signs and symptoms and the fact that
2- EPO would be contra-indicated due to sub-
peripheral pulses are present and that the
optimal blood pressure control
symptoms are not in keeping with claudication
make a diagnosis of peripheral vascular disease 3- He should receive iron supplementation only
also unlikely in this case.
4- He should receive iron supplementation
A prolapsed inetervetbral disc resulting in prior to considering therapy with EPO
lumbosacral nerve root compression would 5- He does not require any treatment for his
produce symptoms and signs such as those anaemia at current levels
demonstrated by this gentleman and further
investigation with a lumbosacral MRI would be
appropriate. Answer & Comments
Answer: 4- He should receive iron
[ Q: 420 ] Question ID #234 supplementation prior to considering therapy
with EPO
A 77 year old man with a 24 year history of type 2
diabetes is reviewed in clinic he feels tired but The failure of erythropoietin production and
denies any osmotic symptoms. His current therapy subsequent development of anemia is more
includes ‘novomix 30’ 32 units in the morning and common in patients with diabetes, particularly
24 units in the evening, ramipril 5 mg daily, females with type 1 diabetes. Co-existent
simvastatin 40 mg daily, indapamide 1.5 mg daily autonomic neuropathy may be an important
and aspirin 75 mg daily. On examination his blood factor as release of erythropoietin is modulated by
pressure is 134 /66 mmHg, his body weight is the sympathetic nervous system. Urinary
stable at 86 kg, his BMI is 32 kg / m2 and he has erythropoietin losses, interstitial fibrosis (90% of
reduced vibration and proprioception sensation in erythropoietin is produced by peri-tubular
both feet. fibroblasts) and angiotensin converting enzyme
inhibition may also be factors.
Investigations reveal:
Patients with anemia secondary to diabetic kidney
Hab1c 7.7% (<6) disease, however demonstrate no difference in
Total Cholesterol 4.4 mmol/l (<5) response to exogenous erythropoietin compared
with patients with non diabetic kidney disease.
Haemoglobin 10.1 g/dl (12-16) Exogenous erythropoietin has been shown to
MCV 86 pl (76-98) reduce Hba1c levels, thus when evaluating
glycaemic control in patients undergoing therapy
for renal anemia, Hba1c estimation alone may be Which of the following is the most appropriate
insufficient. It has long been postulated that approach to the evaluation of this gentleman’s
correction of renal anemia will have a positive renal function?
effect on mortality. This seems likely but at what
(Please select 1 option)
level of haemoglobin (Hb) this benefit would
become evident is less clear. 1- His renal function can be assessed by
monitoring his serum creatinine since serum
Raising haematocrit by 25% has been shown to creatinine bears a linear relationship with
reduce left ventricular mass and cross sectional glomerular function
analyses have demonstrated increased mortality
in patients with Hb < 11 g / dl compared to those 2- eGFR measurements would not be an
with Hb levels > 11 g /dl. Patietns treated to a Hb accurate means of following his renal
level > 13.5 g / dl have been shown to experience function as eGFR underestimates true GFR
higher incidence of congestive heart failure, in people with diabetes with GFR values GFR
stroke, myocardial infarction and death. < 60 ml/min/1.73m2
Consequently, current guidelines advocate that
treatment should maintain stable Hb 3- eGFR calculations for this gentleman require
concentrations between 10.5 and 12.5 g / dl, with correction by multiplying by a factor of 1.2.
adjustments to treatment considered when Hb 4- This gentleman has chronic kidney disease
levels < 11.0 g / dl or >12.0 g / dl. Age alone should
stage 3 and should be treated to a blood
not be determinant of treatment for renal anemia.
pressure target of 130 / 80
Appropriate iron stores are required for the
optimal management of renal anemia with EPO 5- Blood pressure is the primary determinant of
and this is defined by a serum ferritin this gentleman's kidney function
concentration of between 200 and 500 µg / l
combined with either a transferrin saturation >
20% ( unless ferritin > 800 µg / l) or percentage Answer & Comments
hypochromic red cells < 6%( unless ferritin > 800 Answer: 3- eGFR calculations for this gentleman
µg / l). require correction by multiplying by a factor of
1.2.
[ Q: 421 ] Question ID #235
Glomerular filtration rate (GFR) is the gold
A 46 year old man of Afro-Caribbean origin has standard measure of renal function and has a non
recently been diagnosed type 2 diabetes managed linear relationship with serum creatinine. There
by lifestyle modification. He has no previous are three physiological determinants of GFR
medical history of note. On examination his blood namely, Ultrafiltration pressure, permeability of
pressure is 155 / 80 mmHg, his body weight is 94 the glomerular wall and surface area of capillary
Kg, his BMI is 34 kg/m2 and direct fundoscopy is available for filtration. GFR is important because
normal. as it reduces, for reasons not completely clear,
there is an associated increase risk of
Investigations reveal: cardiovascular disease. In the South Tees
Mortality study the adjusted hazard ratio for
Hba1c 6.9% (<6) ischemic heart disease was 3.6 and 8.1 fold higher
for those with eGFR 30 – 59 and < 30 ml/ min/1.73
Total Cholesterol 4.9 mmol/l (<4)
m2 respectively. Consequently, all patients with an
58 ml/min/1.73m² eGFR < 60 ml/min/1.73 m2are defined as having
eGFR
(90 - 110) chronic kidney disease irrespective of their
Serum Creatinine 128 µmol/l (56 - 102) urinalysis. The MDRD equation is used to calculate
eGFR and this under estimates GFR particularly in
Urine Albumin 1.9 mg.mmol/creat
patients with values > 60 ml/min /1.73 m2. GFR
Creatinine Ratio (<2.5)
varies according to gender, muscle mass and other
How would you advise her? Once she has recovered the following is the
appropriate course of action?
(Please select 1 option)
(Please select 1 option)
1- Lower all basal rates by 0.1units/hour
1- Cessation of continuous subcutaneous
2- Lower her basal rates by 0.1units/hour 1pm insulin infusion therapy is indicated as she
to 6pm has had an episode of diabetic ketoacidosis
3- Lower her basal rates by 0.1units / hour 6am 2- Her insulin via the pump needs to be
- 1pm increased by 20%
4- Alter her lunchtime insulin to carbohydrate 3- She should return to a basal bolus regime for
ratio to 1 unit to 15g 3 months to allow her insulin requirements
5- Increase the "low glycaemic index" to be recalculated.
carbohydrate content of her lunch
4- She should be advised to temporarily 2- Continue the pump therapy with the
override to 150% of her usual basal rates if anaesthetist monitoring the blood glucoses
the situation recurs. intra-operatively and giving intravenous
glucose if needed.
5- She should be advised that if an initial bolus
via the pump for hyperglycaemia does not 3- Switch to intravenous glucose and insulin the
lower her glucose then a bolus should be night before the procedure and continue
given by an insulin pen and the pump until eating and drinking
removed and the set up procedure
4- Discontinue the pump on induction of
repeated.
anaesthesia and observe glucoses. Start
intravenous insulin if needed.
Answer & Comments
5- Anaesthetist to alter the pump during the
Answer: 5- She should be advised that if an initial procedure and post-operatively.
bolus via the pump for hyperglycaemia does
not lower her glucose then a bolus should be Answer & Comments
given by an insulin pen and the pump removed
and the set up procedure repeated. Answer: 2- Continue the pump therapy with the
anaesthetist monitoring the blood glucoses
Insulin delivery may become discontinued if there intra-operatively and giving intravenous
is a mechanical fault with the pump or if a cannula glucose if needed.
has become blocked or kinked and this can result
in diabetic ketoacidosis. This can be prevented if Hysteroscopy is a procedure requiring a short
the patient is educated to give an insulin bolus by anaesthetic with an expected rapid recovery
pen or syringe if hyperglycaemia is not responding usually performed as a day case. If continuous
to boluses from the pump. This patient may have subcutaneous insulin infusion therapy is stopped
forgotten this rule particularly if the pump was alternative insulin therapy should be started
commenced in an urgent manner when she had within 1 hour, ideally immediately. If the cessation
poor control or problems in pregnancy. is protracted hyperglycaemia is inevitable and
diabetic ketoacidosis could result. Although using
[ Q: 424 ] Question ID #240 intravenous glucose and insulin therapy from the
night before is not wrong, it is unlikely to be
necessary and will result in an unnecessary
A 53 year old on continuous subcutaneous insulin hospital admission and hourly glucose estimations
therapy is due to be admitted for a hysteroscopy overnight which is disruptive. The anaesthetist is
and endometrial biopsy under general unlikely to be familiar enough with the pump
anaesthetic. She has been on continuous mechanism to alter this and could monitor the
subcutaneous insulin infusion therapy for 5 years glucoses and ensure hypoglycaemia does not
and has an HbA1c of 5.5%. The anaesthetist occur during anaesthesia when it will not be
contacts you for advice on glycaemic management detected.
during surgery.
How will you advise her? correction bolus of 8units of insulin after training
(Please select 1 option) but finds that overnight he is having
hypoglycaemic episodes.
1- Continue with the usual basal rates as per UK
time and temporarily override as needed. How would you advise him?
2- Set the pump at the lowest basal rate for the (Please select 1 option)
duration of the flight, bolus for food as usual 1- Give a bolus of insulin at the start of his
and set the clock on the pump to local time squash match or training and reduce the
on arrival basal rate throughout
3- Switch back to a basal bolus regime using 2- Give a bigger bolus of insulin with his post
insulin glargine and insulin aspart for the squash meal and increase basal rates to
journey bedtime
4- Set the basal rate to 0.2unit / hour and check 3- Increase the basal rates during the squash
blood glucoses every 2 hours and give match / training
insulin boluses as necessary
4- Give a smaller bolus e.g. 4 units and keep the
5- Set the pump to 50% of usual basal rates and basal rates the same across the evening and
give insulin boluses as needed. night.
5- Increase the basal rates leading up to the
Answer & Comments
squash match / training
Answer: 2- Set the pump at the lowest basal rate
for the duration of the flight, bolus for food as Answer & Comments
usual and set the clock on the pump to local
time on arrival Answer: 1- Give a bolus of insulin at the start of his
squash match or training and reduce the basal
rate throughout
The usual basal rates alter depending on the time
of day, the diurnal hormonal variations and usual
routine and travel across several time zones will Giving more insulin as a bolus or by increasing the
disrupt these factors. Setting the pump to the basal rates before exercise may cause
lowest usual basal rate allows travel to be safely hypoglycaemia during exercise. The
undertaken and food boluses and correction catecholamines released during exercise may
boluses can be given as needed. If the pump clock cause hyperglycaemia and the extent may vary
is switched to local time on arrival the usual basal depending on the activity. Hypoglycaemia up to 24
daily rates can resume with minimal disruption. hours after activity may occur. His bolus of 8 units
of insulin is probably excessive for him and
injecting the Insulin just prior to exercise may help
[ Q: 426 ] Question ID #242 to reduce his post exercise hyperglycaemia;
generally 1 unit of insulin will reduce the glucose
A 28 year old man with type 1 diabetes mellitus is by 2.5mmol/l although this will vary depending on
on continuous subcutaneous insulin infusion and insulin sensitivity. Reducing his basal rates post
presents for urgent review as he has developed exercise should help to prevent hypoglycaemia.
problems with erratic glucose levels. He trains
regularly in the gym and plays squash three times
[ Q: 427 ] Question ID #243
a week. He has found his glucoses need to be
greater than 8mmol/l prior to training or squash to
prevent hypoglycaemia but that post training they A 30 year old man takes insulin lispro 6 units three
are usually greater than 18mmol/l. He eats a high times a day and insulin glargine 16units. Despite
carbohydrate meal with a usual bolus plus a efforts from the patient and diabetes team, his
HbA1c remains greater than 9%. Any increase in (Please select 1 option)
his insulin causes recurrent severe hypoglycaemia. 1- Arrange for a district nurse to visit the school
He is to commence continuous subcutaneous
to inject the child using a needle and syringe
insulin infusion.
prior to lunch.
His starting basal rates should be: 2- Continue without a lunchtime bolus and
(Please select 1 option) encourage low glycaemic index food for this
meal
1- 0.5 units / hour
3- Continue without a bolus until the child is
2- 1.0 units / hour
able to administer his own bolus which will
3- 1.5 units / hour probably be within 1 to 2 years
4- 0.1 units / hour 4- Increase the basal rate for an hour over the
5- 0.8 units / hour period following lunch
5- Switch to a different insulin regime such as a
Answer & Comments twice daily pre-mixed regime until the child
is able to manage his own pump
Answer: 1- 0.5 units / hour
Which of the following is the most likely infective 5- Twice daily biphasic insulin
organism in this case?
(Please select 1 option)
Answer & Comments
2- H. Influenzae
The current NICE guideline update on blood
3- Methicillin resistant Staph Aureus glucose control (CG 87, May 2009) advocates
metformin as the first line oral therapy in people
4- Pseudomonas aeruginosa
with type 2 diabetes with sub-optimal glucose
5- Strep Pyogenes control (Hba1c ≥6.5%) unless the person is not
overweight, metformin is contraindicated or not
Answer & Comments tolerated or the person requires rapid therapeutic
response due to hyperglycaemic symptoms. This
Answer: 5- Strep Pyogenes gentleman describes classic hyperglycaemic
symptoms and the absence of any weight loss
suggests that this patient does not currently
The patient has a classical description of cellulitis
require exogenous insulin.
which is most likely to be due to either Staph
Aureus (not typically MRSA) or Streptococcus.
[ Q: 431 ] Question ID #303
[ Q: 430 ] Question ID #302
A 65 year old gentleman with a 3 year history of
type 2 diabetes is reviewed in clinic. He feels
A 52 year man presents with a 3 month history of
generally well and has a fairly predictable lifestyle
general lethargy and polyuria over the last month,
having been retired for the last 2 years. His current
but he denies any obvious weight loss. He has no
medication includes metformin 1 gram bd,
past medical history or family history of note.
ramipril 5 mg od and simvastatin 20 mg od.
On examination his body weight is 83 kg, his BMI
On examination there is no evidence of any
is 30 kg /m2 and his blood pressure measures
microvascular complications, his blood pressure
144/77 mmHg.
measures 145/75 mmHg, his body weight is 88 kg
and his BMI is 31 kg/m2.
Investigations reveal:
U+E Normal
Fasting glucose 14.9 mmol/l 3.5-6
55 mmol/mol
HbA1c 8.3% <6% HbA1c <6
(7.2%)
U+Es Normal
79
eGFR >90
mls/min/1.73m2
Urine analysis - Glucose +++ Ketones +
Total
4.5 mmol/l <5
cholesterol
Which of the following is the most appropriate
treatment for this man? LDL cholesterol 2.9 mmol/l <3
(Please select 1 option) HDL cholesterol 0.9 mmol/l 0.9-2
1- Gliclazide
Which of the following is the most appropriate LDL cholesterol 2.2 mmol/l <3
management for this patient? 0.9-
HDL cholesterol 0.8 mmol/l
2
(Please select 1 option)
1.1-
1- Add Exenatide Triglycerides 2.1 mmol/l
2.
2- Add Empagliflozin
Which of the following would be the most
3- Add Gliclazide
appropriate therapeutic option for managing this
4- Add Sitagliptin man's glycaemic control?
5- Continue with metformin alone (Please select 1 option)
1- Add Exenatide
Answer & Comments 2- Add tds Humalog Insulin
Answer: 5- Continue with metformin alone 3- Add once daily NPH insulin
4- Add once daily insulin glargine
The current NICE guideline update on blood
glucose control (NG 28) advocates the use of a 2nd 5- Add twice daily biphasic insulin
line oral hypoglycaemic agent in people with type
2 diabetes with sub-optimal control (Hba1c ≥ 58 Answer & Comments
mmol/mol 7.5%) receiving maximum tolerated
metformin monotherapy. A TZD/DPP-4i/SU or Answer: 1- Add Exenatide
SGLT2 inhibitor are recommended. In this case,
with the Hba1c at 54 mmol/mol the guidance does
The current NICE guideline update on blood
not suggest any requirmeent to optimise
glucose control (CG 87, May 2009) suggests that in
treatment.
people failing to maintain adequate blood glucose
control (Hba1c 7.5%) insulin should be initiated in
[ Q: 432 ] Question ID #304 the form of either once or twice daily NPH insulin
according to need, in the context of appropriate
A 52 year old bus driver is reviewed in clinic. He structured education. A basal insulin analogue is
has an 8 year history of type 2 diabetes and has recommended if the person can not use the device
background retinopathy affecting both eyes. He is to deliver NPH insulin, optimal dose titration of
generally well and denies any osmotic symptoms NPH insulin is limited by hypoglycaemia, the
or weight loss. His current medication includes person needs help with injecting insulin and a
metformin 1 gram twice daily, glicalzide 160 mg once daily analogue insulin would replace twice
twice daily, simvastatin 40 mg once daily and daily NPH. Exenatide is currently recommended
perindopril 4 mg once daily. according to the NICE guidance as an alternative
to exogenous insulin where insulin use would have
On examination his body weight is 91 kg, his BMI detrimental effects on employment, or in people
is 31 kg/m2 and his blood pressure measures with a BMI 35 Kg /m2 of European descent where
148/78 mmHg. further weight gain would exacerbate problems
associated with high body weight or in people with
Investigations reveal: BMI 35 Kg /m2 where insulin is unacceptable due
to occupational implications or where weight
reduction would benefit other co-morbidities.
U+Es Normal Biphasic insulin within the current NICE guidance
HbA1c 8.7% <6
may be considered as an option in people who are (HbA1c ≥ 6.5%) unless it is contraindicated or not
markedly hyperglycaemic e.g. Hba1c > 9.0%. tolerated. In such situations a sulphonylurea of
lowest acquisition cost – but not glibenclamide –
[ Q: 433 ] Question ID #305 is recommended. Sitagliptin is currently not
recommended in people with moderate renal
impairment defined as a creatinine clearance < 50
A 69 year old lady is reviewed in clinic. She has a ml / min.
six month history of type 2 diabetes which has
been initially managed with lifestyle modification.
She also has a five year history of treated [ Q: 434 ] Question ID #306
hypertension. Her current medication includes
amlodipine 5 mg once daily, simvastatin 20 mg A 44 year old male is referred by his GP as he is
once daily and Aspirin 75 mg once daily. concerned regarding a fasting plasma glucose
value of 5.9 mmol/l and wonders what needs to be
On examination her body weight is 71 kg, her BMI done about this. The patient informs you that he
is 29 Kg/m2 and her blood pressure is 155/75 went to the GP as he was concerned about
mmHg. Investigations reveal: developing diabetes and he tells you that he has
been thristy on occassions and may get up in the
HbA1c 7.1% <6 night twice to urinate. He has a strong family
history of diabetes with his mother and maternal
49 90- uncle suffering from the disorder. He takes no
eGFR
mls/min/1.73m2 110 medication
50-
Creatinine 128 micromol/l
100 On examination, he has a BMI of 32.2 kg/m2, blood
Albumin/creatinine pressure of 132/78 mmHg and he has central
4.1 mg/mmol <3.5 adiposity. A repeat fasting plasma glucose returns
ratio
as 6.1 mmol/l (3.5-6).
Fasting plasma 3.5-
6.1 mmol/l
glucose 6
You give the patient advice concerning the
importance of diet and exercise with emphasis
Which of the following would be the most placed on weight loss.
appropriate approach for managing this lady’s
blood glucose? Which of the following would you suggest
(Please select 1 option) regarding further investigation of his glucose
1- Glibenclamide concentration?
(Please select 1 option)
2- Gliclazide
1- Arrange oral glucose tolerance test
3- Metformin
2- Check HbA1c
4- Pioglitazone
3- Check fructosamine
5- Persist with current lifestyle changes alone
4- No further investigation required
Answer & Comments 5- Repeat fasting glucose in 3 months
Answer: 3- Metformin
Answer & Comments
The current NICE guideline update on blood Answer: 1- Arrange oral glucose tolerance test
glucose control (CG 87, May 2009) advocates the
use of metformin as a first line oral hypoglycaemic
This patient has impaired fasting glucose
agent in people with suboptimal glycaemic control
according to the IDF criteria with a glucose at or
above 6.1 mmol/l and below 7.1 mmol/l. The and her pregnancy is progressing normally.
recommendation would be for an OGTT with up to According to NICE guidelines, what is the most
30% of such cases having a diagnosis of diabetes appropriate time for her Gestational Diabetes
on OGTT. He may have the metabolic syndrome Mellitus (GDM) screening?
although we are not told his waist circumference
(Please select 1 option)
which is a criterion measurement.
1- As early as possible
[ Q: 435 ] Question ID #361 2- At 24 weeks
3- At 28 weeks
A 60 year old male with a 10 year history of type 2
diabetes was referred by his GP for conversion to 4- At 36 weeks
insulin. He was taking metformin 850 mg tds,
5- No indication to check for GDM
gliclazide 160 mg bd and pioglitazone 45 mg od.
He does not smoke and denies any alcohol
consumption. Answer & Comments
Answer: 1- As early as possible
On examination, BMI is 41.2 kg/m2, blood pressure
142/68 mmHg and abdominal examination reveals
2 cm smooth non-tender hepatomegaly. According to NICE guidelines on gestational
diabetes (NICE clinical guidelines 63), the following
In type 2 diabetes, elevations of which liver groups of patients should be screened for GDM:
enzymes are most correlated with non-alcoholic
fatty liver disease? Previous history of GDM
Certain ethnic groups (e.g. south east
(Please select 1 option)
Asians, black Caribbean and middle
1- AST alone eastern)
Previous birth weight of a child > 4.5 Kg
2- ALT and AST
Body mass index > 30 kg/m2
3- ALT and gamma-GT Family history of a first degree relative
with GDM
4- AST and gamma-GT
5- gamma-GT and alkaline phosphatase Screening for GDM in the above mentioned high
risk group patients should be arranged at the
booking appointment.
Answer & Comments
Answer: 3- ALT and gamma-GT [ Q: 437 ] Question ID #475
Up to 20% of patients with T2DM have changes of Which of the following oral hypoglycaemic agents
fatty liver. Elevated concentrations of ALT more is associated with Vitamin B12 deficiency?
than AST (unlike alcoholic liver disease where AST (Please select 1 option)
is usually higher) is typical and gamma-GT is
usually moderately elevated. Bilirubin and alkaline 1- Acarbose
phosphatase are usually normal. 2- Gliclazide
3- Glibenclamide
[ Q: 436 ] Question ID #473
4- Metformin
An Asian lady with previous history of gestational 5- Pioglitazone
diabetes presents around 16 week pregnant in
antenatal clinic. She is otherwise asymptomatic
1- 1%
According to the NICE guidelines, patients should
2- 1-2% be considered to be at high risk for cardiovascular
disease if one or more of the following are
3- 3-4%
present:
4- 5-10%
Increased BMI
5- 20-25%
Hypertensive (> 140/80 mm of Hg)
Microalbuminuria
Answer & Comments Smoker
Deranged lipid profile
Answer: 2- 1-2% Past medical history of cardiovascular
disease
The following table sums up the risk of Family history of cardiovascular disease
development of Type 1 Diabetes in a child with
family history of Type 1 Diabetes: A cardiovascular risk of >10% on QRISK2 warrants
therapy with statin therapy (atorvastatin as
Type of DM Type 1 recommended by NICE).
2- Check fasting and 2-hours post meal blood or haemorrhage. This gentleman has suboptimal
sugars glycaemic control as reflected by his home blood
sugar readings but his HbA1C is inappropriately
3- Check fasting and 1-hour post prandial blood low as a result of recent blood loss. As a result his
sugars insulin dose needs to be increased to control
4- Check blood sugars twice a day impaired home blood sugar readings.
5- Check blood sugar every alternate day In contrast falsely elevated HbA1c values can be
seen in uraemia (Carbamoylated Hb) or in
Answer & Comments presence of Hb F.
A 44 year old gentleman with a body mass index A 25 year old woman with a 7 year history of type
of 47kg/m2 presents to the clinic with poorly 1 diabetes presents to clinic being 8 weeks
controlled Type 2 DM. He has multiple co- pregnant. Her glycaemic control has been
morbidities including sleep apnoea, hypertension excellent with a HbA1c of around 6-6.5% on basal
and osteoarthritis. bolus insulin. She enquiries about possible risk of
major congenital malformations in her offspring.
According to NICE guidelines what is the next best
step in his management? What are the chances of major congenital
(Please select 1 option) malformation in her child?
(Please select 1 option)
1- Bariatric surgery
1- 1%
2- Exenetide
2- 1%
3- Insulin
3- 5-7%
4- Orlistat
4- 10%
5- Sibutramine
5- Same as background population
Answer & Comments
Answer & Comments
Answer: 1- Bariatric surgery
Answer: 5- Same as background population
According to NICE guidelines on management of
overweight and obese adults (NICE clinical The risk of congenital abnormalities and perinatal
guidelines 43), consider surgery for people with mortality is increased in babies born to diabetic
severe obesity if: mothers by two- folds and three -folds
respectively. A good glycaemic control around
They have a BMI of 40 kg/m2 or more, or conception and early pregnancy can reduce the
between 35 kg/m2 and other significant incidence of both the above mentioned
disease ( for example, type 2 diabetes, complications. The major congenital
high blood pressure) that could be abnormalities detected in neonates include
improved if they lost weight congenital heart disease (x 3.4 times higher risk)
All appropriate non-surgical measures and neural tube defects (x3-4 times higher risk)
have failed to achieve or maintain (1). According to the results from The Diabetes
adequate clinically beneficial weight loss Control and Complications Trial (DCCT) a good
for at least 6 months glycaemic control is associated with rates of
They are receiving or will receive intensive spontaneous abortion and congenital
specialist management malformations similar to those seen in non
They are generally fit for anaesthesia and diabetic population (2).
surgery
They commit for long term follow up Reference:
Consider surgery as a first line option for Perinatal mortality and congenital anomalies in
adults with a BMI > 50 kg/m2 in whom babies of women with Type 1 or Type 2
surgical intervention is considered Diabetes in England, Wales and Northern
appropriate; consider Orlistat before Ireland: population based study by Macintosh
surgery if the waiting time is long et al., BMJ 2006;333:177(22 July)
Pregnancy outcomes in the Diabetic Control and 4- Fasting and before every meal
Complication Trial - Am J Obstetrics and
5- Once a day
Gynaecology, 1996 Apr;174(4):1343-53
A 25 year old male is recently diagnosed to have Which of the following test is likely to reveal his
Diabetes Mellitus based on routine screening. His underlying diagnosis?
BMI is 24kg/m2 and there are no ketone bodies in (Please select 1 option)
urine with only 2+ sugars. His recent ultrasound
abdomen did not show any abnormalities. He has 1- Anti GAD antibodies
strong family history of Type 2 DM with few of the 2- Anti islet cells antibodies
family members requiring insulin therapy for
control. 3- Insulin, C peptide levels
4- Renin levels
What's the likely underlying genetic mutation?
(Please select 1 option)
5- Screening for monogenetic diabetes
1- Glucokinase
Answer & Comments
2- HNF 1α
Answer: 5- Screening for monogenetic diabetes
3- HNF 4α
4- IPF The patient has MODY 5, which is secondary to
mutation of HNF-1? and is characterised by onset
5- NeuroD1
of diabetes in the third decade and association
with renal cysts or uterine abnormalities in
Answer & Comments females.
Answer: 2- HNF 1α
Albumin 38 g/L (30-50 g/L) A 45 year old man with Type 2 DM for 8 years
(50-125 presents to clinic with lack of libido and erectile
ALP 175 U/L dysfunction. His BMI is 32 kg/m2 and there is no
U/L)
loss of secondary sexual characteristics. He denies
ALT 125 mU/L (05-58 U/L)
usage of any substance of abuse and his alcohol
35 intake is 14 units in a week. His biochemistry
Bilirubin (0-18)
micromol/L shows:
Proteins 66 g/L (60-80 g/L)
(10-300 FSH 2.2 U/L (1-10)
Ferritin 550 pmol/L
ng/mL) LH 3.5 U/L (3-10)
Transferrin Testosterone 7.5 nmol/L (9-28.5)
52% (15-50%)
saturation Prolactin 350 mU/L (45-400)
HbA1c 8.2% (<6) LFT Normal
Hb 12.8 g/L (12-16)
He undergoes a MRI which shows no obvious
Which of the following is the likely underlying abnormality.
diagnosis?
(Please select 1 option) What is the likely explanation for his low libido?
(Please select 1 option)
1- Glycogen storage disease
1- Anabolic steroid usage
2- Haemochromatosis
2- Haemochromatosis
3- LADA
3- Kallmann syndrome
4- Mitochondrial disease
4- Kleinfelter`s syndrome
5- MODY
5- Obesity
Answer & Comments
Answer & Comments
Answer: 2- Haemochromatosis
Answer: 5- Obesity
Haemochromatosis (Bronze diabetes) is a
hereditary (HFE/haemojvelin/hepcidin and other Erectile dysfunction is defined as inability to
gene mutations) or acquired (e.g. secondary to achieve or maintain an erection for sufficient time
repeated blood transfusions) condition leading to duration and firmness resulting in incomplete
pathological effects of iron accumulation on sexual activity in > 25% of the attempts. About
multiple organs and can present with one or more 10% of men are affected by low libido and erectile
of the following features: dysfunction and the prevalence increases with
age.
Deranged LFT/cirrhosis
Diabetes due to pancreatic β cell failure The following is a list of main causes for erectile
Hypogonadism dysfunction:
Skin pigmentation
What is the most appropriate step in her 3- The risk of colon cancer is increased 2.3 fold
management? in patients on insulin glargine.
(Please select 1 option) 4- There is an increased risk of colon cancer for
1- Dietary advice patients taking insulin but not
sulphonylureas.
2- Intensify the control
5- There is no evidence to suggest
3- Remove insulin pump
sulphonylureas increase the risk of cancer.
4- Replace the tubing of pump
5- Offer more education about Insulin pump Answer & Comments
use
Answer: 1- Population-based cohort studies
support her concerns, showing a higher
Answer & Comments cancer-related mortality in those treated with
insulin or sulphonylureas but not metformin.
Answer: 5- Offer more education about Insulin
pump use
Therapies used to treat patients with diabetes
may increase or decrease their cancer risk.
This lady is on insulin pump and her glycaemic
Population-based cohort studies support this,
control is suboptimal, reflecting a lack of
showing a higher cancer-related mortality in those
monitoring and awareness on her part. In this case
treated with insulin or sulphonylureas compared
it is so important to get this woman to understand
with patients on metformin therapy.
the importance of glycaemic control and
adherance t the pump. She should be offered
further education regarding carbohydrate [ Q: 461 ] Question ID #847
counting, regular blood sugar monitoring and
adjustment of insulin pump basal rate. A 49-year old barrister with a 2 year history of type
2 diabetes mellitus, controlled on diet, attends
[ Q: 460 ] Question ID #846 clinic asking to be prescribed metformin. She has
a family history of colorectal cancer, which has
affected both her parents and a maternal aunty.
A 52 year old solicitor with a 7 year history of type
She has read an article that metformin maybe
2 diabetes presents to clinic concerned about her
protective against cancer in diabetes.
treatment. She is on gliclazide 160mg b.i.d., and
insulin glargine 24 units taken in the evening. She
Which of the following is correct?
has read there may be an increased risk of cancer
associated with her diabetic medications. (Please select 1 option)
Metformin may have a protective effect against The greater incidence of breast, pancreatic and
cancer and pilot studies from a diabetes database colon cancer in patients with type 2 diabetes may
support this. The potential mechanism is that be multifactorial, possibly linked to obesity, insulin
metformin activates adenosine monophosphate- resistance or hyperglycaemia. For example,
activated protein kinase which may suppress carcinoma of the colon is associated with obesity,
tumour formation. hyperglycaemia, metabolic syndrome,
hypertriglyceridaemia, insulin treatment and
raised insulin-like growth factor levels.
[ Q: 462 ] Question ID #848
1- She should not believe what she reads in the 2- In a head-to-head comparative study against
Sunday press? exenatide, liraglutide was associated with a
0.3% superior HbA1c reduction.
2- Type 2 diabetes is associated with a greater
incidence of cancer, particularly of the 3- In a head-to-head comparative study against
pancreas, breast and colon. exenatide, liraglutide was associated with a
0.9% superior HbA1c reduction.
3- Type 2 diabetes is associated with a greater
incidence of cancer, particularly of the 4- In an open-labelled study exenatide has
pancreas, lung and brain. demonstrated a 0.4% superior HbA1c
reduction compared to liraglutide.
A 58 year old patient with a 14 year history of type Patients with T2DM are often reluctant to begin
2 diabetes presents to clinic. His HbA1c is 13.6% insulin and, in many cases, delay the start of
despite taking metformin 1g bd., gliclazide 160mg insulin therapy for quite lengthy periods of time.
bd., and exenatide 10mcg bd. His BMI is 35 kg/m2, This has led to the development of the concept of
his blood pressure is 164/98 mmHg and he has “psychological insulin resistance” (PIR).
pre-proliferative retinopathy, microalbuminuria,
and a distal sensori-motor neuropathy. While clinical lore suggests that PIR is quite
common, there has been little formal study in this
He refuses to consider insulin therapy and is not area. In the United Kingdom Prospective Diabetes
keen on attending a recommended EXPERT Study (UKPDS), of those with T2DM randomized to
patient education programme as he doesn’t like insulin therapy, 27% initially refused. Early reports
talking to other “diabetics”. You diagnose him with from the international Diabetes Attitudes, Wishes,
‘Psychological Insulin Resistance.’ and Needs (DAWN) study indicate that the
majority (54.9%) of insulin-naïve patients worry
Which factor constitute Psychological Insulin about the possibility of insulin therapy. The six
Resistance? features of PIR are as listed in answer C, which is
the best answer.
(Please select 1 option)
Answer: 3- 50%
Answer & Comments
The fifth factor in the concept of psychological Answer: 3- Body fat stores and insulin action are
insulin resistance (PIR) concerns injection related controlled by the temporal interaction of
anxiety. Approximately 50% of patients report circadian neuroendocrine oscillations.
being fearful of injections. Although this is often Bromocriptine modulates neurotransmitter
presumed to be the single, or single largest, action in the brain and is associated with
contributor to PIR, it may be overstated. True HbA1c reductions of 0.56%.
injection phobia is rare, even among insulin-using
patients with diabetes.
Body fat stores and insulin action are controlled by
the temporal interaction of circadian
[ Q: 466 ] Question ID #852 neuroendocrine oscillations. Bromocriptine
modulates neurotransmitter action in the brain.
A 72 year old American tourist with a 4 year Studies using bromocriptine in type 2 diabetes
history of type 2 diabetes is admitted to the acute have shown improvements in glycaemic control
assessment unit with haematemesis. The House- and glucose tolerance with HbA1creductions of
Officer / Foundation Year 1 doctor admits the ~0.56%.
patient and erroneously informs you that the
management of the patients’ diabetes is ‘diet- Despite receiving FDA approval, bromocriptine is
control’. not part of the type 2 diabetes management
guidelines from the National Institute for Health
Upon reviewing the drug chart you note the and Clinical Excellence. It is commonly used in the
patient is on bromocriptine. management of T2DM in the USA.
1- Her symptoms are most likely related to 1- Exenatide acts in a glucose dependent
hayfever in the summer months. manner. Therefore there is negligible risk of
hypoglycaemia.
2- Sitagliptin is associated with nasopharyngitis
and may be contributory. 2- The risk of hypoglycaemia is increased in
patients on exenatide who are also taking a
3- She should be referred to an allergy clinic in
sulphonylurea.
view of her past medical history of atopy.
3- The risk of hypoglycaemia is increased in
4- She should be prescribed an anti-histamine
patients on exenatide who are also taking
and nasal decongestants.
metformin.
5- She should monitor her PEFR twice daily as
4- The risk of hypoglycaemia is increased in
her asthma may have decompensated.
patients on exenatide who are also taking a
thiazolidienedione.
Answer & Comments
5- The risk of hypoglycaemia in decreased if
Answer: 2- Sitagliptin is associated with exenatide is administered one hour before
nasopharyngitis and may be contributory. oral hypoglycaemic agent therapy.
You look back through her medical records and Answer & Comments
note that she has a history of pelvic inflammatory
disease. A urinary Chlamydia antigen test was Answer: 3- Grade 3
positive. An ultrasound scan of her pelvis was
reported as showing bilateral renal cysts and a Wagner’s classification of diabetic foot lesions is as
unicornuate uterus. She also has a history of gout follows:
and is known to binge drink.
Grade 0 – high-risk foot, no ulcer present.
What is the underlying diagnosis? Grade 1 – Superficial ulcer, not infected.
(Please select 1 option) Grade 2 – deep ulcer with or without
cellulitis but no abscess or bone
1- Type 1 diabetes involvement.
2- Polycystic ovarian syndrome Grade 3 – deep ulcer with bone
involvement or abscess formation.
3- MODY 1 Grade 4 – localised gangrene (toe,
4- MODY 3 forefoot, heel).
Grade 5 – gangrene of the whole foot.
5- MODY 5
[ Q: 471 ] Question ID #857
Answer & Comments
Answer: 5- MODY 5 A 25 year old model with a 17 year history of type
1 diabetes presents to clinic concerned about her
finger nails. She has noticed an area of redness
MODY 5 is due to a mutation in HNF1-ß and over her nail folds, which have been present for
accounts for 3% of cases of MODY. The average the last month. She is concerned as this may affect
age of presentation is 22 years associated with her employment prospects.
renal cysts often with uterine abnormalities, gout
and insulin resistance. What is the most likely diagnosis?
(Please select 1 option)
[ Q: 470 ] Question ID #856
1- Bullosis diabeticorum
A 74 year old patient with a 28 year history of type 2- Diabetic thick skin
2 diabetes presents to the diabetic foot clinic with
3- Folliculitis
a deep ulcer on his right forefoot. The podiatrist is
able to prod down to bone and there is clinical 4- ANCA negative small vessel vasculitis
evidence of abscess formation.
5- Periungual telangectasia
According to the Wagner classification of diabetic
foot lesions, what grade ulcer does this patient Answer & Comments
have?
Answer: 5- Periungual telangectasia
(Please select 1 option)
commonly in those with diabetes but are not What is the average delay between starting oral
specific to diabetes. hypoglycaemic therapy to initiating insulin?
(Please select 1 option)
[ Q: 472 ] Question ID #858
1- 2 years.
Despite the availability of many anti-diabetic capsule form by people with diabetes. Many
agents, approximately 60% of patients do not natural products and herbal medicines are not
achieve the target HbA1C level of <7%. Reasons safe when used in conjunction with other
for this include: noncompliance; side-effects of pharmaceutical medicines and can cause
treatments; fear of hypoglycaemia; weight-gain; problems.
problems with dose titration of anti-diabetic
agents; and more stringent HbA1C targets set by
[ Q: 476 ] Question ID #3454
healthcare organisations which tend to change.
You review a 45 year old ecologist who has been Risk from which of the following is increased with
found to have fasting plasma glucose of 8.4 the use of statin therapy?
mmol/L and 9.1 mmol/L on two occasions. She has
(Please select 1 option)
not lost weight and her GAD antibodies are
negative. Her HbA1c is 7.3%, her blood pressure is 1- Diabetes mellitus
130/78mmHg and her BMI is 29 kg/m 2.
2- Erectile dysfunction
You diagnose her with type 2 diabetes with the 3- Immune hypoadrenalism
recommendation that she commences metformin
4- Ischaemic heart disease
500mg b.i.d. in the first instance. She refuses
treatment preferring to use herbal medicines. She 5- Multiple sclerosis
has read that Karela supplements are of value in
type 2 diabetes and would like to try them first.
Answer & Comments
How do Karela supplements benefit patients with Answer: 1- Diabetes mellitus
type 2 diabetes?
(Please select 1 option) Statins have been shown through meta-analyses
1- Karela reduces peripheral insulin resistance. to be associated with a 23% increased risk of new
onset diabetes and the FDA warned of this
2- Karela inhibits hepatic gluconeogenesis. phenomenon in 2012 following results of the
JUPITER study. However, statins are without
3- Karela inhibits free fatty acid oxidation.
question drugs that reduce CV risk and appear to
4- Karela has hypoglycaemic effects. offer beneficial effects in ED.
5- Karela has incretin like effects.
Journal of Pharmacotherapy and
pharmacotherapeutics (2014)Statin induced
Answer & Comments diabetes and its clinical implications
Answer: 4- Karela has hypoglycaemic effects.
[ Q: 477 ] Question ID #3455
The hypoglycaemic effects of Karela are well
documented. It is used traditionally in Asian A 55 year old man with type 2 diabetes is being
cookery and as a traditional medicine to lower treated with metformin and gliclazide. His HbA1c
blood glucose levels. In practice it is difficult to is 76 mmol/mol (9.1%). It is decided to commence
take enough of the Karela plant to have a him on empagliflozin.
significant effect. Karela capsules are an unknown
entity. While certain components of Karela do Which of the following describe this class of drug?
have hypoglycaemic effects there have been no (Please select 1 option)
formal studies to indicate if it can be used safely in
1- Dipeptidyl peptidase –IV inhibitor
Which of the following were the reported results of The New England Journal of Medicine, Zinman B
EMPA-REG? et al (2015) Empagliflozin, Cardiovascular
(Please select 1 option) Outcomes, and Mortality in Type 2 Diabetes
Endocrinology
Endocrinology and Diabetes - Mock
Exam
A 24 year old female has received a six month course of carbimazole for Graves disease. She recently stopped
treatment and attends two months later for subsequent review and complains of recurrence of her previous
symptoms. On examination, she has a modest goitre with pulse of 80 bpm and has a fine tremor of the
outstretched hands. The results of her thyroid function tests reveal:
She informs you that she is keen on getting pregnant at some stage in the future but currently takes the oral
contraceptive, microgynon.
Which of the following would be the most appropriate action for this patient?
(Please select 1 option)
The most appropriate first line treatment for this patient who has recurrent thyrotoxicosis following anti-
thyroid treatment would be radioactive iodine. She should ensure that she does not become pregnant for
four months after treatment but she is already taking the OCP. There is no report of eye disease associated
with the Graves’ so concomitant steroids is probably unnecessary. Following radioiodine treatment,
approximately 80% of patients become hypothyroid.
A 71 year old gentleman is admitted to hospital following admission to hospital with a mechanical fall. On
admission, his random plasma glucose is 12.4 mmol/l (<7.8) and a subsequent fasting plasma glucose
measures 7.9 mmol/l (<6). He is generally well, taking no regular medications and on examination his blood
pressure is 146/86 mmHg and his BMI is 31 kg / m2.
Which of the following is the most important determinant of the future risk of cardiovascular events in this
gentleman?
(Please select 1 option)
The results of the Strong Heart Study indicate that non-HDL-C may be a superior predictor of cardiovascular
events than LDL-C; HDL-C or plasma TG levels alone in both men and women with type 2 diabetes. The utility
of non-HDL-C in predicting cardiovascular events in non-diabetic subjects is supported by data from the SHEP
(Systolic Hypertension in the Elderly Program) and Lipid Research Clinics (LRC) studies, in which non-HDL-C
was found to be a better predictor than LDL-C of cardiovascular events in patients followed over 4.5 years
and 19 years respectively.